Download as pdf or txt
Download as pdf or txt
You are on page 1of 103

VISIONIAS

www.visionias.in

Test Booklet Series

TEST BOOKLET

GENERAL STUDIES (P) 2024 – Test – 4144


C
Time Allowed: Two Hours Maximum Marks: 200

INSTRUCTIONS

1. IMMEDIATELY AFTER THE COMMENCEMENT OF THE EXAMINATION, YOU SHOULD CHECK THAT THIS BOOKLET
DOES NOT HAVE ANY UNPRINTED OR TURN OR MISSING PAGES OR ITEMS, ETC. IF SO, GET IT REPLACED BY A
COMPLETE TEST BOOKLET.

2. ENCODE CLEARLY THE TEST BOOKLET SERIES A, B, C OR D AS THE CASE MAY BE IN THE APPROPRIATE PLACE IN
THE ANSWER SHEET.

3. You have to enter your Roll Number on the Test Booklet in the Box
provided alongside. Do NOT write anything else on the Test Booklet.

4. This Test Booklet contains 100 items (Questions). Each item is printed in English. Each item comprises four
responses (answers). You will select the response which you want to mark on the Answer Sheet. In case you
feel that there is more than one correct response with you consider the best. In any case, choose ONLY ONE
response for each item.

5. You have to mark all your responses ONLY on the separate Answer Sheet provided. See direction in the
answers sheet.

6. All items carry equal marks. Attempt all items. Your total marks will depend only on the number of correct
responses marked by you in the answer sheet. For every incorrect response 1/3rdof the allotted marks will be
deducted.

7. Before you proceed to mark in the Answer sheet the response to various items in the Test booklet, you have to
fill in some particulars in the answer sheets as per instruction sent to you with your Admission Certificate.

8. After you have completed filling in all responses on the answer sheet and the examination has concluded, you
should hand over to Invigilator only the answer sheet. You are permitted to take away with you the Test
Booklet.

9. Sheet for rough work are appended in the Test Booklet at the end.

DO NOT OPEN THIS BOOKLET UNTIL YOU ARE ASKED TO DO SO


1 www.visionias.in ©Vision IAS
1. Consider the following statements with 4. Consider the following statements regarding
respect to the Green Hydrogen Mission of the Water (Prevention and Control of
India: Pollution) Act,1974:
1. It aims to make India a net exporter of 1. Under this act, a cess is levied and
green hydrogen. collected on water consumed by persons
2. It targets to build capabilities to produce operating and carrying on certain types
at least 5 million metric tonnes (MMT) of industrial activities.
2. In 2022, the government amended rules
of green hydrogen per annum by 2030.
under the act to add regulations on the
3. NITI Aayog will be responsible for the
use of water purification systems.
overall coordination and implementation
Which of the statements given above is/are
of the mission.
correct?
How many of the statements given above are
(a) 1 only
correct?
(b) 2 only
(a) Only one
(c) Both 1 and 2
(b) Only two (d) Neither 1 nor 2
(c) All three
(d) None 5. Consider the following:
1. Biodiversity loss
2. With reference to the carbon credit, consider 2. International waters
the following statements: 3. Chemicals and waste
1. One carbon credit is equal to one metric 4. Climate change
ton of carbon dioxide, or in some 5. Renewable energy
markets, carbon dioxide equivalent How many of the above are included in the
gases. Five Focal Areas of the World Environment
2. It is based on the cap-and-trade model. Facility (WEF)?
Which of the statements given above is/are (a) Only two
correct? (b) Only three
(a) 1 only (c) Only four
(d) All five
(b) 2 only
(c) Both 1 and 2
6. Consider the following statements regarding
(d) Neither 1 nor 2
nuclear energy in India:
1. It contributes less than 1% of the total
3. Which of the following are the sources of
electricity generation in the country.
indoor air pollution?
2. India has over 50 nuclear reactors across
1. Phthalate esters the country which produces around
2. Ozone 10,000 MW of nuclear power.
3. Radon 3. Over two-third of the reactors in India
4. Animal dander are Pressurised Heavy Water Reactors.
Select the correct answer using the code How many of the above statements are
given below. correct?
(a) 1, 2 and 3 only (a) Only one
(b) 2 and 4 only (b) Only two
(c) 1, 3 and 4 only (c) All three
(d) 1, 2, 3 and 4 (d) None
2 www.visionias.in ©Vision IAS
7. Hydrodynamic trapping, solubility trapping 10. Arrange the following gases according to
and mineral carbonation are associated with: their global warming potential in ascending
(a) In situ bioremediation order.
(b) Carbon Sequestration
1. Nitrous Oxide
(c) Decomposition of solid waste
2. Methane
(d) Shale gas extraction
3. Sulphur hexafluoride
4. Carbon dioxide
8. Which of the following are the components
Select the correct answer using the code
of the Global Solar Facility under
International Solar Alliance (ISA)? given below.

1. Solar Payment Guarantee Fund (a) 2-4-3-1


2. Solar Insurance Fund (b) 4-1-2-3
3. Global Energy Alliance for People and (c) 4-2-1-3
Planet (d) 2-1-4-3
4. Scaling Solar Applications for
Agriculture Use
11. Consider the following statements regarding
Select the correct answer using the code
biochar:
given below.
1. It is a carbon-enriched biomaterial
(a) 1 and 2 only
generated through incineration.
(b) 1, 3 and 4 only
(c) 3 and 4 only 2. It enriches the soil and prevents it from

(d) 1, 2, 3 and 4 leaching pesticides.


3. It is also an excellent carbon sink and
9. Consider the following statements regarding can be used for carbon sequestration.
Resilient and Inclusive Supply-chain Which of the statements given above is/are
Enhancement (RISE):
correct?
1. It is an initiative of the Organisation for
(a) 1 only
Economic Co-operation and
(b) 2 and 3 only
Development.
(c) 3 only
2. It will help emerging markets and
(d) 1, 2 and 3
developing countries (EMDCs) increase
the manufacturing of clean-energy
products. 12. Which of the following greenhouse gases
Which of the statements given above is/are have the highest atmospheric lifetime in
correct? years?
(a) 1 only (a) Methane
(b) 2 only
(b) Nitrous Oxide
(c) Both 1 and 2
(c) Hydrofluorocarbons
(d) Neither 1 nor 2
(d) Chloroflorocarbons
3 www.visionias.in ©Vision IAS
13. Carbon trading as a concept was initially 16. In the context of the environment, ‘Carbon
introduced under which of the following Leakage’ is best described as:
international agreements/summits? (a) It is the expenses incurred by the nation
(a) Kyoto Protocol, 1997 for carbon emission trading.
(b) Montreal Protocol, 1987 (b) It is an increase in emissions outside a
(c) Earth Summit, 1992 region as a direct result of the policy
(d) Paris Agreement, 2015 measures to cap emissions in the region.
(c) It is the excess of carbon emissions by a
14. Consider the following statements with nation that exceeds prescribed limit even
respect to the Inclusive Conservation after acquiring carbon credits from other
Initiative: nations.
1. It is an initiative to support the
(d) It is a tax evasion mechanism used by
leadership of indigenous peoples and
emitters to avoid paying carbon tax for
local communities in stewarding land,
greenhouse gas emissions.
water and natural resources.
2. It will provide direct financial support to
17. Consider the following statements with
indigenous and locally-led initiatives in
respect to 'Jaivik Pahal' campaign:
Africa, Central and South America, Asia
1. It is the campaign launched by the
and the Pacific region.
automobile industry in India.
3. It is a joint initiative of the United
2. It promotes biofuels produced from
Nations and Conservation International.
renewable biological materials.
How many of the statements given above are
Which of the statements given above is/are
correct?
correct?
(a) Only one
(a) 1 only
(b) Only two
(b) 2 only
(c) All three
(d) None (c) Both 1 and 2
(d) Neither 1 nor 2

15. Consider the following pairs:


GI Tag Product State 18. In the context of space science, consider the

1. Udangudi : Karnataka following statements regarding neutrinos:


Panangarupatti 1. They never interact with any substance.
2. Tangsa Textile : Sikkim 2. They are nearly massless.
3. Khamti Rice : Arunachal Pradesh 3. They do not have any charge.
How many of the pairs given above are How many of the statements given above are
correctly matched? correct?
(a) Only one (a) Only one
(b) Only two (b) Only two
(c) All three (c) All three
(d) None (d) None
4 www.visionias.in ©Vision IAS
19. Consider the following statements regarding 22. With reference to the global climate finance
the Ramsar Convention of 1971: regime, consider the following statements
1. It is a legally binding intergovernmental
regarding the Least Developed Countries
treaty that provides the framework for
Fund (LDCF):
the conservation of wetlands.
1. It was created by the Marrakesh Accords
2. Lotic water bodies are not covered under
the Ramsar Convention. in the 17th conference of parties in 2011.

3. India has the largest network of Ramsar 2. It aims to address the needs of the 51
wetlands in Asia. LDCs which are particularly vulnerable
How many of the above statements are to the adverse impacts of climate
correct?
change.
(a) Only one
3. It supports the preparation and
(b) Only two
implementation of the National
(c) All three
(d) None Adaptation Programmes of Action.

How many of the statements given above are


20. Consider the following statements: correct?
1. E-waste is categorized as hazardous
(a) Only one
waste under the Basel Convention,1989.
(b) Only two
2. India is the world's largest e-waste
(c) All three
producer as per the Global E-waste
Monitor 2020. (d) None

Which of the statements given above is/are


correct? 23. The India GHG Program is:
(a) 1 only (a) an initiative by the Ministry of Science
(b) 2 only
and Technology to develop climate
(c) Both 1 and 2
change mitigation technologies.
(d) Neither 1 nor 2
(b) an initiative by the Ministry of

21. Recently, which of the following villages Environment, Forest and Climate

has been selected as Best Tourism Village Change to reduce greenhouse gas
by the United Nations World Tourism emissions.
Organization (UNWTO)?
(c) an industry-led voluntary framework to
(a) Dhordo
measure and manage greenhouse gas
(b) Mawlynnong
emissions.
(c) Malana
(d) Odanthurai (d) None of the above.

5 www.visionias.in ©Vision IAS


24. In the context of Nanophotonic Electron 26. With reference to 'Zero Budget Natural
Accelerator (NEA), recently seen in the Farming', a term often seen in news, which
of the following statements are correct?
news, consider the following statements:
1. It is a method of farming done without
1. It is the world's smallest particle
using any credit, and without spending
accelerator which has been turned on for any money on purchased inputs.
the first time. 2. It aims to eliminate the use of chemical
2. One of the key goals of developing NEA pesticides.
3. It is not practiced in India.
is to exploit the energy released by the
Select the correct answer using the code
use of miniature particle accelerators for
given below.
medicinal purposes. (a) 1 and 2 only
3. The NEA uses light beams focused at (b) 2 and 3 only
the vacuum tube's pillars to magnify (c) 1 and 3 only

energy, resulting in a very low energy (d) 1, 2 and 3

field.
27. The Worldwide Governance Indicators
How many of the statements given above are (WGI) is often seen in the news. It is an
correct? initiative of
(a) Only one (a) Inter-Parliamentary Union
(b) Only two (b) Organisation for Economic Co-operation
and Development
(c) All three
(c) World Bank
(d) None
(d) United Nations

25. Consider the following statements with 28. Consider the following statements regarding
respect to E-waste Rules (Management), the United Nations Environment Assembly
(UNEA):
2022:
1. It was founded as a result of the UN
1. These rules do not apply to waste
Conference on the Human Environment
batteries and radioactive wastes. (Stockholm Conference) in 1972.
2. It defines an ‘e-retailer’ as a company 2. It is the world’s highest-level decision-
which deals in e-waste online. making body with near-universal
membership for matters related to the
3. It defines 'orphaned products’ as those
environment.
produced more than five years ago by a
3. The session of assembly is conducted
branded company. every year at the UNEP headquarters in
How many of the statements given above are Nairobi, Kenya.
correct? How many of the statements given above are
correct?
(a) Only one
(a) Only one
(b) Only two
(b) Only two
(c) All three (c) All three
(d) None (d) None
6 www.visionias.in ©Vision IAS
29. Consider the following sources of energy: 32. With reference to the 'Ethanol Blending
1. Wind energy programme (EBP) in India', consider the
2. Nuclear energy following statements:
3. Solar energy 1. India has advanced the target of 20
4. Biomass energy percent ethanol blending in petrol from
5. Geothermal energy
2030 to 2025-26.
How many of the above are included in the
2. Department of Food and Public
renewable energy sector in India?
Distribution (DFPD) is the nodal
(a) Only two
(b) Only three department for the promotion of fuel-
(c) Only four grade ethanol-producing distilleries in
(d) All five the country.
Which of the statements given above is/are
30. Consider the following statements with correct?
respect to the Carbon Offsetting and (a) 1 only
Reduction Scheme for International Aviation (b) 2 only
(CORSIA): (c) Both 1 and 2
1. It is the first global market-based
(d) Neither 1 nor 2
measure for any sector that offers a
harmonized way to reduce emissions
33. Consider the following:
from international aviation.
2. Unlike the pilot phase, the participation 1. Increase in Research Octane Number
of states is mandatory in its final phase. 2. Higher flame speed
Which of the statements given above is/are 3. Lesser fuel requirement for the same
correct? engine output
(a) 1 only How many of the above are the benefits of
(b) 2 only ethanol blending?
(c) Both 1 and 2 (a) Only one
(d) Neither 1 nor 2 (b) Only two
(c) All three
31. Consider the following statement with
(d) None
respect to ozone-depleting substances:
1. Montreal Protocol and Kigali
Amendment, both address the issues 34. Which of the following are a part of the
related to ozone-depleting substances. biological treatment of wastewater
2. The Kigali amendment aims to reduce containing hazardous chemicals?
HFCs by 80-85 per cent by the late 1. Neutralisation
2040s. 2. Composting
3. India has successfully met the phase-out 3. Conditioning
targets of all the Ozone Depleting 4. Digestion
Substances as per the Montreal Protocol 5. Lagooning
Schedule.
Select the correct answer using the code
How many of the statements given above are
given below.
correct?
(a) 1 and 5 only
(a) Only one
(b) 2, 3 and 5 only
(b) Only two
(c) All three (c) 2, 3 and 4 only
(d) None (d) 1, 2, 3, 4 and 5
7 www.visionias.in ©Vision IAS
35. Consider the following National Missions 38. Consider the following statements regarding
being implemented in India: the Convention on International Trade in
1. National Mission on Natural Farming Endangered Species of Wild Fauna and
(NMNF) Flora (CITES):
2. National Mission on Strategic 1. It is a non-binding international
Knowledge for Climate Change agreement between governments to
(NMSKCC) regulate international trade in wild
3. National Mission For Enhanced Energy animals and plants.
Efficiency 2. Its secretariat is administered by the
United Nations Environment Programme
4. National Mission on Sustainable Habitat
(UNEP).
How many of the above are not included in
Which of the statements given above is/are
India's National Action Plan On Climate
correct?
Change (NAPCC)?
(a) 1 only
(a) Only one
(b) 2 only
(b) Only two
(c) Both 1 and 2
(c) Only three
(d) Neither 1 nor 2
(d) None
39. Consider the following statements regarding
36. Consider the following statements regarding the International Whaling Commission
the Global Framework for Climate Services (IWC):
(GFCS): 1. It adopted a global moratorium on
1. It is an initiative of the United Nations commercial whaling due to the
Framework Convention on Climate overexploitation of whale stocks in
Change. 1986.
2. It was established in the year 2004 after 2. Aboriginal subsistence whaling is not
the Indian Ocean Tsunami. regulated by the IWC.
Which of the statements given above is/are Which of the statements given above is/are
correct? correct?
(a) 1 only (a) 1 only
(b) 2 only (b) 2 only
(c) Both 1 and 2 (c) Both 1 and 2
(d) Neither 1 nor 2 (d) Neither 1 nor 2

40. Which of the following countries is the part


37. Consider the following statements regarding
of recently proposed IPMDA Initiative?
Gambusia Fish:
1. Japan
1. It migrates from the ocean to lay eggs in
2. Rusia
freshwater.
3. China
2. It is critically endangered.
4. U.S.A.
3. They are known for their symbiotic
5. India
relationship with the Olive Ridley
6. UK
Turtle. 7. South Korea
How many of the statements given above are Select the correct answer using the code
correct? given below.
(a) Only one (a) 2, 3 and 7 only
(b) Only two (b) 1, 2 and 3 only
(c) All three (c) 1, 4 and 5 only
(d) None (d) 1, 4, 5 and 6 only
8 www.visionias.in ©Vision IAS
41. Consider the following statements regarding 44. Recently, the Prime Minister launched the
Global Footprint Network: PM-PVTG scheme on the occasion of
1. It was established by the World Summit Janjatiya Gaurav Divas. In this context,
on Sustainable Development in 2002. consider the following statements:
2. It is responsible for hosting and 1. It aims to bring critical infrastructure to
calculating Earth Overshoot Day. the 75 PVTG communities.
Which of the statements given above is/are 2. To ensure coordination it aims to
correct? appoint one nodal officer for each state.
(a) 1 only 3. The Ministry of Rural Development is
(b) 2 only the nodal ministry for overall policy
(c) Both 1 and 2 planning and coordination of programs.
(d) Neither 1 nor 2 How many of the statements given above are
correct?
42. Which of the following statements is correct (a) Only one
regarding National River Conservation Plan (b) Only two
(NRCP) ? (c) All three
(a) It is under the Ministry of Water (d) None
Resources, River Development & Ganga
Rejuvenation. 45. National Air Quality Monitoring Programme
(b) National Lake Conservation Programme (NAMP) determines the status and trends of
is a sub-scheme under the National ambient air quality with respect to which of
River Conservation Programme. the following?
(c) All the states are covered under the 1. Carbon Dioxide
National River Conservation Plan. 2. Sulphur Dioxide
(d) It includes river front development 3. PM10
works and construction of low cost 4. Wind speed and direction
sanitation toilets. 5. Temperature
6. Oxides of Nitrogen
43. Consider the following statements regarding Select the correct answer using the code
the “Loss and Damage Fund”: given below.
1. It aims to provide financial assistance to (a) 2, 3, 4, 5 and 6 only
nations most vulnerable and impacted by (b) 1, 3 and 6 only
the effects of climate change. (c) 1, 2, 3, 4, 5 and 6
2. It was opertionalised at COP-21, UN (d) 1, 2 and 6 only
Climate Change Conference in Paris,
France. 46. The report ‘Global Landscape of Climate
Which of the statements given above is/are Finance 2023’ is released by:
correct? (a) World Bank
(a) 1 only (b) Intergovernmental Panel on Climate
(b) 2 only Change
(c) Both 1 and 2 (c) World Economic Forum
(d) Neither 1 nor 2 (d) Climate Policy Initiative
9 www.visionias.in ©Vision IAS
47. Consider the following statements regarding 50. Arrange the following in the descending
Project Kusha: order of renewable energy potential of India:
1. It is aimed at developing nuclear
1. Solar energy
submarines for India.
2. It is being jointly developed with the 2. Wind energy
help of Israel. 3. Bioenergy
Which of the statements given above is/are 4. Small Hydro
correct?
Select the correct answer using the code
(a) 1 only
given below.
(b) 2 only
(c) Both 1 and 2 (a) 1-2-4-3
(d) Neither 1 nor 2 (b) 1-2-3-4
(c) 4-1-3-2
48. With reference to the Indian coal, consider
(d) 2-4-1-3
the following statements:
1. India has the fifth largest coal reserves
and is second in coal production after 51. Consider the following statements regarding
China. solar energy sector in India:
2. Gondwana coal makes up more than 95
1. The current installed solar power
% of the total reserves and 99% of the
capacity in the country is more than
production of coal in India.
3. It has high ash and sulphur content but 100GW.
low ash fusion temperature thus making 2. Ground-mounted solar plants has the
India one of the largest importer. largest share followed by roof-top and
How many of the above statements are
hybrid projects respectively.
correct?
(a) Only one Which of the statements given above is/are
(b) Only two correct?
(c) All three (a) 1 only
(d) None
(b) 2 only

49. Recently, the World Health Organisation (c) Both 1 and 2


released the Global TB Report 2023. In this (d) Neither 1 nor 2
context, consider the following statements:
1. Staphylococcus aureus is the primary 52. World Heritage Outlook Report is released
causative agent of tuberculosis.
by:
2. Tuberculosis is an infectious disease that
affects only the lungs. (a) United Nations Educational, Scientific
3. The Mantoux test is used to monitor and Cultural Organization (UNESCO)
blood sugar in Tuberculosis patients. (b) Economic and Social Council
How many of the above statements are
(ECOSOC) of UN
correct?
(a) Only one (c) International Union for the Conservation
(b) Only two of Nature (IUCN)
(c) All three (d) United Nations Environment Programme
(d) None
(UNEP)
10 www.visionias.in ©Vision IAS
53. Consider the following statements regarding 55. Consider the following statements regarding

the Energy Saving Certificates (ESCerts): the International Tropical Timber

1. These are market-based regulatory Organization:


1. It is a multilateral non-governmental
instruments to reduce energy
organization for promoting the
consumption in energy-intensive
sustainable management and
industries.
conservation of tropical forests.
2. ESCerts trading is regulated by the
2. Countries from tropical areas with
Indian Renewable Energy Development timber trade are only eligible to become
Agency members of ITTO.

3. These certificates are issued under the 3. India is one of its producing members.

Perform, Trade and Achieve (PAT) How many of the above statements are
correct?
scheme.
(a) Only one
Which of the statements given above is/are
(b) Only two
correct?
(c) All three
(a) 1 and 2 only
(d) None
(b) 2 only

(c) 1 and 3 only 56. Consider the following statements, with


(d) 1, 2 and 3 respect to Graded Response Action Plan:
1. It is a set of measures implemented in
Delhi-NCR throughout the year for
54. On which of the following principles do air
controlling air pollution.
pollution control devices work?
2. The Commission for Air Quality
1. Gravity separation
Management (CAQM), is an
2. Cyclonic separation
autonomous body tasked with improving
3. Filtration the air quality in Delhi.
4. Wet Scrubbing 3. It calls for a complete ban on eateries,
5. Electrostatic Precipitation restaurants and hotels only when the Air

Select the correct answer using the code Quality Index reaches 450 level.
How many of the statements given above are
given below.
correct?
(a) 2, 3 and 4 only
(a) Only one
(b) 1 and 5 only
(b) Only two
(c) 1, 2, 3, 4 and 5
(c) All three
(d) 2, 3, 4 and 5 only (d) None
11 www.visionias.in ©Vision IAS
57. Consider the following statements regarding 60. Consider the following pairs:
the ‘Champions of the Earth Award’ : Generation of Source
1. It is the United Nations’ highest Bio-Fuels

environmental honor. 1. First : Food sources like sugar,


2. It recognizes the contributions of Generation starch or animal fats
individuals as well as organizations. 2. Second : Non-food crops like
3. Recently, this award was given to Generation stems, husks and wood
laureates who are implementing chips
3. Third : From micro-organisms
solutions and policies to eliminate
Generation like algae
plastic pollution.
4. Fourth : Genetically engineered
How many statements given above are
Generation crops
correct? How many pairs given above are correctly
(a) Only one matched?
(b) Only two (a) Only one
(c) All three (b) Only two
(d) None (c) Only three
(d) All four

58. Consider the following statements regarding


61. The biennial State of Worlds Forests Report
the Annapurna Certificate Programme: is published by:
1. It acknowledges restaurants abroad (a) World Environment Facility (WEF)
contributing to India’s cultural cause (b) United Nations Forum on Forests
through cuisines. (UNFF)
(c) World Forest Watch (WFW)
2. It is an initiative of the Indian Council
(d) The Food and Agriculture Organization
for Cultural Relations.
(FAO)
Which of the statements given above is/are
correct? 62. Consider the following statements with
(a) 1 only respect to Bharat Stage VI norms:
(b) 2 only 1. In April 2020 India transitioned from BS

(c) Both 1 and 2 V emission norms to BS-VI norms.


2. Emissions from petrol cars have to
(d) Neither 1 nor 2
decrease more compared to Diesel cars
to reach the norms prescribed in BS-VI.
59. BHARAT TAP initiative and Nirmal Jal 3. BS-VI-compliant fuel has zero levels of
Prayas initiative are under: sulphur content.
(a) Ministry of Jal Shakti. Which of the statements given above is/are
(b) Ministry of Housing and Urban Affairs. not correct?
(a) 3 only
(c) Ministry of Environment, Forest and
(b) 1 and 2 only
Climate Change.
(c) 2 and 3 only
(d) Ministry of Panchayati Raj (d) 1, 2 and 3
12 www.visionias.in ©Vision IAS
63. Consider the following statements with 66. The Fagradalsfjall system, recently seen in
respect to the short-lived climate pollutants: the news, is associated with?
1. They remain in the atmosphere for a
(a) Irrigation System
much shorter period of time than
methane but their potential to warm the (b) Volcanic system

atmosphere can be many times greater. (c) System to rehabilitate war victim
2. They are responsible for up to 45% of (d) System to attract honey bees
current global warming.
3. Climate & Clean Air Coalition is the
67. Consider the following statements regarding
only international body working to
address this issue. the National Clean Air Program (NCAP):
How many of the statements given above are 1. The NCAP aims to reduce particulate
correct?
matter (PM) concentrations by 20-30%
(a) Only one
in cities by 2030.
(b) Only two
(c) All three 2. It is being implemented by the Ministry
(d) None of Environment, Forest and Climate
Change (MoEFCC).
64. With reference to the disposal of waste, the
Which of the statements given above is/are
term 'Incineration' refers to:
(a) the process in which sedimented water is correct?
subjected to a chemical process to (a) 1 only
extract potable water.
(b) 2 only
(b) a physico-chemical process for
(c) Both 1 and 2
separating suspended and colloidal
impurities from liquids. (d) Neither 1 nor 2
(c) the process in which disinfectants like
chlorine or bleaching powder are added 68. In the context of the economy, consider the
to remove impurities in water.
following statements regarding Project
(d) the process of burning waste at very high
temperatures. Mariana:
1. It is an initiative of the World Bank.
65. Consider the following statements regarding 2. It tests cross-border trading and
White Phosphorous:
settlement of wholesale central bank
1. It ignites in contact with air.
2. It is banned by the Convention on digital currencies.

Certain Conventional Weapons (CCW). Which of the statements given above is/are
Which of the statements given above is/are correct?
correct?
(a) 1 only
(a) 1 only
(b) 2 only
(b) 2 only
(c) Both 1 and 2 (c) Both 1 and 2
(d) Neither 1 nor 2 (d) Neither 1 nor 2
13 www.visionias.in ©Vision IAS
69. Consider the following statements regarding 72. Consider the following statements regarding
'SATAT' scheme:
election funding in India:
1. The scheme envisages to target
production of 15 MMT (million tons) of 1. An individual can donate as much as he
compressed biogass by 2023. wants to a political party.
2. It aims to encourage entrepreneurs to set
2. A company can donate up to 10% of the
up compressed biogass plants, produce
and supply CBG to Oil Marketing average of the preceding three years'
Companies.
profits.
3. The scheme will help in better use of
municipal solid waste along with 3. There is no legal expenditure limit on
agricultural residue. expenditure by political parties.
How many of the statements given above are
How many of the statements given above are
correct?
(a) Only one correct?
(b) Only two (a) Only one
(c) All three
(d) None (b) Only two

(c) All three


70. Invasive species Specialist Group' (that
(d) None
develops Global Invasive Species Database)
belongs to which one of the following
organizations? 73. With reference to India’s updated
(a) The International Union for
Nationally Determined Contribution (NDC),
Conservation of Nature
(b) The United Nations Environment consider the following statements:
Programme
1. It aims to reduce emissions intensity of
(c) The United Nations World Commission
for Environment and Development. its GDP by 45 percent by 2030, from
(d) The World Wide Fund for Nature 2000 as the base year.

2. It aims to achieve 50 percent of its


71. Consider the following statements regarding
the National Organ and Tissue Transplant energy capacity from renewable based
Organisation (NOTTO):
energy resources by 2030.
1. It operates under the Ministry of Science
and Technology. 3. It targets net zero emissions by 2070.
2. It oversees organ procurement but How many of the above statements are
distribution is left to the states.
correct?
Which of the statements given above is/are
correct? (a) Only one
(a) 1 only
(b) Only· two
(b) 2 only
(c) Both 1 and 2 (c) All three
(d) Neither 1 nor 2 (d) None
14 www.visionias.in ©Vision IAS
74. Consider the following statements regarding 77. In the context of Indian culture, the term
Just Energy Transition Partnerships (JETPs): Vajra Mushti Kalaga was recently seen in
1. JETPs aim to bridge the gap between the news. It is a
developed and developing nations in (a) Buddhist sect
moving towards clean energy. (b) form of wrestling
2. The first JETP was announced at the (c) folk dance
Sharm el-Sheikh Climate Change
(d) form of puppetry
Conference (COP 27).
3. South Africa, Bangladesh, and
78. Which of the following statements correctly
Venezuela are the first three countries to
describes the "Non-Attainment Cities' under
receive funding under a JETP.
the National Clean Air Programme (NCAP)?
Which of the statements given above is/are
(a) Cities that do not participate in attaining
correct?
National Ambient Air Quality Standards
(a) 1 only
(b) 1 and 2 only (NAAQS) for a time bound period.

(c) 2 and 3 only (b) Cities identified by the Central Pollution


(d) 3 only Control Board (CPCB) who have not
attained 40% to 50% reduction in
75. Consider the following statements regarding particulate matter.
the Convention on Conservation of (c) Cities that do not meet the National
Migratory Species (CMS): Ambient Air Quality Standards
1. CMS is the only global (NAAQS) over a 5-year period for PM
intergovernmental organisation 10 or N02.
established exclusively for the (d) None of the above.
conservation and management of
migratory species.
79. Which of the following activities could best
2. It is an environmental treaty under the
prevent smog?
aegis of the United Nations Environment
1. Fuel up cars during the cooler hours of
Programme.
the day
Which of the statements given above is/are
2. Use of low-volatile organic compound
correct?
paints
(a) 1 only
3. Use of electric appliances instead of gas
(b) 2 only
powered lawnmowers
(c) Both 1 and 2
4. Use of public transportation
(d) Neither 1 nor 2
Select the correct answer using the code
given below.
76. In the context of security, 'Prachand' is a/an
(a) light combat helicopter. (a) 1 only

(b) inter-continental ballistic missile. (b) 1 and 3 only


(c) 2, 3 and 4 only
(c) nuclear submarine.
(d) 1, 2 , 3 and 4
(d) anti-tank guided missile.
15 www.visionias.in ©Vision IAS
80. Recently, millions of people have been 83. Consider the following statements with
internally displaced due to ongoing conflict respect to Corporate Average Fuel
with the rebel group Mouvement du 23 Mars Efficiency/Economy (CAFE) regulations:
(M23). To which country does this M23
1. They are primarily designed for oil
rebel group belong?
companies to increase fuel efficiency.
(a) Democratic Republic of Congo
(b) Palestine 2. CAFE is the weighted average of

(c) Sudan emissions and fuel economy for the


(d) Syria entire fleet sold in the country.
3. They were first notified in 2017 by the
81. Consider the following pairs: Union Ministry of Petroleum and
Bio- Used for
Natural Gas.
Medical
4. The existing limit for average CO2
Waste
emissions happens to be 100g/km.
Bag
1. Red : Discarded or Expired How many of the statements given above are
medicine correct?
2. Yellow : Soiled Waste (a) Only one
3. Blue : Glassware (b) Only two
4. White : Waste sharps including
(c) Only three
metal
(d) All four
How many pairs given above are correctly
matched?
(a) Only one 84. Consider the following statements with
(b) Only two respect to the Central Pollution Control
(c) Only three Board:
(d) All four 1. It can prohibit the use of any fuel or
appliance that may cause air pollution.
82. Consider the following statements regarding
2. It can inspect any control equipment,
the Atmospheric Waves Experiment :
industrial plant or manufacturing process
1. It is aimed at studying the interactions
between terrestrial and space weather. for the prevention of air pollution.

2. It is an initiative of the World 3. It may establish or recognize a


Meteorological Organization. laboratory for performing its functions
3. It will be launched and mounted on the under the Air (Prevention and Control of
exterior of the International Space Pollution) Act,1981.
Station (ISS).
How many of the statements given above are
How many of the statements given above are
correct?
correct?
(a) Only one
(a) Only one
(b) Only two (b) Only two
(c) All three (c) All three
(d) None (d) None
16 www.visionias.in ©Vision IAS
85. Consider the following statements regarding 88. Which of the following are the by-products
Global Methane Pledge (GMP): of coal based power plants in India?
1. Oxides of aluminium and calcium
1. Its aim is to collectively reduce methane
2. Arsenic
emissions by at least 30% below 2020 3. Boron
levels by 2030. 4. Chromium
5. Lead
2. It was launched at COP-26 of United
Select the correct answer using the code
Nations Climate Change Conference given below.
(UNFCCC) in Glasgow, Scotland. (a) 1, 3 and 5 only
3. India did not sign it because of its (b) 1, 4 and 5 only
(c) 2, 3, 4 and 5 only
concerns over the trade, farm and (d) 1, 2, 3, 4 and 5
livestock sector in the economy.
How many of the above statements are 89. Consider the following pairs:
Alternative fuel Produced from
correct?
1. Renewable : Cellulose material
(a) Only one Gasoline
(b) Only two 2. Renewable Diesel : Fats and oil
(c) All three 3. Sustainable : Woody biomass
(d) None Aviation
Fuel
How many pairs given above are correctly
86. Consider the following: matched?
1. Extension of existing Airports (a) Only one
(b) Only two
2. Ports and Harbour dealing in Fish
(c) All three
3. Road Highway Toll Plazas (d) None
How many of the above do not require
90. Consider the following statements with
public consultation under Environment
respect to the Rotterdam Convention:
Impact Assessment rules? 1. It is a multilateral treaty to promote
(a) Only one shared responsibilities in relation to the
(b) Only two importation of hazardous chemicals.
2. The movements of hazardous chemicals
(c) All three
listed in Annex III are subject to the
(d) None Prior Informed Consent procedure.
3. Exports are only allowed if the State of
import has consented to the future
87. Which of the following institutions publishes
import of the specific chemical.
the Global Energy Transition Index? 4. It also covers chemicals imported for the
(a) International Energy Agency purpose of research and analysis.
(b) World Economic Forum How many of the statements given above are
correct?
(c) Cornell University, USA
(a) Only one
(d) The Energy and Resources Institute, (b) Only two
India (c) Only three
(d) All four
17 www.visionias.in ©Vision IAS
91. 'Climate Neutral Now Initiative', an initiative 94. In the context of the electoral process in
of UNFCCC is: India, consider the following statements
regarding the term "ENCORE" :
(a) a certification scheme for non-parties to
1. It is software for complete candidate and
the convention. election management for returning
(b) an initiative to act now in order to officers.
2. It was developed by the National
achieve a climate-neutral world by 2030.
Technical Research Organisation.
(c) a certification scheme based on the
Which of the statements given above is/are
completion of nationally determined correct?
contributions. (a) 1 only
(b) 2 only
(d) an initiative to promote the voluntary
(c) Both 1 and 2
use of carbon market mechanisms.
(d) Neither 1 nor 2

92. Consider the following statements with 95. Consider the following statements regarding
ocean thermal energy in India:
respect to Chemical Oxygen Demand:
1. Ocean thermal energy is produced by
1. COD determines the amount of oxygen harnessing the temperature difference
required for the chemical oxidation of an between cold current and warm current.
organic matter. 2. India’s first ocean thermal energy plant
is being implemented in Kavaratti in
2. It helps differentiate between
Lakshadweep.
biologically oxidizable and biologically 3. The National Institute of Ocean
inert material. Technology (NIOT) is the implementing
agency for the plant.
3. It helps determine the efficiency of
How many statements given above are
treatment plants and pollution loads.
correct?
How many of the statements given above are (a) Only one
correct? (b) Only two
(c) All three
(a) Only one
(d) None
(b) Only two
(c) All three 96. Consider the following countries:
(d) None 1. Russia
2. Malaysia
3. Cambodia
93. The Palermo Convention recently seen in the 4. Indonesia
news deals with How many of the above countries are
(a) transnational organized crime. members of Global Tiger Forum?
(a) Only one
(b) sea level rise.
(b) Only two
(c) land degradation (c) All four
(d) global Financial Architecture. (d) None
18 www.visionias.in ©Vision IAS
97. Consider the following statements regarding 99. Consider the following pairs:
India's Carbon Credit Trading Scheme, Convention Related Subject
2023: 1. Basel : Transboundary
1. Under the scheme, MoEFCC will issue Convention Movements of
carbon credit certificates to the entities Hazardous Wastes and
that surpass the assigned targets. their Disposal.
2. The Central Electricity Regulatory 2. Hong Kong : Safe and
Commission (CERC) will regulate the Convention Environmentally Sound
trading of carbon credits under the Recycling of Ships.
scheme. 3. Stockhom : Persistent Organic
Which of the statements given above is/are Convention Pollutants
correct? How many pairs given above are correctly
(a) 1 only matched?
(b) 2 only (a) Only one
(c) Both 1 and 2 (b) Only two
(d) Neither 1 nor 2 (c) All three
(d) None
98. Cocoyoc declaration is associated with
(a) sustainable development 100. Climate Smart Agriculture aims to:
(b) ozone layer depletion 1. Adapting and building resilience of
(c) wildlife conservation crops and livestock to climate change
(d) nuclear non-proliferation 2. Sustainably increasing agricultural
productivity
3. Reducing greenhouse gas emissions
Which of the statements given above is/are
correct?
(a) 1 only
(b) 1 and 3 only
(c) 2 and 3 only
(d) 1, 2 and 3

Copyright © by Vision IAS


All rights are reserved. No part of this document may be reproduced, stored in a retrieval system or
transmitted in any form or by any means, electronic, mechanical, photocopying, recording or otherwise,
without prior permission of Vision IAS.
19 www.visionias.in ©Vision IAS
VISIONIAS
www.visionias.in

ANSWERS & EXPLANATIONS


GENERAL STUDIES (P) TEST – 4144 (2024)

Q 1.B
 Union Government of India approved the National Green Hydrogen Mission with an outlay of ₹ 19,744
crore from FY 2023-24 to FY 2029-30. The overarching objective of the Mission is to incentivize the
commercial production of green hydrogen and make India a net exporter of the fuel. The Mission
will facilitate demand creation, production, utilization and export of Green Hydrogen. Hence,
statement 1 is correct.
 The Mission will have wide-ranging benefits- the creation of export opportunities for Green Hydrogen
and its derivatives; Decarbonisation of industrial, mobility and energy sectors; reduction in dependence on
imported fossil fuels and feedstock; development of indigenous manufacturing capabilities; creation of
employment opportunities; and development of cutting-edge technologies. It aims to achieve the above
objectives, the Mission will build capabilities to produce at least 5 Million Metric tonnes (MMT) of
Green Hydrogen per annum by 2030, with the potential to reach 10 MMT per annum with the
growth of export markets. The Mission will support the replacement of fossil fuels and fossil fuel-based
feedstocks with renewable fuels and feedstocks based on Green Hydrogen. Hence, statement 2 is
correct.
o Achievement of Mission targets is expected to reduce a cumulative ₹ 1 lakh crore worth of fossil
fuel imports by 2030.
 The Ministry of New and Renewable Energy (MNRE) will be responsible for the overall
coordination and implementation of the Mission. The Mission Secretariat, headquartered in
MNRE, will formulate schemes and programs for financial incentives to support the production,
utilization and export of Green Hydrogen and its derivatives. Hence, statement 3 is not correct.

Q 2.C
 A carbon credit (also known as carbon offset) is a credit for greenhouse emissions reduced or removed
from the atmosphere by an emission reduction project, which can be used by governments, industry, or
private individuals to compensate for the emissions they generate elsewhere.
 Those that cannot easily reduce emissions can still operate, at a higher financial cost.
 Carbon credits are based on the "cap-and-trade" model that was used to reduce sulfur pollution in the
1990s. Hence, statement 2 is correct.
 One carbon credit is equal to one metric ton of carbon dioxide, or in some markets, carbon dioxide
equivalent gases (CO2-eq), and are bought and sold through international brokers, online retailers,
and trading platforms. Hence, statement 1 is correct.
 Offsetting one metric ton of carbon means that there will be one less Mt of carbon dioxide in the
atmosphere than there would otherwise have been. The Kyoto Protocol provides for three mechanisms
that enable countries, or operators in developed countries, to acquire greenhouse gas reduction
credits:
o Under Joint Implementation (JI) a developed country with relatively high costs of domestic
greenhouse reduction would set up a project in another developed country.
o Under the Clean Development Mechanism (CDM) a developed country can “sponsor” a
greenhouse gas reduction project in a developing country where the cost of greenhouse gas reduction
project activities is usually much lower, but the atmospheric effect is globally equivalent. The
developed country would be given credits for meeting its emission reduction targets, while the
developing country would receive the capital investment and clean technology or beneficial change in
land use.
o Under International Emissions Trading (IET) countries can trade in the international carbon credit
market to cover their shortfall in Assigned Amount Units (AAUs). Countries with surplus units can
sell them to countries that are exceeding their emission targets under Annex B of the Kyoto Protocol.
1 www.visionias.in ©Vision IAS
 Cap and trade is an approach that harnesses market forces to reduce emissions cost-effectively. Like
other market-based strategies, it differs from “command-and-control” approaches where the government
sets performance standards or dictates technology choices for individual facilities.
o Cap and trade allows the market to determine a price on carbon, and that price drives investment
decisions and spurs market innovation.
o Cap and trade differs from a tax in that it provides a high level of certainty about future emissions, but
not about the price of those emissions (carbon taxes do the inverse).

Q 3.D
 Indoor Air Pollution: The air within homes and other buildings can sometimes be more polluted than the
outdoor air even in the largest and most industrialised cities. Indoor air quality is an important concern for
the health and comfort of the occupants.
 Some of the sources of indoor air pollution are:
o Radon: Radon is an invisible, radioactive atomic gas that results from the radioactive decay of
radium, which may be found in rock formations beneath buildings or in certain building materials
themselves. Radon is the second most frequent cause of lung cancer, after cigarette smoking.
o Second-hand smoke: It is tobacco smoke which affects other people other than the 'active' smoker. It
includes both a gaseous and a particulate phase, with particular hazards arising from levels of carbon
monoxide and very small particulates.
o Biological chemicals: They can arise from a host of means, like moisture induced growth of
mould colonies and natural substances released into the air such as animal dander and plant
pollen. They are allergens and aggravate asthama.
o Ozone: Ozone is produced by ultraviolet light from the Sun hitting the Earth's atmosphere, lightning,
certain high-voltage electric devices and as a by-product of other types of pollution.
o Air Freshener: Many air fresheners employ carcinogens, volatile organic compounds and
known toxins such as phthalate esters in their formulas. Most of the products that have been
studied contain chemicals that can aggravate asthma and affect reproductive development.
 Hence, option (d) is the correct answer.

Q 4.D
 The Water (Prevention and Control of Pollution) Act was enacted in 1974 to provide for the
prevention and control of water pollution and for the maintaining or restoring of wholesomeness of
water in the country. The Act was amended in 1988.
 The Water (Prevention and Control of Pollution) Cess Act was enacted in 1977, to provide for the
levy and collection of a cess on water consumed by persons operating and carrying on certain types
of industrial activities. Hence, statement 1 is not correct.
 This cess is collected with a view to augment the resources of the Central Board and the State Boards for
the prevention and control of water pollution constituted under the Water (Prevention and Control of
Pollution) Act, 1974.
 The Act was last amended in 2003.
 On October 9, 2021, India’s Ministry of Environment, Forests and Climate Change published
“Environment (Protection) 115 Amendment Rules, 2021” to add regulation on the use of water
purification systems (WPS). Hence, statement 2 is not correct.
 Users of domestic water purification systems (DWPS) and other water purification systems (ODPWS)
have to comply with the guidelines issued by the Central Pollution Control Board (CPCB) within six
months of the promulgation of this regulation.

Q 5.C
 The Global Environment Facility:
The Global Environment Facility has a unique governing structure organized around an Assembly, the
Council, the Secretariat, 18 agencies, a Scientific and Technical Advisory Panel, and the Evaluation
Office.
 Financial mechanism provision by GEF
The GEF provides funding to assist developing countries in meeting the objectives of international
environmental conventions. The GEF serves as a "financial mechanism" to five conventions, which are
the Convention on Biological Diversity (CBD), the United Nations Framework Convention on
Climate Change (UNFCCC), the Stockholm Convention on Persistent Organic Pollutants (POPs),
the UN Convention to Combat Desertification (UNCCD), and Minamata Convention on Mercury.

2 www.visionias.in ©Vision IAS


 Five Focal Areas
It supports developing countries’ work to address the world’s most pressing environmental issues. GEF
organizes its work around five focal areas, They are as follows
o Biodiversity loss,
o Chemicals and waste
o Climate change
o International waters
o Land degradation
o and take an integrated approach to support more sustainable food systems, forest management, and
cities. Hence answer (c) is the correct answer.

Q 6.A
Status of India’s Nuclear Energy
 Nuclear energy is the fifth-largest source of electricity for India which contributes about 3% of the
total electricity generation in the country. Hence statement 1 is not correct.
 India has over 22 nuclear reactors in 7 power plants across the country which produces 6780 MW
of nuclear power. Hence statement 2 is not correct.
 In addition, one reactor,Kakrapar Atomic Power Project (KAPP-3) has also been connected to the grid in
January- 2021.
 18 reactors are Pressurised Heavy Water Reactors (PHWRs) and 4 are Light Water Reactors
(LWRs). Hence statement 3 correct.
 The existing nuclear power capacity of 6780 MW is going to be increased to 22480 MW by the year 2031
on progressive completion of projects under construction and accorded sanction. More nuclear power
plants are also planned in future.

Q 7.B
 The reduction of anthropogenic carbon dioxide (CO2) in the atmosphere is crucial for mitigating climate
change. CO2 capture and storage (CCS) is considered as one of the most promising options for carbon
reduction. The main means is the injection of CO2 into structural reservoirs in deep, permeable geologic
formations.
 Trapping mechanisms for geological sequestration or carbon sequestration include hydrodynamic
trapping, solubility trapping, and mineral trapping. Hence option (b) is the correct answer.
o Hydrodynamic trapping refers to that CO2 which is trapped as supercritical fluid or gas under a low-
permeability caprock. Carbon dioxide, being less dense than the formation fluid, will rise buoyantly
until it encounters a caprock.
o Solubility trapping: When CO2 is injected into a reservoir, a portion of the injected CO2 will dissolve
in the formation water in the aquifer and the dissolution of CO2 per unit volume of water is a function
of pressure, temperature, and salinity of the aqueous phase.
o Mineral carbonation refers to the incorporation of CO2 in a stable mineral phase via reactions with
mineral and organic matter in the formation. Over time the injected CO 2 will dissolve into the local
formation water and initiate a variety of geochemical reactions.
Q 8.A
 The International Solar Alliance (ISA) announced that the Global Solar Facility (GSF), formed by it
to stimulate investments into solar power projects, is set to receive a capital contribution of $35 million.
The Indian government is considering a $25 million investment as capital contribution in the GSF in
addition to $10 million coming from the ISA.
o It is a payment guarantee mechanism expected to stimulate investments into solar projects, with
two financial components:-
 Solar Payment Guarantee Fund to provide a partial guarantee and enable investments in
geographies that do not receive investments.
 Solar Insurance Fund to reduce the burden of insurance premium for solar developers in pre-
revenue phase of project.
 Initiatives taken by ISA but not under Solar Facility:-
o Green Grids Initiative - One Sun, One World, One Grid (OSOWOG).
o Global Energy Alliance for People and Planet (GEAPP) launched at COP26 with USD10
billions.
o ISA’s Programme on Scaling Solar Applications for Agriculture Use (SSAAU).
 Hence option (a) is the correct answer.

3 www.visionias.in ©Vision IAS


Q 9.B
 Resilient and Inclusive Supply-chain Enhancement—or RISE was launched by World Bank and
Japan—joined by Italy, the Republic of Korea, Canada, and the United Kingdom. Hence statement
1 is not correct.
 Resilient and Inclusive Supply-chain Enhancement—or RISE—initiative will help emerging
markets and developing countries (EMDCs) increase manufacturing of clean-energy products and
boost their participation in the minerals industry—leading to quality local jobs and economic
growth. Hence statement 2 is correct.
o Japan, Canada, Italy, the Republic of Korea, and the United Kingdom have pledged an initial total
contribution of more than $40 million to RISE, and more donors are expected to contribute.
 The energy transition will create a trillion-dollar market, with tens of millions of jobs along global mineral
value chains. It represents an opportunity for EMDCs to build manufacturing capacity, boost job growth,
and strengthen long-term economic development.

Q 10.C

 Greenhouse Gasses
o Greenhouse gases are a type of gas that traps heat in the atmosphere. This is what causes global
warming. Some of the most common greenhouse gasses include carbon dioxide (CO2), methane
(CH4), nitrous oxide (N2O), and fluorinated gases (SF6, HFCS, and PFCs).
 Global warming potential (GWP)
o Global warming potential (GWP) is a metric used to compare the relative impact of different
greenhouse gases on global warming. The GWP of a gas is defined as the amount of energy it absorbs
over a given period of time, relative to the same amount of CO2.

4 www.visionias.in ©Vision IAS


 Carbon dioxide (CO2):
o Carbon dioxide (CO2) is one of the primary greenhouse gases responsible for global warming. Global
warming potential (GWP) is a metric used to compare the relative impact of different greenhouse
gases on global warming. The GWP of a gas is defined as the amount of energy it absorbs over a
given period of time, relative to the same amount of CO2.
o The GWP of CO2 is defined as 1, meaning that it is the reference point for all other greenhouse
gases. The GWP of other gases is calculated by comparing their radiative forcing to that of CO2. For
example, methane (CH4) has a GWP of 21, which means that it traps 21 times more heat than the
same amount of CO2 over a 100-year period.
o GWPs are typically calculated over a 100-year time horizon, but they can also be calculated for
shorter or longer periods. This is because different greenhouse gases have different atmospheric
lifetimes. CO2, for example, has a very long atmospheric lifetime of about 100 years, meaning that it
can remain in the atmosphere for a long time and continue to trap heat. Methane, on the other hand,
has a shorter atmospheric lifetime of about 10 years, meaning that it is removed from the atmosphere
more quickly.
 Methane (CH4)
o Methane is a greenhouse gas that is about 56 times more potent than carbon dioxide. It is a major
contributor to climate change. Methane is released into the atmosphere from a variety of sources,
including:
 Natural sources: such as wetlands, wildfires, and termites.
 Human sources: such as agriculture, landfills, and oil and gas production.
 Nitrous Oxide (N2O)
o Nitrous oxide is a greenhouse gas that is about 280 times more potent than carbon dioxide. It is a
major contributor to climate change. Nitrous oxide is released into the atmosphere from a variety of
sources, including:
 Agricultural sources: such as fertilizers, manure, and irrigated agriculture.
 Industrial sources: such as nitric acid production and combustion processes.
 Hydrofluorocarbons (HFCs)
o Hydrofluorocarbons (HFCs) are a group of greenhouse gases that are used as refrigerants and in foam-
blowing applications. HFCs are very potent greenhouse gases. They are thousands of times more
potent than carbon dioxide. HFCs are not naturally occurring in the atmosphere. They are released
into the atmosphere from a variety of sources, including:
 Refrigeration and air conditioning
 Foam-blowing: HFCs are used in foam-blowing applications, such as in the production of
insulation and packaging materials.
 Sulphur Hexafluoride (SF6)
o Sulphur hexafluoride (SF6) is a greenhouse gas that is about 23,000 times more potent than carbon
dioxide. It is the most potent greenhouse gas known to man. SF6 is not naturally occurring in the
atmosphere. It is released into the atmosphere from a variety of sources, including:
o Electrical equipment: SF6 is used as an insulator in electrical equipment, such as power transformers
and switchgear.
o Semiconductor manufacturing: SF6 is used in the semiconductor manufacturing process.
 Hence option (c) is the correct answer.

Q 11.B
 Biochar is defined as a carbon-rich material produced during the pyrolysis process (not
incineration) that is a thermochemical decomposition of biomass with a temperature of about ≤700°C in
the absence or limited supply of oxygen. Hence statement 1 is not correct.
o Both incineration and pyrolysis are forms of combustion in which the thermal decomposition of
matter takes place. The key difference between incineration and pyrolysis is that incineration is the
combustion of organic matter in the presence of oxygen whereas pyrolysis is the combustion of
organic matter in the absence of oxygen.
 Biochar is primarily carbon (∼85%), but it can also contain oxygen, hydrogen, and inorganic ash if
present in the parent biomass. The heating value of biochar is in the range 25–32 MJ/kg dry basis, which
is substantially higher than that of the parent biomass or its liquid product. As biomass is carbon neutral,
the combustion of biochar is considered more environmentally friendly than coal.
 Biochar is characterized by a large pore surface area. Hence it has a large number of nonfuel uses such as
the adsorption of chemicals and carbon storage in the ground. When added back to the soil, biochar

5 www.visionias.in ©Vision IAS


absorbs and retains water and nutrients. Biochar enriches the soil and prevents it from leaching
pesticides and other nutrients into the runoff. Hence statement 2 is correct.
 Biochar is also an excellent carbon sink. When biomass is charred, it sequesters, or stores, its carbon
content. When biochar is added back to the soil, it can continue to absorb carbon and form large
underground stores of sequestered carbon that can lead to negative carbon emissions and healthier
soil. Hence statement 3 is correct.

Q 12.B
 Atmospheric lifetime of a greenhouse
o The atmospheric lifetime of a greenhouse gas is the average time it takes for half of the molecules of
that gas to be removed from the atmosphere. In other words, it is the length of time that a greenhouse
gas remains in the atmosphere before being removed through natural processes, such as chemical
reactions or absorption by the Earth's surface.
o The atmospheric lifetime of a greenhouse gas is important because it determines how long the gas will
continue to contribute to the greenhouse effect and climate change. Greenhouse gases with longer
atmospheric lifetimes, such as nitrous oxide (N2O) and hydrofluorocarbons (HFCs), will have a more
significant impact on climate change than gases with shorter atmospheric lifetimes, such as methane
(CH4).
Details of Green House gases:
 Methane (CH4)
o Atmospheric lifetime: 12 years
o Global warming potential (GWP): 25
o Sources: Natural sources such as wetlands, wildfires, and termites, as well as human sources such as
agriculture, landfills, and oil and gas production
o Properties: Methane is a colorless, odorless, and flammable gas. It is a potent greenhouse gas, with a
GWP of about 25 times that of carbon dioxide.
 Nitrous oxide (N2O)
o Atmospheric lifetime: 109 years.
o Hence option (b) is the correct answer.
 GWP: 280
o Sources: Natural sources such as soils and oceans, as well as human sources such as agriculture and
industrial processes
o Properties: Nitrous oxide is a colorless, odorless, and non-flammable gas. It is a potent greenhouse
gas, with a GWP of about 300 times that of carbon dioxide.
 Hydrofluorocarbons (HFCs)
o Atmospheric lifetime: Hundred years
o GWP: Thousands
o Sources: Man-made sources such as refrigeration and air conditioning, foam-blowing, and
semiconductor manufacturing
o Properties: HFCs are a group of colorless, odorless, and non-flammable gases. They are potent
greenhouse gases, with GWPs of thousands of times that of carbon dioxide.
 Chlorofluorocarbons (CFCs)
o Atmospheric lifetime: 50-100 years
o GWP: Thousands
o Sources: Man-made sources such as refrigerants, propellants, and foam-blowing agents
o Properties: CFCs are a group of colorless, odorless, and non-flammable gases. They are potent
greenhouse gases, with GWPs of thousands of times that of carbon dioxide.

Q 13.A
 Carbon trading began under the Kyoto Protocol of 1997 (which came into force in 2005). Under this,
‘certified emission reductions’ or CERs, were issued to entities that put up projects that reduced emissions
— such as wind, solar, or energy efficiency. Hence option (a) is the correct answer.
o It allows nations which are unable to meet their reduction targets to purchase carbon credits. Paris
Agreement also allows voluntary trading between countries to meet their NDC goals.
o If a country reduces more GHG emissions than its target, it can sell the emission reduction to another
country as an “internationally traded mitigation outcome.

6 www.visionias.in ©Vision IAS


Q 14.B
 Global Environment Facility (GEF), International Union for Conservation of Nature (IUCN) and
Conservation International announced the launch of a new global initiative to support the leadership
of Indigenous peoples and local communities in stewarding land, water and natural
resources. Hence, statement 1 is correct and 3 is not correct.
 The Inclusive Conservation Initiative (ICI) will support enhanced Indigenous and community stewardship
across 7.5 million hectares of landscapes, seascapes and territories with high biodiversity and
irreplaceable ecosystems. Recognizing the continuing historical role of Indigenous peoples and local
communities in safeguarding natural ecosystems, ICI will provide direct financial support to
Indigenous and locally-led initiatives in Africa, Central and South America, Asia and the Pacific.
Hence, statement 2 is correct.
 ICI provides site-based investments in nine subprojects to prioritize Indigenous and local community
organizations to take the lead in carrying out inclusive, culturally appropriate processes for decision-
making and strategy development that they have defined, implementing activities within their respective
territories, landscapes and/or sea scapes.
 The establishment of these nine subprojects in 12 countries demonstrates that there is high demand for
these inclusive finance models, as they were selected among over 400 Expressions of Interest (EOIs) that
were received from 80 countries.

Q 15.A
 Udangudi ‘Panangkarupatti’ (palm jaggery/ gur) from Tamil Nadu has been given a Geographical
Indication (GI) tag.
o The palm jaggery preparation procedure in this area is traditional to date without the inclusion of any
additional modern strategies.
o Triple superphosphate and phosphoric acid are used in many other areas, but no such chemical
additives are used in Udangudi Panangkarupatti. Hence pair 1 is not correctly matched.
o The karupatti prepared from the palm sap from the region around Udangudi in Tiruchendur taluk in
Thoothukudi district has some uniqueness. This is due to the presence of red sand dune soil found in
the region.
o This soil holds less groundwater. The moisture content in the atmosphere is less because of the dry
climatic conditions, which leads to high sucrose content, in turn adding taste.
 Khamti rice is a variety of sticky rice produced in the Namsai district of Arunachal Pradesh and is
known for its taste. It is cultivated by traditional Khampti tribal farmers. Hence pair 3 is correctly
matched.
 Tangsa textile is made by the Tangsa tribe of Changlang district in Arunachal Pradesh and is
famous for its exotic designs and colors. Hence pair 2 is not correctly matched.
 The first ever yak milk product, Arunachal Pradesh Yak Churpi also received a GI tag recently.

Q 16.B
 Carbon leakage refers to the situation that may occur if, for reasons of costs related to climate
policies, businesses were to transfer production to other countries with laxer emission constraints.
This could lead to an increase in their total emissions. Hence option (b) is the correct answer.
 How does Carbon Leakage work?
o We take an example of two countries A and B.
o Country A has a very strict emission policy and due to its stringent policy, the costs involved in the
production increases.
o Country B has a less strict and flexible emission policy and due to this flexible policy, the costs
involved in the production are less as compared to country A, keeping all other factors constant.
o So, a company located in country A faces increased costs due to emissions pricing as a result of the
strict climate policy. The company would take some action and as a result may decide to go for
reducing, closing or even relocating the production to Country B with less stringent climate
policies. This means that the Country A was though able to cut emissions, but now Country B
will increase the emissions due to transfer of greenhouse gas intensive industries from Country
A to B. The result is more Green House Gases emission and more industrial jobs.
 Other associated concepts –
o Carbon Pricing is an instrument that captures the external costs of greenhouse gas (GHG)
emissions - the costs of emissions that the public pays for, such as damage to crops, health care costs
from heat waves and droughts, and loss of property from flooding and sea level rise - and ties them to
their sources through a price, usually in the form of a price on the carbon dioxide (CO2) emitted.
7 www.visionias.in ©Vision IAS
o Carbon Tax - A carbon tax is a fee imposed on businesses and individuals that works as a sort
of "pollution tax." The tax is a fee imposed on companies that burn carbon-based fuels, including
coal, oil, gasoline, and natural gas.
o Carbon Trading is the buying and selling of credits that permit a company or other entity to emit a
certain amount of carbon dioxide. Carbon trade agreements allow for the sale of credits to emit
carbon dioxide between nations as part of an international agreement aimed at gradually
reducing total emissions.
o Zero Carbon Law by New Zealand - The Zero Carbon Act puts in place targets to reduce all
greenhouse gases: Carbon dioxide and nitrous oxide have to reduced to net zero by 2050.

Q 17.C
 As India shifts gears to meet its net-zero target by 2070, the transition towards clean energy and multiple
decarbonization initiatives will optimize the use of the country's energy mix and reduce the proportion of
fossil fuels over the long run.
 Since the Indian automotive industry has always been a key indicator of the nation’s economic growth,
contributing to approximately 6.5 percent of India’s overall gross domestic product; 40 percent of its
manufacturing GDP and supporting about 30 million direct and indirect jobs, the growth of this industry
should be in sync with the government’s vision of ‘green growth’. Going forward, the industry would
need to continue to build on a strong foundation of sustainability, environmental consciousness and safety.
Hence, a range of initiatives have been introduced by the Indian automobile industry under the
aegis of the Society of Indian Automobile Manufacturers (SIAM).
 One of the initiatives is for biofuels. Through a campaign called ‘Javik Pahal’, the industry
promotes biofuels produced from renewable biological materials. Alternative fuels can play a pivotal
role in powering vehicles, reducing the reliance on petroleum imports, lowering greenhouse gas emissions
and also improving farmer incomes. Hence statements 1 and 2 are correct.

Q 18.B
 Recent context: China to build the world’s largest ‘ghost particle’ or Neutrino detector.
 Neutrinos are a type of electron but, like neutrons, they do not have any charge. They are among the
most abundant particles in our universe — with trillions of neutrinos passing through you at any given
second — and also among the tiniest. Hence statement 3 is correct.
 Neutrinos were long believed to be massless until scientists found evidence that they do have a very
small mass. Hence statement 2 is correct.
 Neutrinos’ weak charge and almost nonexistent mass have made them notoriously difficult for scientists
to observe. They can only be “seen” when they interact with other particles.
o The rarity of interactions with other particles makes them almost impossible to track. That’s why
they’re called ghost particles — the vast majority skirt around undetected.
 They rarely interact with other particles. But rarely doesn’t mean “never”. Sometimes they interact with
water molecules, which is why China is building its ghost molecule telescope underwater. Hence
statement 1 is not correct.
 Scientists have observed ghost particles in fleeting instances when the particles create byproducts after
traveling through water or ice. These “muons” create flashes of light that can be detected by sophisticated
underwater telescopes and offer one of the fews ways to study the energy and source of neutrinos.
 Right now, the largest neutrino-detecting telescope is the University of Madison-Wisconson’s “IceCube”
telescope. Situated deep in the Antarctic, the telescope’s sensors span around 1 cubic kilometer.

Q 19.B
 The Convention on Wetlands of International Importance (Ramsar Convention):
o The Convention on Wetlands of International Importance (Ramsar Convention) was adopted in 1971
and entered into force in 1975.
o It is a legally binding intergovernmental framework instrument embodying the commitments of its
member countries to maintain the ecological character of their Wetlands of International Importance
and to plan for the "wise use", or sustainable use, of all wetlands in their territories. Hence statement
1 is correct
o The Convention’s mission is “the conservation and wise use of all wetlands through local and national
actions and international cooperation, as a contribution towards achieving sustainable development
throughout the world”.

8 www.visionias.in ©Vision IAS


 What are wetlands under the Ramsar Convention:
o Wetlands are areas where water is the primary factor controlling the environment and the associated
plant and animal life. They occur where the water table is at or near the surface of the land, or where
the land is covered by water. The Ramsar Convention takes a broad approach in determining the
wetlands which come under its aegis.
o Under the text of the Convention (Article 1.1), wetlands are defined as: “areas of marsh, fen,
peatland or water, whether natural or artificial, permanent or temporary, with water that
is static or flowing, fresh, brackish or salt, including areas of marine water the depth of which at
low tide does not exceed six meters”.
o This definition includes lotic water bodies, such as rivers and streams, if they meet specific
criteria. For example, they must be important waterbird habitats, representative examples of particular
types of wetland systems, or ecologically important for other reasons, such as providing flood control
or sediment retention. Hence statement 2 is not correct.
 Lotic water bodies:
o Lotic water bodies are water bodies that have a current or flow of water, such as rivers, streams,
and creeks.
o They are characterized by their high dissolved oxygen content and their ability to transport sediment
and nutrients.
o Lotic water bodies support a variety of aquatic and riparian (streamside) organisms, including fish,
insects, and plants.
 Lentic water bodies:
o Lentic water bodies are water bodies that do not have a current or flow of water, such as ponds,
lakes, and marshes.
o They are characterized by their low dissolved oxygen content and their tendency to stratify (form
layers) due to differences in temperature and density.
o Lentic water bodies support a variety of aquatic and emergent (water-loving) plants, as well as
amphibians, reptiles, and waterfowl.
 Five major wetland types are generally recognized under the Ramsar Convention:
o marine (coastal wetlands including coastal lagoons, rocky shores, and coral reefs);
o estuarine (including deltas, tidal marshes, and mangrove swamps);
o lacustrine (wetlands associated with lakes);
o riverine (wetlands along rivers and streams);
o palustrine (meaning “marshy” - marshes, swamps and bogs).
 Ramsar Wetlands in India:
o In the 75th year of Independence, the number of Ramsar sites in India stands at 75, covering an area
of 13,26,678 ha, thanks to the recent addition of 11 more wetlands to the list in 2022.
o Among the eleven new sites,
 four are in Tamil Nadu (Chitrangudi Bird Sanctuary, SuchindramTheroor Wetland Complex,
Vaduvur Bird Sanctuary, Kanjirankulam Bird Sanctuary)
 three in Odisha (Tampara Lake, Hirakud Reservoir, Ansupa Lake)
 two in Jammu & Kashmir (Hygam Wetland Conservation Reserve, Shallbugh Wetland
Conservation Reserve)
 and one each in Madhya Pradesh (Yashwant Sagar) and Maharashtra (Thane Creek).
 Currently India, with 75, has the largest network of Ramsar Wetlands in Asia surpassing China.
Hence statement 3 is correct.

Q 20.A
 E-waste, electronic waste, e-scrap and end-of-life electronics are terms often used to describe used
electronics that are nearing the end of their useful life, and are discarded, donated or given to a recycler.
 The UN defines e-waste as any discarded products with a battery or plug, and features toxic and
hazardous substances such as mercury, that can pose severe risk to human and environmental health.
 The overarching objective of the Basel Convention on the Control of Transboundary Movements of
Hazardous Wastes and their Disposal is to protect human health and the environment against the
adverse effects of hazardous wastes. E-waste is categorized as hazardous waste due to the presence
of toxic materials such as mercury, lead and brominated flame retardants are considered as
hazardous waste according to the Basel Convention. In addition, transboundary movements of hazardous
and other wastes, including e-waste ending up in dumps, are deemed to be illegal traffic under the Basel
Convention, Article 9. Hence, statement 1 is correct.

9 www.visionias.in ©Vision IAS


 India is the third largest electronic waste generator in the world after China and the USA and these
three countries together contributed 38% of total 53.6 million tonnes (Mt) of e-waste generated world
wide in 2019. Hence, statement 2 is not correct.

Q 21.A
 Recently, in 2023 the Dhordo village in the Kutch district of Gujarat was awarded as the Best
Tourism Village by the United Nations World Tourism Organization (UNWTO).
o From Salty marshland to an Iconic destination of the Western Indian Subcontinent – Dhordo, in the
Rann of Kutch, has become the face of Gujarat’s development.
o Tourism was launched through Rann Utsav, a four-month festival whereby a special tent city
was created in the desertic land with all the necessary amenities made available to tourists.
o It is during these four months that the village comes to life displaying all the natural and cultural
resources/products to its consumers.
 Mawlynnong is a village in the East Khasi Hills district of the Meghalaya state in North East India. It is
notable for its cleanliness and also was chosen by Discover India magazine as Asia's cleanest village.
 The remote village of Malana is nestled in the pristine Parvati Valley of Himachal Pradesh, and stands
as a mysterious and enchanting destination. Known for its unique customs, centuries-old traditions, and
breathtaking natural beauty, Malana is a place that has captured the imaginations of travelers for years.
 Odanthurai of Tamil Nadu has taken energy generation to another level. The village not only produces
its own electricity but also sells it to the state’s government. The village has come up with self-help
ventures Wind-power generation.
o They have replaced the grid electricity with a 9kW biomass gasifier power generation system for the
means of pumping water to the houses.
 Hence option (a) is the correct answer.

Q 22.B
 The Least Developed Countries Fund (LDCF) was established at the 7th Conference of the Parties
in 2001 (COP7) to meet the adaptation needs of least developed countries (LDCs). Hence, statement
1 is not correct.
 LDCF is active in sectors including water, agriculture and food security, health, disaster risk management
and prevention, infrastructure and fragile ecosystems and has the largest portfolio of adaptation projects of
its kind. The Global Environmental Facility (GEF) administers the LDCF as a specialized trust fund and
serves as a basis for programming resources.
 The LDCF aims to address the needs of the 51 LDCs which are particularly vulnerable to the
adverse impacts of climate change. As a priority, the LDCF supports the preparation and
implementation of the National Adaptation Programmes of Action (NAPAs), which are country-
driven strategies that identify the immediate needs of LDCs in order to adapt to climate change.
Hence, statements 2 and 3 are correct.

Q 23.C
 The India GHG Program led by WRI India, Confederation of India Industry (CII) and The Energy and
Resources Institute (TERI) is an industry-led voluntary framework to measure and manage
greenhouse gas emissions. Hence option (c) is the correct answer.
 The programme builds comprehensive measurement and management strategies to reduce emissions and
drive more profitable, competitive and sustainable businesses and organisations in India.
 The programme is supported by the Shakti Sustainable Energy Foundation, the German Federal Ministry
for the Environment, Nature Conservation and Nuclear Safety (BMU) and Pirojsha Godrej Foundation.

Q 24.C
 Recently, Scientists have made history by successfully turning on the world's smallest particle
accelerator for the first time. With a size similar to a small coin, this astonishing discovery
holds enormous potential for a variety of applications, including the use of miniature particle
accelerators for medicinal purposes. Hence statements 1 and 2 are correct.
 The NEA is a cutting-edge technology made up of a microscopic microprocessor and an even
smaller vacuum tube made up of thousands of individual pillars.
 By directing microscopic laser beams onto these tiny pillars, researchers have developed a way for
accelerating electrons. Surprisingly, the main acceleration tube within the NEA is only 0.02 inches (0.5
millimeters) long, which is 54 million times shorter than the massive 16.8-mile-long (27-kilometer) ring
10 www.visionias.in ©Vision IAS
housing CERN's Large Hadron Collider (LHC) in Switzerland, the world's largest and most powerful
particle accelerator.
 Nano-Scale Precision: The interior of the NEA's tiny tunnel is around 225 nanometers broad. According
to the National Nanotechnology Institute, the thickness of a human hair ranges between 80,000 and
100,000 nanometers.
 Acceleration of Electrons: Researchers from the Friedrich-Alexander University of Erlangen-Nuremberg
(FAU) in Germany used the NEA to accelerate electrons in a recent study published in the journal Nature.
They effectively increased electron energy from 28.4 kiloelectron volts (keV) to 40.7 keV, a 43%
increase. This is the first successful activation of a nanophotonic electron accelerator, which was first
proposed in 2015.
 It is a Particle Acceleration on a Microchip.
 In contrast to Large Particle Accelerators: The LHC uses almost 9,000 magnets to create a magnetic
field for particle acceleration, whereas the NEA uses light beams focused at the vacuum tube's
pillars to magnify energy, resulting in a far lower energy field. Electrons accelerated by the NEA have
substantially less energy than electrons accelerated by major colliders such as the LHC. However, experts
believe that by experimenting with different materials or stacking many tubes together, they can improve
particle acceleration even further. Despite this, the NEA will not achieve energy levels equivalent to those
of massive particle colliders. Hence statement 3 is correct.
 Possibilities for Medical Applications: One of the key goals of developing these small accelerators is
to exploit the energy released by accelerated electrons to deliver tailored medicinal treatments,
perhaps replacing more invasive forms of radiotherapy used to battle cancer cells. The ultimate goal
is to mount a particle accelerator on an endoscope, allowing radiotherapy to be delivered directly to
afflicted parts of the body. This transformational application, however, remains a long-term goal.
Q 25.A
 E-Waste Rules (Management), 2022
 These rules shall apply to every manufacturer, producer refurbisher, dismantler and recycler involved in
the manufacture, sale, transfer, purchase, refurbishing, dismantling, recycling and processing of e-waste or
electrical and electronic equipment listed in Schedule I, including their components, consumables, parts
and spares which make the product operational.
 But they shall not apply to - (a) waste batteries as covered under the Battery Waste Management
Rules, 2022; (b) packaging plastics as covered under the Plastic Waste Management Rules, 2016; (c)
micro-enterprise as defined in the Micro, Small and Medium Enterprises Development Act, 2006
(27 of 2006); and (d) radio-active wastes as covered under the provisions of the Atomic Energy Act,
1962 (33 of 1962) and rules made there under. Hence, statement 1 is correct.
 ‘e-retailer’ means an individual company or business entity that uses an electronic network such as
the internet, social media, telephone or any other media, to sell its goods. Hence, statement 2 is not
correct.
 ‘historical e-waste’ means e-waste generated from electrical and electronic equipment as specified in
Schedule I which was available on the date from which these rules came into force;
 'orphaned products’ means non-branded or assembled electrical and electronic equipment as
specified in Schedule I or those produced by a company which has closed its operations; Hence,
statement 3 is not correct.
 ‘extended producer responsibility’ means the responsibility of any producer of electrical or electronic
equipment as given in Schedule-I for meeting recycling targets as per Schedule-III and Schedule-IV, only
through registered recyclers of e-waste to ensure environmentally sound management of such waste;

Q 26.A
 Statement 1 is correct. ‘Zero budget’ farming promises to end a reliance on loans and drastically cut
production costs, ending the debt cycle for desperate farmers. The word ‘budget’ refers to credit and
expenses, thus the phrase 'Zero Budget' means without using any credit, and without spending any money
on purchased inputs.
 Statement 2 is correct. The main aim of ZBNF is eliminate use of chemical pesticides and uses
biological pesticides and promote of good agronomic practices. Farmers use earthworms, cow dung,
urine, plants, human excreta and such biological fertilizers for crop protection.
 Statement 3 is not correct. It has attained wide success in southern India, especially the southern Indian
state of Karnataka where it first evolved. The movement in Karnataka state was born out of collaboration
between Mr Subhash Palekar, who put together the ZBNF practices, and the state farmers association
Karnataka Rajya Raitha Sangha (KRRS).
11 www.visionias.in ©Vision IAS
Q 27.C
 Recent context : Raising concerns over the use of World Bank’s Worldwide Governance Indicators
in ratings assessment by credit ratings agencies, especially for emerging economies, Chief Economic
Adviser V Anantha Nageswaran said there is a need for the World Governance Index to be more
transparent and less subjective.
 The World Bank’s Worldwide Governance Indicators provide a ranking of 215 countries territories
based on six dimensions of governance: ‘Voice and Accountability’; ‘Political Stability and Absence of
Violence’; ‘Government Effectiveness’; ‘Regulatory Quality’; ‘Rule of Law’ and ‘Control of Corruption.
 The WGI were developed in 1999 by two World Bank researchers, Daniel Kaufmann and Aart Kraay. The
data are updated annually each September.
 The WGI aggregate data from more than 30 think tanks, international organizations,
nongovernmental organizations, and private firms across the world selected on the basis of three key
criteria: 1) they are produced by credible organizations; 2) they provide comparable cross-country data;
and 3) they are regularly updated.
 Hence option (c) is the correct answer.

Q 28.A
 The United Nations Environment Assembly (UNEA):
o The United Nations Environment Assembly (UNEA) is the world's highest-level decision-
making body on the environment. It is a universal body with the membership of all 193 UN
Member States. Hence statement 2 is correct.
o UNEA meets every two years to set the global environmental agenda, provide overarching policy
guidance, and define policy responses to address emerging environmental challenges. Hence
statement 3 is not correct.
 Establishment of UNEA
o UNEA was established in 2012 as a result of the United Nations Conference on Sustainable
Development (UNCSD), also known as Rio+20. Hence statement 1 is not correct.
o The UN General Assembly then adopted resolution on strengthening and upgrading UNEP and
establishing universal membership of its Governing Council (GC).
o In March 2013, the UN General Assembly further adopted resolution, which changed the designation
of the UNEP Governing Council (GC) to the UNEA of the UNEP.
 Key Functions of UNEA
UNEA plays a pivotal role in shaping international environmental policy and driving action towards a
sustainable future. Its key functions include:
o Setting environmental priorities: UNEA identifies and prioritizes key environmental issues that
require international attention and action.
o Developing policy frameworks: UNEA develops and adopts policy frameworks to address global
environmental challenges, providing a roadmap for action.
o Monitoring and review: UNEA monitors the implementation of environmental policies and
agreements, conducting regular reviews to assess progress and identify areas for improvement.
o Strengthening environmental governance: UNEA strengthens environmental governance by
providing a platform for international cooperation and collaboration on environmental matters.
o Promoting sustainable development: UNEA promotes sustainable development by integrating
environmental considerations into social and economic policies.
 Structure of UNEA
UNEA is governed by the Conference of the Parties (COP), which is composed of representatives from all
193 UN Member States. The COP meets every two years to deliberate and adopt decisions on
environmental matters.

Q 29.D
 The Ministry of New and Renewable Energy (MNRE) is the nodal Ministry of the Government of India
for all matters relating to new and renewable energy. The broad aim of the Ministry is to develop and
deploy new and renewable energy to supplement the energy requirements of the country.
 Renewable Energy in India includes:
o Wind energy
o Nuclear energy
o Solar energy
o Biomass energy

12 www.visionias.in ©Vision IAS


o Geothermal energy
o Hydropower
 Hence option (d) is the correct answer.

Q 30.A
 As a specialized UN agency to address all matters related to international civil aviation, including
environmental protection, ICAO has been diligently addressing GHG emissions from international
aviation. The ICAO agreement on carbon-neutral growth and CORSIA complements the ambition of the
Paris Agreement and constitutes the most significant international climate-related agreement since its
adoption.
 CORSIA is the first global market-based measure for any sector and represents a cooperative
approach that moves away from a “patchwork” of national or regional regulatory initiatives. It
offers a harmonized way to reduce emissions from international aviation, minimizing market
distortion, while respecting the special circumstances and respective capabilities of ICAO Member
States. Hence, statement 1 is correct.
 CORSIA complements the other elements of the basket of measures by offsetting the amount of CO2
emissions that cannot be reduced through the use of technological improvements, operational
improvements, and sustainable aviation fuels with emissions units from the carbon market.
 Why does the Paris Agreement not include international aviation emissions?
o Specifically, governments working under the auspices of the UNFCCC have agreed that while all
domestic GHG emissions are dealt with under the UNFCCC, GHG emissions associated with
international aviation and maritime transport are to be dealt with under ICAO and
International Maritime Organization (IMO), respectively. This approach is consistent with similar
UNFCCC decisions that also apply to the Kyoto Protocol.
o In this regard, GHG emissions from domestic aviation, as per other domestic sources, are calculated
as part of the UNFCCC national GHG inventories and are included in national totals (part of the
Nationally Determined Contributions (NDCs) of the Paris Agreement), while GHG emissions from
international aviation are reported separately and are not included in NDCs.
 CORSIA will be implemented in three phases, as follows:
o Pilot phase: from 2021 to 2023;
o First phase: from 2024 to 2026; and
o Second phase: from 2027 to 2035.
 For the pilot and first phase, participation is voluntary, while for the second phase, participation is
based on the States' RTK level in 2018 and voluntary participation. Hence, statement 2 is not
correct.

Q 31.B
 The Montreal Protocol on Substances that Deplete the Ozone Layer is a global agreement to protect
the Earth’s ozone layer by phasing out the chemicals that deplete it. This phase-out plan includes both
the production and consumption of ozone-depleting substances.
 The landmark agreement was signed in 1987 and entered into force in 1989.
 The most recent amendment, the Kigali Amendment, called for the phase-down of hydrofluorocarbons
(HFCs) in 2016. These HFCs were used as replacements for a batch of ozone-depleting substances
eliminated by the original Montreal Protocol.
 Although they do not deplete the ozone layer, they are known to be powerful greenhouse gases and, thus,
contributors to climate change.The Montreal Protocol provided a set of practical, actionable tasks that
were universally agreed on.
 Under the Kigali Amendment; Parties to the Montreal Protocol will phase down production and
consumption of Hydrofluorocarbons, commonly known as HFCs.
o Hydrofluorocarbons were introduced as a non-ozone-depleting alternative to
Hydrofluorocarbons (HFCs). Hence, statement 1 is not correct.
13 www.visionias.in ©Vision IAS
o While HFCs do not deplete the stratospheric ozone layer, they have a high global warming potential
ranging from 12 to 14,000, which has an adverse impact on climate.
o Recognizing the growth in the use of HFCs, especially in the Refrigeration and Air-conditioning
sector the Parties to the Montreal Protocol, reached an agreement at their 28th Meeting of the Parties
(MOP) held in October 2016 in Kigali, Rwanda to add HFCs to the list of controlled substances and
approved a timeline for their gradual reduction by 80-85 per cent by the late 2040s. Hence,
statement 2 is correct.
o India will complete its phase-down of HFCs in 4 steps from 2032 onwards with a cumulative
reduction of 10% in 2032, 20% in 2037, 30% in 2042 and 85% in 2047.
o All amendments and adjustments of the Montreal Protocol, prior to the Kigali Amendment have
Universal support.
o India has successfully met the phase-out targets of all the Ozone Depleting Substances as per the
Montreal Protocol Schedule. Hence, statement 3 is correct.
Q 32.C
 Ethanol, anhydrous ethyl alcohol is produced from sugarcane, maize, wheat, etc which are having high
starch content. In India, ethanol is mainly produced from sugarcane molasses by the fermentation process.
Ethanol can be mixed with the gasoline to form different blends.
 The Ethanol Blending Programme (EBP) seeks to achieve the blending of Ethanol with motor spirit with a
view to reducing pollution, conserving foreign exchange and increasing value addition in the sugar
industry enabling them to clear cane price arrears of farmers.The level of ethanol blending in petrol in
India has reached 9.99 per cent.
 India had targeted 10 per cent ethanol blending in petrol by the end of 2022 and 20 per cent blending by
2030 as per National Biofuel Policy-2018. Later the Central Government advanced it by five years for
achieving 20% ethanol blending in petrol.
 The amended National Biofuel Policy-2018 has now set the new target for 2025-26 instead of
2030. To achieve this target within the timeframe the central government announced premium rates for
ethanol produced from sugar syrup, cane juice as well as B-heavy molasses. Hence statement 1 is
correct.
 The Centre has also targeted a 5 per cent blending of biodiesel with diesel by 2030. Mixing 20 per cent
ethanol in a petrol can potentially reduce the auto fuel import bill by a yearly Rs 30,000 crore.
 In India, the Department of Food and Public Distribution (DFPD) is the nodal department for the
promotion of fuel-grade ethanol-producing distilleries in the country. It launched the Interest
Subvention Scheme for enhancement and augmentation of the ethanol production capacity. Hence
statement 2 is correct.

Q 33.B
 Ethanol is one of the principal biofuels, which is naturally produced by the fermentation of sugars by
yeasts or via petrochemical processes such as ethylene hydration. It has medical applications as an
antiseptic and disinfectant. It is used as a chemical solvent and in the synthesis of organic compounds,
apart from being an alternative fuel source.
 Ethanol blending offers significant advantages such as an increase in the Research Octane Number
(RON) of the blend, fuel-embedded oxygen and higher flame speed. These properties of ethanol
help in complete combustion and reduce vehicular emissions such as hydrocarbon, carbon
monoxide and particulate matter.
 The calorific value of ethanol is around 2/3rd of gasoline. This indicates that the increase in ethanol
content will decrease the heating value of the ethanol-gasoline blend. Hence, more fuel is required to
achieve the same engine power output.
 However, ethanol has a higher octane number and thus the engine can be operated with a high
compression ratio without knocking. This increases the efficiency of the engine considerably.
 This combined with optimal spark timing negates the fuel economy debit due to the low calorific
value of ethanol. Hence, ethanol is considered as an efficient fuel provided suitable modifications
are made in the vehicle. Hence, option (b) is the correct answer.

Q 34.C
 The wastewater containing hazardous waste chemicals should be detoxified and neutralised through
treatment.
 There are many technologies available for treating hazardous wastes before they are ultimately disposed
of.
14 www.visionias.in ©Vision IAS
 Their aim is to modify the physical and/or chemical properties of the wastes so that they are rendered
harmless.
 Selection of a treatment process depends on many factors such as the nature of the waste, the desired
characteristics of the output stream, and economic and energy considerations.
 The treatment technologies can be divided into the following groups, namely:
o Physical treatment
 is conducted using various methods such as phase separation. Phase separation includes three
steps, namely: lagooning, prolonged storage in tanks and sludge drying in beds. Lagooning and
tank storage are collectively used to separate particulate impurities.
o Chemical treatment
 is used to facilitate the complete breakdown of hazardous wastes and more usually to modify the
chemical properties of the wastes, e.g., to reduce water solubility or to neutralise acidity or
alkalinity. The techniques involve oxidation, chemical reduction, neutralization, heavy metal
precipitation, oil-water separation and solvents/ fuels recovery.
o Biological treatment
 The gross impurities obtained from the treatment of sewage are collectively known as sludge,
which is given biological treatment, before disposal.
 This is known as sludge processing which has become important since improvements in industrial
waste water treatment.
 The typical technologies for sludge processing include conditioning, digestion, composting,
thickening or dewatering and solidification.
 Conditioning: In this step, the sludge is exposed to the atmosphere for a stipulated period
until a desired consistency is reached.
 Digestion: In this process, the sludge is treated with bacteria which break down the long-
chain compounds into simpler ones.
 Composting: In this step, the organic matter in the waste sludge is converted into a usable
stable material.
o Solidification
 processes convert liquid waste into insoluble, rock-hard material and are used as pre-treatment
prior to landfill disposal. This is usually done by mixing the waste with various reactants to
produce a solid mass. The basic aim of the solidification process is to immobilize the hazardous
constituents of the waste so that these do not leach out at the landfill disposal site.
o Incineration
 Apart from the above-mentioned methods, incineration is also a method of detoxification, in
which oxidation of waste detoxifies the waste from its toxic proportion.
o Hence option (c) is the correct answer.

Q 35.A
 National Action Plan on Climate Change (NAPCC)
The National Action Plan on Climate Change (NAPCC) is a comprehensive framework for India's climate
change strategy. It was launched in 2008 and outlines eight key missions to address climate change in
various sectors of the economy.
o The Eight Missions and Objectives of NAPCC
 National Mission on Sustainable Habitat: Promote sustainable urban planning and
transportation to reduce the impact of cities on the environment. It is executed under the Ministry
of Housing and Urban Affairs (MoHUA)
 National Mission for Enhanced Energy Efficiency: Improve energy efficiency across all
sectors of the economy to reduce greenhouse gas emissions. It is executed under the Ministry of
Power (MoP)
 National Mission For Sustainable Agriculture (Nmsa): Promote sustainable agricultural
practices to reduce greenhouse gas emissions and enhance food security. It is executed under the
Ministry of Agriculture and Farmers Welfare (MoAFW)
 National Water Mission: Conserve and efficiently use water resources to adapt to the impacts of
climate change. It is executed under the Ministry of Jal Shakti.
 National Mission For Sustaining The Himalayan Ecosystem: Protect and preserve the
Himalayan ecosystem, which is highly vulnerable to climate change. It is executed under the
Department of Science & Technology, Ministry of Science & Technology.
 National Solar Mission: It aims to increase the share of solar energy in India's energy mix. It is
executed under the Ministry of New and Renewable Energy (MNRE)
15 www.visionias.in ©Vision IAS
 National Mission on Strategic Knowledge for Climate Change (NMSKCC): It aims to
generate and disseminate knowledge about climate change to inform policy decisions and
adaptation strategies. It is executed under the Department of Science & Technology, Ministry of
Science & Technology.
 National Mission on Natural Farming (NMNF)
o The National Mission on Natural Farming (NMNF) is a separate and independent scheme launched
by the Government of India in 2023-24. It aims to promote natural farming practices in India and to
reduce the use of chemical fertilizers and pesticides. It is not included in NAPCC.
o To motivate farmers to adopt chemical-free farming and enhance the reach of natural farming, the
Government has formulated the National Mission on Natural Farming (NMNF) as a separate and
independent scheme from 2023-24 by upscaling the Bhartiya Prakritik Krishi Paddati (BPKP).
o The success of NMNF will require behavioral change in farmers to shift from chemical-based inputs
to cow-based locally produced inputs and thus requires continuous creation of awareness, training,
handholding, and capacity building of farmers in the initial years.
 Hence Option (a) is the correct answer.

Q 36.D
 Recent context: India is embarking on a major program to launch its maiden national-level framework
towards providing climate services and information.
o Spearheaded by the India Meteorological Department (IMD), the National Framework for Climate
Services (NFCS) envisions bringing a seamless working platform for users of climate information and
services and helping decide and mitigate climate risks for key sectors — agriculture, energy, disaster
management, health, and water.
 In line with the global framework, the national framework will be based on country-specific weather and
stakeholder needs. Unlike the GFCS, the nodal agency for the formulation and implementation of the
national framework in India will be the IMD.
 The Global Framework for Climate Services (GFCS) was established by the international
community at the World Climate Conference-3 in 2009 to enable better management of the risks of
climate variability and change, and adaptation to climate change, through the development and
incorporation of science-based climate information and prediction into planning, policy and practice on
the global, regional and national scale. Hence statements 1 and 2 are not correct.
o At the invitation of the Government of Switzerland, World Climate Conference-3 (WCC-3) was held
in Geneva, Switzerland, from 31 August to 4 September 2009.
o It was organized by the World Meteorological Organization (WMO), in collaboration with the United
Nations Educational, Scientific and Cultural Organization (UNESCO), the United Nations
Environment Programme (UNEP), the Food and Agriculture Organization of the United Nations
(FAO), the International Council for Science (ICSU) and other intergovernmental and non-
governmental partners.
 The main sources of GFCS funding thus far are Member’s contributions to the GFCS Trust Fund
or through bilateral and multi-lateral investments for projects in selected countries or regions.
o In addition, various actors can support projects – listed in a compendium of GFCS projects – or
designate their activities as contributing to the GFCS if they fulfill the criteria endorsed by the
Intergovernmental Board on Climate Services (IBCS)
Q 37.D
 Recent context: A new study has revealed that two species of mosquitofish also known as Gambusia Fish
have invaded various ecosystems across India.
 Gambusia fish are native to the southeastern United States and parts of Mexico, but they have been
introduced to various regions worldwide for mosquito control.
 Gambusia fish are often used for biological mosquito control. They are known for their voracious appetite
for mosquito larvae. Introducing these fish into water bodies can help reduce mosquito populations.
 Gambusia fish are livebearers, meaning they give birth to live young ones instead of laying eggs.
They are known for their rapid reproduction rates.
 While Gambusia fish are effective at controlling mosquito larvae, their introduction to non-native
habitats has been controversial. In some cases, they have outcompeted or preyed upon native fish
species, affecting the local ecosystem. Hence statement 1 is not correct.
o Wildlife biologists and conservations consider mosquitofish to be among the hundred most
detrimental invasive alien species. Thus they are not critically endangered. Hence statement 2 is
not correct.

16 www.visionias.in ©Vision IAS


o In 2018, the National Biodiversity Authority of the Government of India also designated G. affinis
and G. holbrooki as invasive alien species.
 Gambusia Fish do not have any symbiotic relationship with Olive Ridley Turtles. Hence statement 3
is not correct.
Q 38.B
 The Convention on International Trade in Endangered Species of Wild Fauna and Flora (CITES):
o The Convention on International Trade in Endangered Species of Wild Fauna and Flora (CITES) is an
international agreement between governments that aims to ensure that international trade in specimens
of wild animals and plants does not threaten the survival of the species in the wild. It is a legally
binding agreement that is implemented by the Parties to the Convention, which are currently 183
countries. Hence statement 1 is not correct.
o Although CITES is legally binding on the Parties – in other words, they have to implement the
Convention – it does not take the place of national laws.
o CITES was drafted as a result of a resolution adopted in 1963 at a meeting of members of IUCN (The
World Conservation Union).
 CITES Implementation:
CITES is implemented through a system of permits and quotas. All international trade in specimens of
species listed on the CITES Appendices must be accompanied by a permit from the appropriate export or
import authority. Quotas are also used to control the trade in certain species, such as elephants and tigers.
o CITES Appendices: CITES has three Appendices:
 Appendix I: Includes all species threatened with extinction that are or may be affected by
international trade.
 Appendix II: Includes all species which are not necessarily threatened with extinction but which
may become so if trade is not strictly controlled.
 Appendix III: Includes all species which any Party to the Convention identifies as being subject
to regulation within its jurisdiction for the purpose of preventing or restricting exploitation and
which it has requested the Conference of the Parties to the Convention to add to the Appendices.
 The CITES Secretariat
The CITES Secretariat is administered by UNEP and is located in Geneva, Switzerland. It has a
pivotal role, fundamental to the Convention, and its functions are laid down in Article XII of the text of
the Convention. Hence statement 2 is correct. They include:
o Playing a coordinating, advisory, and servicing role in the working of the Convention.
o Assisting with communication and monitoring the implementation of the Convention to ensure that its
provisions are respected.
o Providing assistance in the fields of legislation, enforcement, science, and training.
Q 39.A
 The International Whaling Commission (IWC):
The International Whaling Commission (IWC) was established in 1946 to regulate whaling and conserve
whale stocks. The Commission has 88 member countries and meets every other year to review the status
of whale stocks and adjust conservation measures.
The IWC regulates three types of whaling: commercial, scientific research, and aboriginal subsistence
whaling.
 Commercial Whaling
o In 1986, a global moratorium was placed on commercial whaling due to overexploitation of
whale stocks. However, some countries, such as Norway and Iceland, continue to engage in
commercial whaling. Hence statement 1 is correct.
o In recent years, Norway and Iceland have caught whales commercially. The Government of Norway
lodged a formal objection to the moratorium decision when it was introduced. Iceland left the IWC in
1992 and re-joined in 2002 with a reservation to the moratorium. When the moratorium was
introduced, the Russian Federation also registered an objection but did not exercise it. The
moratorium is binding on all other members of the IWC.
 Scientific research
o Scientific research whaling is permitted under special permits issued by the IWC. Japan's lethal
scientific research in the North Pacific and Southern Oceans ended in 2019.
 Subsistence whaling
o Aboriginal subsistence whaling is also regulated by the IWC. Aboriginal subsistence whaling is
the practice of hunting and killing whales for food, cultural, and spiritual purposes by
indigenous peoples who have a long history of whaling traditions. Hence statement 2 is not
correct.
17 www.visionias.in ©Vision IAS
o Indigenous communities in Denmark, the Russian Federation, St. Vincent and the Grenadines, and the
United States are permitted to engage in this type of whaling.
o The IWC places a strong emphasis on scientific advice and has established a Scientific Committee to
provide guidance. The Scientific Committee meets annually and has produced catch-limit algorithms
to ensure sustainable whaling practices.
 Members
o Membership of the IWC is open to any country in the world. Each member formally adheres to the
1946 International Convention for the Regulation of Whaling and is represented by a Commissioner
who is nominated by their government. Currently, there are 88 member nations in the IWC.
o India has been a member of the International Whaling Commission since 1981 and has
maintained a consistent stand on whale conservation.
o Japan left the IWC in 2019 and began to catch whales commercially the same year. Having left
the IWC is no longer bound by the moratorium. Japanese catches are also reported to the IWC.
 Other G2O nations that are not members of the IWC include Canada, Indonesia, Saudi Arabia, and
Turkiye along with Japan.
Q 40.C
 Indo-Pacific Partnership for Maritime Domain Awareness (IPMDA) is a technology and training
initiative to enhance maritime domain awareness in the Indo-Pacific region and to bring increased
transparency to its critical waterways. IPMDA harnesses innovative technology, such as commercial
satellite radio frequency data collection, to provide partners across Southeast Asia, the Indian Ocean
region, and the Pacific with near real-time information on activities occurring in their maritime zones.
 To contribute to maritime domain awareness in the Indo-Pacific region, which is fundamental for stability
and prosperity. At the 2022 Quad Leaders’ Summit in Tokyo, Quad Leaders announced the Indo-
Pacific Partnership for Maritime Domain Awareness (IPMDA) to enhance existing maritime domain
awareness capabilities.
 Only QUAD countries are members of the IPMDA initiative, which are India, Japan, the USA, and
Australia.
 Hence option (c) is the correct answer.

Q 41.B
 Statement 1 is not correct: Global Footprint Network is an independent think tank originally founded in
2003. It was established as a charitable not-for-profit organization. Global Footprint Network develops
and promotes tools for advancing sustainability, including the ecological footprint and biocapacity, which
measure the amount of resources we use and how much we have. These tools aim at bringing ecological
limits to the center of decision-making.
 Every year, Global Footprint Network produces a new edition of its National Footprint Accounts, which
calculate Ecological Footprint and biocapacity of more than 200 countries and territories from 1961 to the
present. It has also launched Ecological Footprint Explorer, an open data platform for the National
Footprint Accounts.
 Statement 2 is correct: Earth Overshoot Day marks the date when humanity’s demand for ecological
resources and services in a given year exceeds what Earth can regenerate in that year. Earth Overshoot
Day is hosted and calculated by Global Footprint Network.
 World Summit on Sustainable Development, 2002 (WSSD)
o Also known as the Earth Summit, it was held in Johannesburg, South Africa. It was convened to
discuss sustainable development by the United Nations. WSSD gathered a number of leaders from
business and non-governmental organizations, 10 years after the first Earth Summit in Rio de Janeiro.
(It was therefore also informally nicknamed "Rio+10").
Q 42.D
 The National River Conservation Plan (NRCP) was initiated in 1995. Subsequently, all the projects for
river cleaning in the country were brought under NRCP. It is under the purview of Ministry of
Environment, Forest & Climate Change. Hence, option (a) is not correct.
 The objective of the NRCP is to reduce the pollution load in rivers through implementation of various
pollution abatement works which include -:
o Interception and diversion works/ laying of sewerage systems to capture raw sewage flowing into the
rivers through open drains and diverting them for treatment.
o Setting up of Sewage Treatment Plants (STPs) for treating the diverted sewage
o Construction of Low Cost Sanitation Toilets to prevent open defecation on river banks.
o Construction of Electric Crematoria and Improved Wood Crematoria to conserve the use of wood.

18 www.visionias.in ©Vision IAS


o River Front Development works, such as improvement of bathing ghats.
o Public participation & awareness and capacity building, etc.
 Hence, option (d) is correct.
 NRCP doesn’t have any sub-schemes. National Lake Conservation Programme is a separate Centrally
Sponsored Schemes (CSS) under Ministry of Environment, Forest & Climate Change with an objective to
restore and conserve the urban and semi-urban lakes of the country. Since 2013, National Lake
Conservation Programme along with the National Wetlands Conservation Programme (NWCP) have been
merged into a new integrated scheme named ‘National Plan for Conservation of Aquatic Eco-systems
NPCA’. Hence, option (b) is not correct.
 Currently, NRCP (excluding Ganga and its tributaries) has covered polluted stretches of 33 rivers in 76
towns spread over 15 States at a sanctioned cost of Rs.4801.57 crore. NRCP covers the following states -
Andhra Pradesh; Goa; Gujarat; Jammu & Kashmir; Jharkhand; Karnataka; Kerala; Madhya Pradesh;
Maharashtra; Nagaland; Odisha; Punjab; Sikkim; Tamil Nadu; Telangana. Hence, option (c) is not
correct.

Q 43.A
 Recently, at COP28, 196 parties adopted the decision to operationalise the Loss and Damage fund.
Hence statement 2 is not correct.
 Loss and damage refers to the negative consequences that arise from the unavoidable risks of climate
change, like rising sea levels, prolonged heatwaves, desertification, the acidification of the sea and
extreme events, such as bushfires, species extinction and crop failures.
 The fund was set up during COP27 at Sharm el-Shaikh in Egypt, 2022. The establishment of a Loss
and Damage Fund was, for many, the highlight of the United Nations Climate Conference (COP 27) and
the culmination of decades of pressure from climate-vulnerable developing countries.
 The fund aims to provide financial assistance to nations most vulnerable and impacted by the effects of
climate change. Hence sattement 1 is correct.

Q 44.A
 Recently, the Prime Minister launched the ₹24,000-crore Pradhan Mantri Particularly Vulnerable
Tribal Groups (PM PVTG) Mission aimed at the holistic development of around 28 lakh primitive
tribals spread across 220 districts across the country, on the occasion of tribal icon Birsa Munda’s
birth anniversary and the third “Janjatiya Gaurav Diwas.
 The mission is aimed to improve the socio-economic conditions of the 75 particularly vulnerable
tribal groups (PVTGs) by saturating PVTG families and habitations with basic facilities such as safe
housing, clean drinking water, and sanitation, improved access to education, health and nutrition, road and
telecom connectivity, and sustainable livelihood opportunities. Hence statement 1 is correct.
 PVTGs are characterized by a “pre-agriculture level of technology, stagnant or declining population,
extremely low literacy, and subsistence level of the economy. PVTGs were recognized as a separate
category based on the findings of the 1961 Dhebar Commission.

19 www.visionias.in ©Vision IAS


 To ensure coordination and effective implementation, The ministry has appointed 1 nodal officer
for each PVTG community and they are visiting their habitations to understand their
requirements. Hence statement 2 is not correct.
 The Ministry of Tribal Affairs is the Nodal Ministry for overall policy planning and coordination of
programs for the development of STs. Hence statement 3 is not correct.
 Hence option (a) is the correct answer.

Q 45.A
 Central Pollution Control Board is executing a nationwide programme of ambient air quality monitoring
known as the National Air Quality Monitoring Programme (NAMP).
 The network consists of 804 operating stations covering 344 cities/towns in 28 states and 6 Union
Territories of the country.
 The objectives of the N.A.M.P. are to determine the status and trends of ambient air quality;
o to ascertain whether the prescribed ambient air quality standards are violated;
o to Identify Non-attainment Cities;
o to obtain the knowledge and understanding necessary for developing preventive and corrective
measures and
o to understand the natural cleansing process undergoing in the environment through pollution dilution,
dispersion, wind-based movement, dry deposition, precipitation and chemical transformation of
pollutants generated.
 Under N.A.M.P., four air pollutants viz ., Sulphur Dioxide (SO2), Oxides of Nitrogen as NO2,
Respirable Suspended Particulate Matter (RSPM / PM10) and Fine Particulate Matter
(PM2.5) have been identified for regular monitoring at all the locations.
 Monitoring meteorological parameters such as wind speed and wind direction, relative humidity
(RH) and temperature were also integrated with monitoring air quality.
 Hence, option (a) is the correct answer.

Q 46.D
 Climate finance is one of the key agenda items up for discussion at the much-awaited 28th Conference of
Parties (COP28) to the United Nations Framework Convention on Climate Change to be held in Dubai
later this month.
 Climate Policy Initiative’s (CPI) new report Global Landscape of Climate Finance 2023 brings
together the latest data and analysis in this regard.
 The report breaks down the flow of climate finance by its application, geographical distribution and
sources, using data from 2021 and 2022. The flows tracked in the report “represent targeted climate
mitigation and adaptation-specific project-level allocation of capital”
 The Climate Policy Initiative: It is an independent non-profit research group and international
climate policy organization based in San Francisco, California with other offices worldwide. CPI is
supported primarily by philanthropic organizations and government development finance.

Q 47.B
 Recent context: The Government of India has given the approval for "Acceptance of Necessity" for
Project Kusha recently.
 Project Kusha focuses on the development of Long-Range Surface-to-Air Missiles (LR-SAM) and
ultimately an air defense system for India. Hence statement 1 is not correct.
 It is being jointly developed with Israel Aerospace Industries, Israel’s major aerospace and aviation
manufacturers. Hence statement 2 is correct.
 The mobile LR-SAM, with its long-range surveillance and fire control radars, would have different types
of interceptor missiles designed to hit hostile targets at 150 km, 250 km, and 350 km ranges.
 LR-SAM isn’t the only project India is developing with Israel. India’s Kalyani Strategic Systems has
already entered a joint venture with Israel’s Rafael Advanced Systems Ltd for indigenous production of
Medium-Range Surface to Air Missile (MRSAM).
Q 48.B
Coal Sector in India:
 India has the fifth largest coal reserves, is second in coal production (after China) and is among the
largest importer of coal (mainly from Indonesia, South Africa and Australia). Hence statement 1 is
correct.
20 www.visionias.in ©Vision IAS
 Coal accounts for almost 50% share in the total installed electricity generation capacity in India.
 Indian coal has high ash content, high ash fusion temperature but low sulphur content. Hence
statement 3 is not correct.
Distribution of Coal in India:
 Gondwana Coal Fields (250 million years old): Gondwana coal makes up to 98 % of the total
reserves and 99 % of the production of coal in India. Hence statement 2 is correct.
 Gondwana coal forms India’s metallurgical grade as well as superior quality coal.
 It is found in Damodar (Jharkhand-West Bengal), Mahanadi (Chhattisgarh-Odisha), Godavari
(Maharashtra), and Narmada valleys.
 Tertiary Coal Fields (15 – 60 million years old): Carbon content is very low but is rich in moisture
and Sulphur.
 Tertiary coalfields are mainly confined to extra-peninsular regions Important areas include Assam,
Meghalaya, Nagaland, Arunachal Pradesh, Jammu and Kashmir, Himalayan foothills of Darjeeling in
West Bengal, Rajasthan, Uttar Pradesh, and Kerala.

Q 49.D
 Tuberculosis, often called TB, is a bacterial infection that primarily affects the lungs. It's caused
by Mycobacterium tuberculosis and usually spreads through the air when an infected person coughs or
sneezes. Symptoms include coughing, chest pain, fatigue, weight loss, and fever.
 Mycobacterium tuberculosis is the primary agent responsible for TB and not Staphylococcus
aureus. Hence statement 1 is not correct.
 The bacteria usually attack the lungs, but TB bacteria can attack any part of the body such as the
kidney, spine, and brain. Not everyone infected with TB bacteria becomes sick. As a result, two TB-
related conditions exist: latent TB infection (LTBI) and TB disease. Hence statement 2 is not correct.
 The Mantoux tuberculin skin test (TST) is one method of determining whether a person is infected
with Mycobacterium tuberculosis, and not to monitor sugar level. Hence statement 3 is not correct.
 As per the global TB Report 2023, the Reduction of TB incidence has almost doubled the pace of the
Decline in global TB Incidence, Improvement in treatment coverage to 80% of the estimated TB cases;
an increase of 19% over the previous year, In India TB incidence declined by 16% and TB mortality
reduction by 18% since 2015.

Q 50.B
 With a population of 1.3 billion, India has a massive demand for energy to fuel its rapidly growing
economy. From a power deficit nation at the time of Independence, the efforts to make India energy-
independent have continued for over seven decades. Today, we are a power surplus nation with a total
installed electricity capacity of over four lakh MW.
 Keeping in mind the sustainable development goals, India's power generation mix is rapidly shifting
towards a more significant share of renewable energy. Today, India is the world's third largest
producer of renewable energy, with 40% of its installed electricity capacity coming from non-fossil
fuel sources.
 India has an estimated renewable energy potential of about 900 GW from commercially exploitable
sources, viz., wind - 102 GW (at 80-metre mast height); small hydro - 20 GW; bioenergy - 25 GW;
and 750 GW solar power, assuming 3 percent wasteland is made available. Hence, option (b) is the
correct answer.

Q 51.B
India’s Solar Photovoltaic (PV) Capacity:
 As of the latest data available from Ministry of New & Renewable Energy (MNRE), India’s solar
photovoltaic (PV) capacity has surged to an impressive 71,780.74 MW (71.78 GW) and is is fourth
largest in the world. Hence statement 1 is not correct.
 This capacity is distributed across various segments of the solar energy sector, with rooftop solar,
ground-mounted solar, hybrid projects, and off-grid solar playing significant roles in this energy
revolution.
 Ground-mounted solar plants has the largest share followed by roof-top and hybrid projects
respectively. Hence statement 2 is correct.

21 www.visionias.in ©Vision IAS


Q 52.C
 The World Heritage Outlook Report-IUCN:
o The World Heritage Outlook Report is the flagship assessment of the status of natural World Heritage
sites by the International Union for Conservation of Nature (IUCN). The report is produced every 3
years and provides an overview of the conservation status of all natural World Heritage sites, as well
as trends and challenges. Hence option (c) is the correct answer.
 The latest World Heritage Outlook Report, 2020:
o It found that the overall outlook for natural World Heritage sites is of concern, with only 63% of sites
assessed as being in good or good with some concerns condition. The remaining 37% of sites were
assessed as being of significant concern or critical condition.
o The report identified a number of factors that are threatening the status of natural World Heritage
sites, including:
 Climate change
 Habitat loss and fragmentation
 Invasive alien species
 Pollution
 Unsustainable tourism
 Overexploitation of natural resources
 The report also found that there has been some progress in conserving natural World Heritage sites in
recent years. For example, the number of sites that are well-managed has increased from 25% in 2000 to
38% in 2020. However, the report warned that this progress is not enough to keep pace with the threats
that natural World Heritage sites face.

Q 53.C
 The Perform Achieve and Trade Scheme is one of the initiatives under the National Mission on Enhanced
Energy Efficiency (NMEEE). Energy Saving Certificate (ESCert) under the Perform, Trade and
Achieve (PAT) scheme is a market-based mechanism to reduce the specific energy consumption (SEC)
in energy-intensive large industries. Hence statements 1 and 3 are correct.
 This mechanism is facilitated through the trading of Energy Saving Certificates (ESCerts) which are
issued to those plants that have overachieved their targets. Those plants which underachieve their targets
are entitled to purchase ESCerts.
 As per PAT rules, when a designated consumer overachieves the notified SEC targets in the compliance
year, the ESCerts are to be issued by Central Government for the difference of quantity between notified
target and the achieved SEC.
 ESCerts are traded on Indian Energy Exchange (IEX) and Power Exchange India Limited (PXIL).
ESCerts trading is regulated by the Central Electricity Regulatory Commission (CERC) with the Bureau
of Energy Efficiency (BEE), Ministry of Power as the Administrator. Hence statement 2 is not correct.

Q 54.C
 Air pollution control devices (APCD) are a series of devices which are used to prevent a variety of
different pollutants, both gaseous and solid, from entering the atmosphere mainly out of the industrial
stacks.
22 www.visionias.in ©Vision IAS
 These control devices can be separated into two broad categories namely:
o devices which control the amount of particulate matter escaping into the environment, and
o devices which control the acidic gas emissions into the atmosphere.
 The APCDs collect particulate emissions work on the principle of (i) gravity separation, (ii) cyclonic
separation, (iii) filtration, (iv) electrostatic precipitation, and (v) wet scrubbing.
 The most commonly used APCDs are:
o dust catchers
o cyclones or multi-cyclones,
o fabric filters also known as bag houses,
o electrostatic precipitators (wet and dry types), and
o scrubbers.
 The first four control devices control the amount of particulate matter escaping into the environment while
scrubbers control the acidic gas emissions into the atmosphere.
 Hence, option (c) is the correct answer.

Q 55.A
 The International Tropical Timber Organization (ITTO):
o The International Tropical Timber Organization (ITTO) is an intergovernmental organization that
promotes the sustainable management and conservation of tropical forests and the expansion and
diversification of international trade in tropical timber from sustainably managed and legally
harvested forests. Hence statement 1 is not correct.
o It was established in 1986 under the International Tropical Timber Agreement (ITTA), which
was sponsored by the United Nations Conference on Trade and Development (UNCTAD) and ratified
in 1985.
 The main objectives of the ITTO are to:
o Develop internationally agreed policy guidelines and norms to encourage sustainable forest
management (SFM) and sustainable tropical timber industries and trade.
o Assist tropical member countries in adapting such guidelines and norms to local circumstances and to
implement them in the field through projects and other activities.
o Collect, analyze, and disseminate data on the production and trade of tropical timber.

 Members:
ITTO divides its members into two categories - Producer Countries and Consumer Countries.
 Producer Countries:
o Producer countries are countries that are located between the Tropic of Cancer and the Tropic of
Capricorn, have tropical forest resources, and/or are net exporters of tropical timber in volume terms.
These countries are typically developing countries that rely on the export of tropical timber products
for a significant portion of their income.
o There are 37 Producer Countries which include Bolivia, Nigeria, India, etc. Hence statement 3 is
correct.
 Consumer Countries:
o Consumer countries are countries that are located outside of the tropics and generally import more
primary tropical wood products than they export. These countries are typically developed countries
that have a high demand for tropical timber products.
o There are 38 Consumer countries including Albania, Australia, China, etc.
The ITTO is open to all countries, regardless of whether they have tropical forests or engage in
the timber trade. As of today, the ITTO has 156 member countries, including developed and
developing countries with and without tropical forests. Hence statement 2 is not correct.
 Structure of the ITTO
o The ITTO is governed by the International Tropical Timber Council (ITTC), which is composed of
representatives from all ITTO member countries. The ITTC meets every two years to deliberate
and adopt decisions on environmental matters.
Q 56.A
 The Graded Response Action Plan is a set of anti-air pollution measures, generally implemented in
Delhi-NCR during the winter. Hence, statement 1 is not correct.
 The Commission for Air Quality Management (CAQM), an autonomous body tasked with
improving the air quality in Delhi and its adjoining areas, made crucial changes to the Graded
Response Action Plan (GRAP) last year and again in July. Hence, statement 2 is correct.
23 www.visionias.in ©Vision IAS
 GRAP for the NCR is implemented in four different stages based on the stages of adverse air quality in
Delhi. Stage 1 - 'poor' (AQI 201-300); Stage 2 - 'very poor' (AQI 301-400); Stage 3 - 'severe' (AQI 401-
450); and Stage 4 - 'severe plus' (AQI >450).
 Under the new plan, BS-III petrol and BS-IV diesel four-wheelers will be immediately banned in Delhi
and Gurugram, Faridabad, Ghaziabad and Gautam Buddh Nagar if the AQI crosses the 400-mark.
 The CAQM, a statutory body formed under an Act in 2021, amended the GRAP in 2022 to ensure
proactive implementation of anti-air pollution steps based on forecasts up to three days in advance.
 Earlier, the authorities would implement these measures, including a ban on construction and demolition,
entry of high-emission vehicles and the use of coal and firewood, only after the pollution level touched a
particular threshold.
 It also calls for a complete ban on the use of coal and firewood in eateries, restaurants and hotels as
soon as the AQI crosses the 200 mark. Earlier, the authorities would implement this measure under
Stage 2. Hence, statement 3 is not correct.New steps suggested as part of the revised GRAP include
"strict action to curb air pollution at all identified hotspots in the region under Stage 2".
 Under Stage 3, states will have to impose strict restrictions on the plying of BS-III petrol and BS-IV diesel
four-wheelers in Delhi and Gurugram, Faridabad, Ghaziabad and Gautam Buddh Nagar. The authorities
may discontinue physical classes in schools for children up to Class 5 in such a scenario, the CAQM
suggested.
 At AQI above 450, four-wheelers registered outside Delhi, other than electric vehicles and those using
CNG and BS-VI diesel, will not be permitted to enter Delhi. However, those carrying essential
commodities or providing essential services are exempt.
 All other existing measures taken under different stages of GRAP will continue.

Q 57.C
 Champions of the Earth award honour individuals, groups, and organizations whose actions have
transformative impact on environment. Hence statement 2 is correct.
 Awarded annually since its inception in 2005, it is UN’s highest environmental honour which
recognizes outstanding leaders from government, civil society and private sector. Hence statement 1
is correct.
 To eliminate plastic pollution, 2023 award recognized in four categories. Hence statement 3 is
correct.
o Policy leadership category: Mayor Josefina Belmonte of Quezon City (Philippines) to ban single-use
plastics, a trade-in programme for plastic pollution etc.
o Inspiration and Action category: Ellen MacArthur Foundation (United Kingdom) in mainstreaming
lifecycle approach, including for plastics.
o Entrepreneurial Vision category:
 Blue Circle (China) uses blockchain technology and internet of things to track and monitor full
lifecycle of plastic pollution.
 José Manuel Moller (Chile) is founder social enterprise (Algramo) dedicated to providing refill
services that reduce plastic pollution and lower everyday essentials costs.
o Science and Innovation category: Council for Scientific and Industrial Research (South Africa) uses
cutting-edge technology and multidisciplinary research to develop innovations to tackle plastic
pollution.

Q 58.C
 Recent context: Six Indian restaurants from around the world serving desi cuisine have been chosen for
this year’s Annapurna Certificate Programme.
 It is an initiative by the Indian Council for Cultural Relations (ICCR) to recognize Indian
restaurants abroad that are contributing to the country’s cultural cause in the culinary way. Hence
both statements 1 and 2 are correct.
 Those who were shortlisted from a total of 115 nominations received from Indian missions abroad are
Balaji Dosa, Kandy, Sri Lanka; Indian Street Food & Co., Stockholm, Sweden; Amber India Restaurant,
San Francisco, USA; Naans and Curries, An Ethnic Indian Restaurant, San Jose, Costa Rica; Mumtaz
Mahal Restaurant, Muscat, Oman; and Namaste Indian Restaurant, Ulaanbaatar, Mongolia.
 The Indian Council for Cultural Relations, is an autonomous organization of the Government of
India, involved in India's global cultural relations, through cultural exchange with other countries
and their people.

24 www.visionias.in ©Vision IAS


o The Indian Council for Cultural Relations (ICCR) was founded in 1950 by Maulana Abul Kalam
Azad, independent India’s first Education Minister.
o Its objectives are to actively participate in the formulation and implementation of policies and
programs about India’s external cultural relations
o To foster and strengthen cultural relations and mutual understanding between India and other
countries; to promote cultural exchanges with other countries and people, and to develop relations
with nations.

Q 59.B
 BHARAT TAP initiative and Nirmal Jal Prayas initiative are under the Ministry of Housing and
Urban Affairs.
 BHARAT TAP initiative:
o Focus: Promotes low-flow fixtures and sanitaryware to reduce water usage.
o Goal: Aim to achieve a minimum 40% reduction in water consumption through taps, showers, and
other fittings.
o Implementation: Encourages the use of Bureau of Indian Standards (BIS) certified water-efficient
plumbing fixtures.
o Benefits: Reduced water usage, lower water bills, lessened strain on water resources, and potentially
decreased energy consumption for water treatment and pumping.
o Launched by: Ministry of Housing and Urban Affairs (MoHUA) in collaboration with the Indian
Plumbing Association (IPA).
 Nirmal Jal Prayas:
o Focus: Tackles wastewater management and promotes sustainable water reuse.
o Goal: Aim to save 500 crore litres of water per year through wastewater treatment and reuse.
o Implementation: Supports development and adoption of wastewater treatment technologies,
encourages rainwater harvesting, and promotes public awareness campaigns.
o Benefits: Reduced reliance on freshwater sources, improved sanitation and hygiene, potential for
agricultural irrigation or industrial use of treated wastewater.
o Launched by: National Real Estate Development Council (NAREDCO) through its MAHI (Municipal
Authorities Helping Infrastructure) initiative.
 The National Real Estate Development Council(NAREDCO) was established as an autonomous self-
regulatory body in 1998 under the aegis of the Ministry of Housing and Urban Affairs.
 Hence, option (b) is the correct answer.

Q 60.D
 About Biofuels
o Refers to liquid transportation fuels, such as ethanol and biodiesel, derived from agricultural produce,
forests or any other organic material (feedstock).
o Based on the feedstock (raw materials) used to produce biofuels they are classified into four
generations.
o Presently, first-generation biofuels are the main source of biofuels globally despite 140 billion
tonnes of agro-waste or biomass generated from agriculture every year.
o First Generation Biofuels: These are made from food sources such as sugar, starch, vegetable
oil, or animal fats using conventional technology. They include Bioalcohols, Biodiesel, Vegetable
oil, Bioethers, Biogas. Hence pair 1 is correctly matched.
o Second Generation Biofuels: These are produced from non-food crops or portions of food crops
that are not edible and considered as wastes, e.g., stems, husks, wood chips, and fruit skins and
peeling. Hence pair 2 is correctly matched.
o Third Generation Biofuels: These are produced from micro-organisms like algae. Hence pair 3
is correctly matched.
o Fourth Generation Biofuels: In the production of these fuels, crops that are genetically engineered
to take in high amounts of carbon are grown and harvested as biomass. The crops are then
converted into fuel using second-generation techniques. Hence pair 4 is correctly matched.
o The fuel is pre-combusted, and the carbon is captured. Then the carbon is geo-sequestered, meaning
that the carbon is stored in depleted oil or gas fields or in unmineable coal seams.

25 www.visionias.in ©Vision IAS


Q 61.D
 The State of the World's Forests Report: The State of the World's Forests Report, is a flagship
publication of the Food and Agriculture Organization of the United Nations (FAO). It is published
every two years and provides a comprehensive overview of the state of the world's forests. The recent
publication is released in 2022 which focuses on the potential of forests to contribute to green recovery
and a transition to sustainable economies. Hence option (d) is the correct answer.
 Key Findings
 The 2022 edition of The State of the World's Forests found that:
o Deforestation and forest degradation are continuing at an alarming rate.
o Forests are facing a number of other threats, including climate change, wildfires, and invasive species.
o Despite these threats, forests still provide a wide range of benefits to people and the planet.
o Forests can play a key role in green recovery and a transition to sustainable economies.
 Recommendations
o The report makes a number of recommendations for how to protect and sustainably manage forests,
including:
 Halting deforestation and restoring degraded lands
 Investing in sustainable forest management
 Promoting forest-based livelihoods
 Strengthening international cooperation on forest issues

Q 62.D
 The Bharat Stage emission standards are standards instituted by the government to regulate the output
of air pollutants from motor vehicles from internal combustion engine equipment, including motor
vehicles. India has been following European (Euro) emission norms, although with a time lag of five
years.
 In 2016, the government said India would directly progress from BS-IV norms to BS-VI, skipping the
intermediary stage of BS-V. In April 2020, India moved towards more stringent BS-VI norms. Hence
statement 1 is not correct.
 The main difference in standards between the existing BS-IV and the new BS-VI auto fuel norms is the
presence of sulphur. The BS-VI fuel is estimated to bring around an 80 per cent reduction of sulphur,
from 50 parts per million to 10 ppm. Hence statement 3 is not correct.
 While the difference in emission levels is not that drastic for petrol engines, it is significant for diesel
vehicles. For example, the nitrous oxide emission levels (responsible for acid rain) will drop by 25
percent in the case of petrol vehicles and 68 percent for diesel vehicles. Hence getting a BS-IV diesel
engine to comply with BS-VI emission norms requires major changes to the hardware and layout of a
diesel engine compared to a petrol engine. That is why many car manufacturers in India are already
rolling out BS-VI compliant petrol models, but holding back on diesel models. Hence statement 2 is not
correct.
 The new technologies needed to be included in the vehicles include Diesel Particulate filter to expel
particulate matters, Selective catalytic reduction and exhaust gas regulator to reduce NOx emissions.

26 www.visionias.in ©Vision IAS


Q 63.B
 Short-lived climate pollutants are powerful climate forcers that remain in the atmosphere for a
much shorter period of time than carbon dioxide, yet their potential to warm the atmosphere can be
many times greater. Hence, statement 1 is not correct.
 Certain short-lived climate pollutants are also air pollutants that have harmful effects on people,
ecosystems and agricultural productivity.
 The short-lived climate pollutants black carbon, methane, tropospheric ozone, and
hydrofluorocarbons (HFCs) are the most important contributors to anthropogenic global warming
after carbon dioxide, responsible for up to 45% ( according to CCAP a UN convened initiative) of
current global warming. Hence, statement 2 is correct.
 If no action to reduce emissions of these pollutants is taken in the coming decades, they are expected to
account for as much as half of the warming caused by human activity.
 In 2011 The United Nations Environment Programme (UNEP) and the World Meteorological
Organisation (WMO) found that readily available solutions targeting compounds known as short-lived
climate pollutants (SLCPs) would slow the rate of global warming much faster than action on carbon
dioxide alone. These solutions would also reduce air pollution, representing “win-win” results for the
climate, air quality, and human well-being within a relatively short timeframe.
 In 2012, the importance of seizing this opportunity to reduce SLCPs and achieve the associated climate
and development benefits led to action by the governments of Bangladesh, Canada, Ghana, Mexico,
Sweden, and the United States, along with UNEP. They jointly set out to treat short-lived climate
pollutants as an urgent and collective challenge. Together, they formed the Climate & Clean Air
Coalition (CCAC) – with its Secretariat and Trust Fund hosted by UNEP – to support fast action and
deliver benefits across climate, public health, energy efficiency, and food security.
 Since its founding, the CCAC has raised awareness of the need to act on these pollutants to rapidly reduce
the rate of warming in the near term, improved the science behind them, and carried out activities in
emitting sectors to prove the feasibility of available solutions.
 It is the only international body working to connect climate and clean air action. Hence, statement 3
is correct.

Q 64.D
 Incineration is a waste treatment process that involves the combustion of organic substances
contained in waste materials. Incineration and other high-temperature waste treatment systems are
described as "thermal treatment". Hence the correct option is (d)
 Incineration of waste materials converts the waste into ash, flue gas and heat. The ash is mostly formed by
the inorganic constituents of the waste and may take the form of solid lumps or particulates carried by the
flue gas. The flue gases must be cleaned of gaseous and particulate pollutants before they are dispersed
into the atmosphere. In some cases, the heat that is generated by incineration can be used to generate
electric power.
 As of July 2017, thermal-based waste-to-energy (WTE) plants in India have the capacity to process
5,300 tonnes of garbage and produce 63.5 MW per day.
 According to a 2015-16 report by the Ministry of New and Renewable Energy (MNRE), this capacity
can be enhanced to 1,075 MW by 2031 and to 2,780 MW by 2050.
 The new policy seeks to increase the energy generated from solid waste to 330 MW per day in the next
five months, and over 400 percent increase from the current installed capacity, and to 511 MW by 2018-
19. To this end, five WTE plants are already under construction in the country and tenders have been
floated to build another 47.

27 www.visionias.in ©Vision IAS


Q 65.A
 Recent context: Lebanon accuses Israel of white phosphorus attacks.
 White phosphorus is a chemical waxy solid substance typically appearing yellowish or colorless, and
some have described its odor as resembling garlic.
o It ignites instantly upon contact with air. Hence statement 1 is correct.
o It is often used by militaries to illuminate battlefields, generate a smokescreen, and as an incendiary.
o Once ignited, white phosphorus is very difficult to extinguish. It sticks to surfaces like skin and
clothing.
o White phosphorus is harmful by all routes of exposure.
o White phosphorus can cause deep and severe burns, penetrating even through bone, and has been
known to reignite after initial treatment. After exposure, the priority is to stop the burning process.
 Convention on Certain Conventional Weapons (CCW) imposes restrictions on the use of white
phosphorus, to safeguard civilians. It is not banned. Hence statement 2 is not correct.
 The United Nations Convention on Certain Conventional Weapons, concluded in Geneva on October 10,
1980, and entered into force in December 1983, seeks to prohibit or restrict the use of certain conventional
weapons that are considered excessively injurious or whose effects are indiscriminate.

Q 66.B
 The Fagradalsfjall system is related to the volcanic system. It is situated on the Reykjanes Peninsula
in Iceland, a region known for its remarkable geological and geothermal features. It is the “world’s
newest baby volcano.” It had been dormant for eight centuries before erupting in 2021, 2022 and
2023.
 The Reykjanes Peninsula is located in the southwestern part of Iceland and is known for its proximity
to the Mid-Atlantic Ridge, a tectonic boundary where the North American and Eurasian tectonic
plates meet. This geological setting makes the Reykjanes Peninsula a hub of dynamic volcanic and
geothermal activity.
 The eruption marked the first time in over 800 years that a volcanic event had occurred in this particular
area. The eruption is characterized by its effusive style, where molten lava flows out relatively calmly
compared to more explosive volcanic eruptions.
 Deep under the Earth’s surface, intense heat melts rocks to form magma, a thick flowing substance
lighter than solid rock. This drives it upwards and most of it gets trapped in magma chambers deep
underground. Over time, this viscous liquid cools and solidifies once again. However, a tiny fraction
erupts through vents and fissures on the surface, causing volcanic eruptions. The movement of magma
close to the Earth’s surface exerts a force on the surrounding rock, which often causes earthquake
swarms.
 Hence option (b) is the correct answer.

Q 67.B
 Background
o India is grappling with a severe air pollution crisis, with many cities consistently exceeding safe limits
for particulate matter (PM) and other pollutants. In response to this growing concern, the
Government of India launched the National Clean Air Programme (NCAP) in 2019.
 What are the Target Levels?
o The NCAP initially set a target of reducing key air pollutants PM10 and PM2.5 by 20-30% in 2024,
taking the pollution levels in 2017 as the base year to improve upon.
o However, in September 2022, the Centre moved the goalposts and set a new target of a 40% reduction
in particulate matter concentration, by 2026. Hence Statement 1 is not correct.
 Objectives
o The NCAP is a comprehensive national-level program aimed at addressing air pollution across the
country. Its primary objectives are:
o Reduce Particulate Matter (PM) Concentrations: The NCAP aims to reduce PM concentrations by
20-30% in cities across India by 2024, and by 50% in non-attainment cities by the same year.
o Improve Air Quality: The NCAP aims to improve air quality in all cities to meet the National
Ambient Air Quality Standards (NAAQS) by 2024.
o Strengthen Air Quality Monitoring: The NCAP aims to strengthen air quality monitoring
infrastructure and data management across India.
 Key Components
The NCAP encompasses a range of measures to achieve its objectives, including:
28 www.visionias.in ©Vision IAS
o Strengthening Emission Standards: The NCAP proposes stricter emission standards for vehicles and
industries to reduce their contribution to air pollution.
o Promoting Cleaner Fuels and Technologies: The NCAP encourages the adoption of cleaner fuels,
such as compressed natural gas (CNG) and electric vehicles, and promotes the use of energy-efficient
technologies.
o Improving Urban Planning and Transportation Systems: The NCAP emphasizes the need for
better urban planning and transportation systems to reduce traffic congestion and associated
emissions.
o Strengthening Monitoring and Enforcement Mechanisms: The NCAP aims to strengthen
monitoring and enforcement mechanisms to ensure compliance with emission standards and
regulations.
 Implementation
The NCAP is being implemented by the Ministry of Environment, Forest and Climate Change
(MoEFCC) in collaboration with various state governments, municipal corporations, and industry
stakeholders. Hence statement 2 is correct.
o The program has a two-phase implementation plan:
 Phase 1 (2019-2024): Focus on reducing PM concentrations and improving air quality in non-
attainment cities.
 Phase 2 (2024-2028): Expand the scope of the program to cover all cities in India and achieve
further reductions in air pollution.

Q 68.B
 The Bank for International Settlements (BIS), in collaboration with the central banks of France,
Singapore, and Switzerland, has announced the successful conclusion of Project Mariana. Hence
statement 1 is not correct.
o Project Mariana was jointly developed by three BIS Innovation Hub centers, including the Swiss,
Singapore, and Eurosystem Hub Centers, in partnership with the Bank of France, the Monetary
Authority of Singapore, and the Swiss National Bank.
 This innovative project focused on testing the cross-border trading and settlement of wholesale
central bank digital currencies (wCBDCs) between financial institutions while incorporating
decentralized finance (DeFi) technology concepts on a public blockchain. Hence statement 2 is
correct.
 The project explored how multi-currency settlement may be performed atomically while maintaining the
independence of respective domestic settlement systems.
 The project's proof of concept achieved successful cross-border trading and settlement of hypothetical
euro, Singapore dollar, and Swiss franc WCBDCs between simulated financial institutions.
 Project Mariana's architectural design addresses the balance between central banks' need for domestic
oversight and autonomy and financial institutions' desire for efficient cross-border wCBDC management.
o This equilibrium is maintained through the use of a common token standard on a public blockchain,
facilitating interoperability and seamless exchange of wCBDCs across various local payment and
settlement systems managed by participating central banks.
o Consequently, Mariana offers possible strategies for incorporating an international dimension into
current wCBDC design explorations.

Q 69.C
 'SATAT’ (Sustainable Alternative Towards Affordable Transportation) scheme on Compressed Bio
Gas (CBG) was launched in 2018. The scheme envisages target production of 15 MMT (million tons)
of CBG 2023 from 5000 Plants. Independent entrepreneurs shall set up the CBG Plants. Hence,
statement 1 is correct.
 It encourages entrepreneurs to set up CBG plants, produce & supply CBG to Oil Marketing
Companies (OMCs) for sale as automotive & industrial fuels. Hence, statement 2 is correct.
 The SATAT initiative has the potential to boost the availability of more affordable transport fuels,
better use of agricultural residue, cattle dung and municipal solid waste, and provide additional revenue
sources to farmers. It will also benefit vehicle users as well as farmers andentrepreneurs. Hence,
statement 3 is correct.
 It will boost entrepreneurship, rural economy and employment and provide additional sources of
revenue to farmers. It will also help achievenation’s climate change goals and bring down dependency
on natural gas and crude oil imports and act as a buffer against crude oil and gasprice fluctuations.

29 www.visionias.in ©Vision IAS


 It will also help to integrate with existing networks such as city gas distribution (CGD) networks to
boost supplies to domestic and retail users in existing and upcoming markets. Besides retailing from OMC
fuel stations, CBS can at a later date be injected into CGD pipelines too forefficient distribution and
optimized access of cleaner and more affordable fuel.

Q 70.A
 The Invasive Species Specialist Group (ISSG) is a global network of scientific and policy experts on
invasive species, organized under the auspices of the Species Survival Commission (SSC) of the
International Union for Conservation of Nature (IUCN). Hence option (a) is the correct answer.
 The ISSG promotes and facilitates the exchange of invasive species information and knowledge across the
globe and ensures the linkage between knowledge, practice and policy so that decision making is
informed. The two core activity areas of the ISSG are policy and technical advice, and information
exchange through our online resources and tools and through networking.

Q 71.D
 Recent context: National Organ and Tissue Transplant Organisation data shows four out of five organ
recipients in the country are men highlighting gender disparity.
 National Organ and Tissue Transplant Organization (NOTTO) is a National level organization set
up under the Directorate General of Health Services, Ministry of Health and Family Welfare. It has
the following two divisions :
o National Human Organ and Tissue Removal and Storage Network
o National Biomaterial Centre
 Hence statement 1 is not correct.
 National Organ and Tissue Transplant Organization functions as an apex center for All India
activities of coordination and networking not just for procurement but the distribution of Organs
and Tissues and registry of Organs and Tissues Donation and Transplantation in the
country. Hence statement 2 is not correct.
o The following activities would be undertaken to facilitate Organ Transplantation most safely in the
shortest possible time and to collect data to develop and publish the National registry:
 Lay down policy guidelines and protocols for various functions.
 Network with similar regional and state-level organizations.
 All registry data from States and Regions would be compiled and published.
 Creating awareness, and promotion of organ donation and transplantation activities.
 Co-ordination from procurement of organs and tissues to transplantation when the organ is
allocated outside the region.
 Dissemination of information to all concerned organizations, hospitals, and individuals.
 Monitoring of transplantation activities in the Regions and States and maintaining data in this
regard.
 To assist in data management for organ transplant surveillance & organ transplant and Organ
Donor registry.
 Consultancy support on the legal and non-legal aspects of donation and transplantation.
 Coordinate and Organize trainings for various cadre of workers.
Q 72.B
 Recent context: Recently a 5 judge bench of the Supreme Court has started hearing the petitioner's case
challenging the constitutional validity of electoral bonds.
 Electoral bonds are interest-free bearer bonds or money instruments that can be purchased by companies
and individuals in India from authorized branches of the State Bank of India (SBI).
o These bonds are sold in multiples of Rs 1,000, Rs 10,000, Rs 1 lakh, Rs 10 lakh, and Rs 1 crore. They
can be purchased through a KYC-compliant account to make donations to a political party. The
political parties have to encash them within a stipulated time.
30 www.visionias.in ©Vision IAS
 The name and other information of the donor are not entered on the instrument and thus electoral bonds
are said to be anonymous. There is no cap on the number of electoral bonds that a person or
company can purchase.
 Political parties that secured at least 1% of the votes polled in the recent Lok Sabha or State Assembly
elections and are registered under the RPA can get a verified account from the Election Commission of
India (ECI). The bond amounts are deposited in this account within 15 days of their purchase.
o The political party has to encash the amount within those 15 days, and the amount received as a
donation gets deposited into the Prime Minister's Relief Fund. These bonds, however, are not
available for purchase all the time.
 In India, there are no donation limits on individuals. Moreover, the Finance Act, of 2017 also removed
any official contribution limits on companies. In other words, an individual or a company can donate
as much as they want to a political party. Hence statement 1 is correct and statement 2 is not
correct.
 Similarly, there is no legal expenditure limit on expenditure by political parties. A party can spend
as much as it wants for its national or state-level campaign as long as it does not spend that money
on the election of any specific candidate. Hence statement 3 is correct.
 However, parties are required to disclose donations of more than Rs 20,000, unless they are made through
electoral bonds.
o Parties are not required to disclose the sum or the source of any single donation that is below Rs
20,000.
o This is where the legal loophole steps in — parties generally break large donations from a single
donor into multiple small donations. This practice exempts them from any disclosure requirement.
Q 73.A
 India’s updated Nationally Determined Contribution (NDC) the United Nations Framework
Convention on Climate Change (UNFCCC).
o It would protect the interests of the country and safeguard its future development needs based on the
principles and provisions of the UNFCCC.
 India at the 26th session of the Conference of the Parties (COP26) to the UNFCCC held in
Glasgow, United Kingdom, expressed to intensify its climate action by presenting to the world five
nectar elements (Panchamrit) of India’s climate action.
 This update to India’s existing NDC translates the ‘Panchamrit’ announced at COP 26 into enhanced
climate targets.
 As per the updated NDC, India now stands committed to reduce Emissions Intensity of its GDP by
45 percent by 2030, from 2005 level. Hence statement 1 is not correct.
 India aims to achieve about 50 percent cumulative electric power installed capacity from non-fossil
fuel-based energy resources by 2030. Hence statement 2 is not correct.
 It targets net zero emissions by 2070 and it does not bind it to any sector specific mitigation
obligation or action. Hence statement 3 is correct.
 India’s goal is to reduce overall emission intensity and improve energy efficiency of its economy over
time and at the same time protecting the vulnerable sectors of economy and segments of our society.
Q 74.A
 Just Energy Transition Partnerships (JETP) aim to bridge the gap between developed and
developing nations in moving towards clean energy. Essentially, JETP is a financing mechanism. In
a Partnership, wealthier nations fund a coal-dependent developing nation to support the country’s
own path to phase out coal and transition towards clean energy while addressing the social
consequences. Hence, statement 1 is correct.
 JETP funding can go through grants, loans, or investments. As of March 2023, the donor pool includes the
International Partners Group (IPG) and the Glasgow Financial Alliance for Net Zero (GFANZ) Working
Group. The IPG consists of Japan, the USA, Canada, Denmark, France, Germany, Italy, Norway, the EU,
and the UK. The GFANZ Working Group comprises multilateral and national development banks and
finance agencies such as HSBC and Citi Bank.
 The first Just Energy Transition Partnership was with South Africa and was announced at COP 26
Glasgow in November 2021. The funders, five of the current IPG members, pledged 8.5 billion USD
in the first financing round. A year later, at COP 27 Sharm El Sheikh, South Africa published its
JETP Implementation Plan (JETP IP). This JETP is expected to prevent up to 1-1.5 gigatons of
emissions from the atmosphere over the next 20 years. Hence, statement 2 is not correct.
 The second Partnership was announced at the G20 Bali Summit in November 2022. Indonesia is set to
receive an initial 20 billion USD in public and private financing over the next three to five years. The
31 www.visionias.in ©Vision IAS
donors will assist via grants, concessional loans, market-rate loans, guarantees, private investments, and
technical assistance. In February 2023, Indonesia launched the Secretariat for the Just Energy
Transition Partnership.
 The next one was Vietnam. The Vietnam JETP was announced in December 2022 after a lengthy
negotiation process. This Partnership will assist Vietnam in finance, technology, and capacity building. It
will also support the country’s policy and regulation improvement to increase private investment in
renewable energy. Hence, statement 3 is not correct.

Q 75.C
 The Convention on the Conservation of Migratory Species of Wild Animals (Bonn Convention; CMS) is
an environmental treaty under the aegis of the United Nations Environment Programme. Hence,
statement 2 is correct.
 It provides a global platform for the conservation and sustainable use of migratory animals and their
habitats. CMS is the only global and UN-based intergovernmental organisation established
exclusively for the conservation and management of terrestrial, aquatic and avian migratory species
throughout their range. Hence, statement 1 is correct.
 Migratory species threatened with extinction are listed on Appendix I of the Convention. CMS Parties
strive towards strictly protecting these animals, conserving or restoring the places where they live,
mitigating obstacles to migration and controlling other factors that might endanger them. Besides
establishing obligations for each State joining the Convention, CMS promotes concerted action among the
Range States of many of these species.
 Migratory species that need or would significantly benefit from international co-operation are listed in
Appendix II of the Convention. For this reason, the Convention encourages the Range States to conclude
global or regional agreements.

Q 76.A
 Recent Context: Indian Army successfully test-fired rocket and turret guns of indigenous Light Combat
Helicopter (LCH) Prachand (means fierce).
o It was inducted into Indian Air Force in 2022.
o LCH Prachand is a multi-role combat helicopter, designed and developed by Hindustan
Aeronautics Ltd.
o Only attack helicopter in the world that can land and take off at an altitude of 5,000 metres.
o Capable of firing air-to-ground and air-to-air missiles.
o Fitted with 5.8-tonne twin-engine named Shakti engine, primarily designed for deployment in high-
altitude areas (like Siachen glacier).
o It has best stealth features, armored-shield systems, and dark-mode attack capability.
 Hence option (a) is the correct answer.

Q 77.B
 The ‘’Vajra Mushti Kalaga’’ is a form of wrestling different from conventional grappling and entails
two jettys taking a swipe at each other’s head with a knuckleduster.
o The fight is real and the jettys make all efforts to draw blood from the opponent’s head and a referee
intervenes on noticing the first drop.
 Whosoever draws the blood from the opponent’s head first is declared the winner. Though this form of
wrestling was popular during the period of the Vijayanagar rulers who reigned between the 14th
and the 17th centuries, it has gone extinct and takes place only during Dasara. It is not associated with
the Vajrayana form of Buddhism.
 Medieval travelers from the Portuguese noticed this form of wrestling during the Navaratri celebrations in
the Vijayanagar empire and have left detailed accounts of it. Historically this form of martial art was
popular and has a hoary past but has been on a decline and has gone out of vogue in modern times.
 Hence option (b) is the correct answer.

Q 78.C
 Ministry of Environment, Forest and Climate Change (MoEFCC) launched National Clean Air
Programme (NCAP) in January, 2019 with an aim to improve air quality in 131 cities (non-
attainment cities and Million Plus Cities) in 24 States/UTs by engaging all stakeholders. The
programme envisages to achieve reductions up to 40% or achievement of National Ambient Air Quality
Standards for Particulate Matter10 (PM 10) concentrations by 2025-26.

32 www.visionias.in ©Vision IAS


 The NCAP targets to achieve 20% to 30% reduction in concentrations of PM10(particulate matter of
diameter between 10 and 2.5 micrometer) and PM2.5(particulate matter of diameter 2.5 micrometer or
less) by the year 2024, keeping 2017 as the base year for comparison of concentration.
 Non-attainment cities (NAC)- Cities are declared nonattainment if over a 5-year period they
consistently do not meet the National Ambient Air Quality Standards (NAAQS) for PM 10
(Particulate matter that is 10 microns or less in diameter) or N02 (Nitrogen Dioxide).
 Central Pollution Control Board (CPCB) identified 94 non-attainment cities (NAC) while circulating the
draft NCAP in 2018 that was revised to 102 NAC's in January 2019. Two more were added to the list in
2020, taking the total to 124. The list of cities under NCAP focus now stands at 131 cities in 2021 with
a million-plus population.
 Hence, option (c) is the correct answer.

Q 79.D
 Smog is air pollution that reduces visibility. The term "smog" was first used in the early 1900s to describe
a mix of smoke and fog. The smoke usually came from burning coal. Smog was common in industrial
areas and remains a familiar sight in some cities today.
 Today, most of the smog we see is photochemical smog. Photochemical smog is produced when sunlight
reacts with nitrogen oxides and at least one volatile organic compound (VOC) in the atmosphere. Nitrogen
oxides come from car exhaust, coal power plants, and factory emissions. VOCs are released from
gasoline, paints, and many cleaning solvents. When sunlight hits these chemicals, they form airborne
particles and ground-level ozone—or smog.
 Smog is still a problem in many places. Everyone can do their part to reduce smog by changing a few
behaviours, such as:
o Drive less. Walk, bike, carpool, and use public transportation whenever possible.
o Take care of cars. Getting regular tune-ups, changing oil when scheduled, and inflating tires to
the proper level can improve gas mileage and thus reduce emissions.
o Fuel up during the cooler hours of the day—night or early morning. This prevents gas fumes
from heating up and producing ozone.
o Avoid products that release high levels of VOCs. For example, use low-VOC paints.
o Avoid gas-powered yard equipment, like lawnmowers. Use electric appliances instead.
 Hence, option (d) is the correct answer.

Q 80.A
 The Mouvement Du 23 Mars (M23) is an armed group operating in the Democratic Republic of the
Congo (DRC). The prominent rebel groups in the DRC, besides M23 include the Allied Democratic
Forces (ADF) and the Cooperative for Development of the Congo (CODECO).
 Recently, the UN International Organization of Migration (IOM) reported that the number of people who
have been internally displaced in the Democratic Republic of Congo (DRC) has risen to 6.9 million. In
the eastern province of North Kivu, nearly a million people have been displaced due to the ongoing
conflict with the rebel group, Mouvement du 23 Mars (M23).
 The conflict in the DRC dates back to the 1990s when it went through two civil wars in 1996 and 1998.
The conflict erupted in the wake of the Rwandan genocide in 1994 where ethnic Hutu extremists killed
nearly one million minority ethnic Tutsis and non-extremist Hutus.

33 www.visionias.in ©Vision IAS


 The armed groups have been supported by the governments of Rwanda, Uganda, and Burundi at
various points, acting as proxies for each country’s interests in the region.
 Hence option (a) is the correct answer.

Q 81.C
 These Bio-Medical Waste (Management and Handling) Rules apply to all persons who generate, collect,
receive, store, transport, treat, dispose of, or handle bio-medical waste in any form including hospitals,
nursing homes, clinics, dispensaries, veterinary institutions, animal houses, pathological laboratories,
blood banks, and Ayush. These rules shall apply to all persons who generate, collect, receive, store,
transport, treat, hospitals, clinical establishments, research or educational institutions, health camps,
medical or surgical camps, vaccination camps, blood donation camps, first aid rooms of schools, forensic
laboratories and research labs.
 Only Pair 1 is not correctly matched.

 Hence, option (c) is the correct answer.


34 www.visionias.in ©Vision IAS
Q 82.B
 Recent context: The National Aeronautics and Space Administration (NASA) is set to launch the
Atmospheric Waves Experiment (AWE) to study one of the important drivers of Space weather – the
Earth’s weather.
 AWE is a first-of-its-kind NASA experimental attempt aimed at studying the interactions between
terrestrial and Space weather. Hence statement 1 is correct but statement 2 is not correct.
 It is planned under NASA’s Heliophysics Explorers Program, the $42 million mission will study the links
between how waves in the lower layers of the atmosphere impact the upper atmosphere, and thus, Space
weather.
 AWE will be launched and mounted on the exterior of the Earth-orbiting International Space
Station (ISS). From the vantage point, it will look down at the Earth and record the colorful light bands,
commonly known as airglow. Hence statement 3 is correct.
o AWE will measure the airglow at mesopause (about 85 to 87 km above the Earth’s surface), where
the atmospheric temperatures dip to minus 100 degrees Celsius.
o At this altitude, it is possible to capture the faint airglow in the infrared bandwidth, which appears the
brightest enabling easy detection.
 Onboard AWE is an Advanced Mesospheric Temperature Mapper (ATMT), an instrument that will scan
or map the mesopause

Q 83.A
 The crackdown on fossil fuel emissions continues, with the recent implementation of CAFE II norms
(April 2022).
 In short, more stringent emission regulations are designed to get car manufacturers to cut down on
their overall CO2 levels, chiefly through software and hardware-related upgrades and by shifting to
electrified or hybridized powertrains. Hence, statement 1 is not correct.
 CAFE stands for Corporate Average Fuel Efficiency/Economy, so it is not the emission and economy
of the individual models that are considered, but the weighted average of emissions and fuel economy
for the entire fleet sold in the country. Hence, statement 2 is correct.
 This means, that the manufacturer can very well sell heavy, highly polluting and fuel-efficient cars, but
then they will have to make that up by selling huge volumes of much more efficient and light cars.
 The Corporate Average Fuel Economy (CAFE) norms are primarily designed to increase fuel efficiency,
which, in turn, will lower a company’s overall CO2 footprint.
 At present, the existing limit for average CO2 emissions happens to be 130g/km. Hence, statement 4
is not correct.
 With CAFE II in effect, the target has been lowered to 113g/km. The key difference between CAFE
norms and BS6 II is that the former focuses on reducing all manner of harmful by-products from a car’s
exhaust (sulphur, nitrogen oxide, etc), while the latter focuses exclusively on CO2 emissions.
 However, both norms force manufacturers to lower fuel consumed by their vehicles, and simultaneously,
move towards electric mobility.
 These standards were released by the Union Ministry of Power (MoP) for the first time in 2017,
under the Energy Conservation Act, of 2001. They apply to petrol, diesel, LPG and CNG vehicles.
Hence, statement 3 is not correct.
Q 84.A
 Air (Prevention and Control of pollution) Act,1981
 The act Provides for the prevention, control and abatement of air pollution. It makes provisions, Interalia,
for Central and State Boards, power to declare pollution control areas, restrictions on certain industrial
units, the authority of the Boards to limit the emission of air pollutants, power of entry, inspection, taking
samples and analysis, penalties, offences by companies and Government and cognizance of offences etc.
 The Act specifically empowers the State Government to designate air pollution areas and to prescribe the
type of fuel to be used in these designated areas.
 Power to declare air pollution areas: Section 19
o The Act states that the State Government, after consulting the State Board, may declare an area within
the State as an air pollution area.
o The State Government may also order for the extension or reduction of an air pollution area or may
even merge one or more areas to make a new pollution area or any part or parts thereof.
o The State Government after consulting the State Board, may also by notification in the Official
Gazette, prohibit the use of any fuel or appliance that may cause or is likely to cause air
pollution. Hence, statement 1 is not correct.

35 www.visionias.in ©Vision IAS


o The State Government may also prohibit the burning of any material (which is not a fuel) if it causes
or is likely to cause air pollution. This is also done after consultations with the respective State Board.
 Powers and Functions
o Central Board- Section 16
 The main function of the Central Board shall be to improve the quality of air and to prevent,
control or abate air pollution in the country.
 It may:
 Advise the Central Government on any matter concerning the improvement of the quality of
air.
 Plan and cause to be executed a nation-wide programme for the prevention, control or
abatement of air pollution
 Co-ordinate the activity of the State and resolve disputes among them
 Provide technical assistance and guidance to the state boards, carry out and sponsor
investigations and research relating to problems of air pollution
 Organize through mass media a comprehensive programme regarding the prevention, control
or abatement of air pollution
 The Central Board may establish or recognize a laboratory to enable the Central Board
to perform its functions under this section efficiently. Hence, statement 3 is correct.
o State Board - Section 17
 To plan a comprehensive programme for the prevention, control or abatement of air pollution;
 Advise the State Government on any matter concerning the prevention of air pollution;
 To inspect, at all reasonable times, any control equipment, industrial plant or
manufacturing process, and to give by order such directions to such person as it may be
necessary to take steps for the prevention of air pollution. Hence, statement 2 is not correct.
 It shall collect and disseminate information regarding air pollution. It shall organise training and
mass awareness programmes regarding air pollution control, prevention and abatement.
 It shall lay down standards for the emission of air pollutants into the atmosphere from
automobiles or industries, or any other pollutant from any source. However, a ship or aircraft
cannot come into the ambit of a source.
 The State Boards shall also advise the State Government regarding the suitability of any location
which is to be used for setting up any industry, keeping in mind the air quality which would be
impacted if that industry is set up.
 The Boards shall also set up labs in their States, to enable the State Board to perform its functions
effectively.
 Hence, option (a) is the correct answer.

Q 85.C
Global Methane Pledge
 The Global Methane Pledge was launched at COP (Conference of Parties) 26 in November 2021 in
Glasgow, Scotland to catalyse action to reduce methane emissions and is led by the United States
and the European Union. Hence statement 2 is correct.
 It has 111 country participants who together are responsible for 45% of global human-caused methane
emissions.India, which is not a part of the Global Methane Pledge, is among the top five methane emitters
globally. Most emissions can be traced back to agriculture.
 By joining the Pledge, countries commit to work together in order to collectively reduce methane
emissions by at least 30% below 2020 levels by 2030. Hence statement 1 is correct.
 India did not sign the pledge because of its concerns over the impact on trade, on the country's vast farm
sector, and the role of livestock in the rural economy. Hence statement 3 is correct.

Q 86.C
 Environmental Impact Assessment (EIA):
o The UNEP defines Environmental Impact Assessment (EIA) as a tool used to identify the
environmental, social and economic impacts of a project prior to decision-making. It aims to
predict environmental impacts at an early stage in project planning and design, find ways and means
to reduce adverse impacts, shape projects to suit the local environment and present the predictions and
options to decision-makers.
o EIA comes under the Notification on Environmental Impact Assessment (EIA) of developmental
projects 1994 under the provisions of the Environment (Protection) Act, 1986.
36 www.visionias.in ©Vision IAS
o “Public Consultation” refers to the process by which the concerns of locally affected persons and
others who have a plausible stake in the environmental impact of the project or activity are
ascertained with a view to taking into account all the material concerns in the project or activity
design as appropriate.
 2022 Amendments in EIA rules
o Thermal power plants up to 15 MW based on biomass or non-hazardous municipal solid
waste using auxiliary fuel such as coal, lignite or petroleum products up to 15 per cent have also been
exempted — as long as the fuel mix is eco-friendly.
o Fish handling ports and harbours-Taking into account issues of livelihood security of fishermen
involved at fish handling ports and harbours, and the less pollution potential of these ports and
harbours compared to others, increasing the threshold of ports which exclusively deals in fish
handling, and cater to small fishermen, will be exempted from environmental clearance.
o Toll plazas that need more width for the installation of toll collection booths to cater to a large
number of vehicles are exempted.
o Airport-Expansion activities in existing airports related to terminal building expansion without an
increase in the airport’s existing area, rather than expansion of runways also exempted.
o Strategic Projects-For projects of strategic importance-“Highway projects related to defence and
strategic importance in border States are sensitive in nature and that needs to be executed on priority
keeping in view strategic, defence and security considerations.
o Exemption of such projects from the requirement of Environmental Clearance in border areas, subject
to specified Standard Operating Procedure along with standard environmental safeguards for such
projects for self-compliance by the agency executing such projects.Any project which is 100 km from
the Line of Control, among other locations, will be exempted from an environmental clearance before
construction.
 Hence option (c) is the correct answer.

Q 87.B
 Global Energy Transition Index is released by World Economic Forum (WEF).In the latest published
index, India has moved up two places to rank 76th out of 115 economies. Hence option (b) is the correct
answer.
 It ranks economies on how well they are able to balance energy security and access with environmental
sustainability and affordability. The WEF index considers both the current state of the countries’ energy
system and their structural readiness to adapt to future energy needs.
 The ‘transition readiness’ component of the index has taken into account six individual indicators:
o capital and investment
o regulation and political commitment
o institutions and governance
o institutions and innovative business environment
o human capital and consumer participation
o energy system structure
 India’s Performance- India is amongst the countries with high pollution levels and has a relatively high
CO2 intensity in its energy system. India has made significant strides to improve energy access in recent
years, and currently scores well in the area of regulation and political commitment towards energy
transition. However, India is the only amongst the five economies to improve its rank since last year.
 Global Rankings: Sweden remains on the top on this annual list and is followed by Switzerland and
Norway in the top three.

Q 88.D
 Composition, Generation and Utilization of Fly Ash in India
 Fly ash is a by-product of coal-based power plants.
 It is a fine powder with substantial amounts of oxides of silica, aluminium and calcium. It also
contains traces of Arsenic, Boron, Chromium, lead etc. which leads to air and water pollution if
disposed on land. Hence option (d) is the correct answer.
 With low grade of Indian coal, its ash content is as high as 30-45% in comparison to imported coal with
10- 15%.
 With nearly 55% of our total power production through coal and lignite based Thermal Power Plants
(TPP), the fly ash generation in India is very high.

37 www.visionias.in ©Vision IAS


Q 89.C
 Renewable diesel can be produced by several different technology pathways. Currently, commercial
production facilities are using the hydrotreating pathway and fats, oils, and greases are the most common
feedstocks. It is made from fats and oils, such as soybean oil or canola oil, and is processed to be
chemically the same as petroleum diesel. Hence, pair 2 is correctly matched.
 Renewable gasoline (also called green or drop-in gasoline) is a fuel produced from biomass sources
through a variety of biological, thermal, and chemical processes. The fuel is chemically identical to
petroleum gasoline. It can be produced from various biomass sources. These include lipids (such as
vegetable oils, animal fats, greases, and algae) and cellulosic material (such as crop residues, woody
biomass, and dedicated energy crops). Hence, pair 1 is correctly matched.
 Sustainable aviation fuel (SAF), made from non-petroleum feedstocks, is an alternative fuel that reduces
emissions from air transportation. SAF can be blended at different levels with limits of 10% to 50%,
depending on the feedstock and how the fuel is produced. SAF can be produced from non-petroleum-
based renewable feedstocks including, but not limited to, the food and yard waste portion of
municipal solid waste, woody biomass, fats/greases/oils, and other feedstocks. Hence, pair 3 is
correctly matched.

Q 90.C
 The Rotterdam Convention (formally, the Rotterdam Convention on the Prior Informed Consent
Procedure for Certain Hazardous Chemicals and Pesticides in International Trade) is a multilateral treaty
to promote shared responsibilities in relation to the importation of hazardous chemicals. Hence,
statement 1 is correct.
 The convention promotes an open exchange of information and calls on exporters of hazardous chemicals
to use proper labelling, include directions on safe handling, and inform purchasers of any known
restrictions or bans.
 Signatory nations can decide whether to allow or ban the importation of chemicals listed in the treaty, and
exporting countries are obliged to make sure that producers within their jurisdiction comply.
 The movements of hazardous chemicals listed in Annex III are subject to the Prior Informed
Consent procedure (“PIC procedure”). Hence, statement 2 is correct.
 Exports are only allowed if the State of import has consented to the future import of the specific
chemical through an Import Response. Hence, statement 3 is correct.
 To achieve its objectives the Convention includes two key provisions, namely the Prior Informed Consent
(PIC) Procedure and Information Exchange.
o The Prior Informed Consent (PIC) procedure – The PIC procedure is a mechanism for formally
obtaining and disseminating the decisions of importing Parties as to whether they wish to receive
future shipments of those chemicals listed in Annex III of the Convention and for ensuring
compliance with these decisions by exporting Parties.
o Information Exchange - The Convention facilitates information exchange among Parties for a very
broad range of potentially hazardous chemicals. The Convention requires each Party to notify the
Secretariat when taking a domestic regulatory action to ban or severely restrict a chemical.
 The Convention does not apply to chemicals imported for the purpose of research and analysis.
Hence, statement 4 is not correct.
 What is the maximum quantity of a chemical that can be imported for such purposes?
o The Convention does not specify an amount that is considered for the purpose of research or analysis.
Some Parties in implementing the Convention have set a level of 10 kilograms whereas others have
set lower amounts.
o Whatever amount Parties choose to apply, it is important to recognize that these should be small
amounts compared to commercially traded quantities and must not be likely to affect human health
and the environment.

Q 91.D
 The Climate Neutral Now Initiative is one of several initiatives launched by the UNFCCC secretariat to
increase climate action by engaging non-party stakeholders (sub-national governments, companies,
organizations, and individuals). It was launched in 2015 based on a mandate to promote the voluntary
use of carbon market mechanisms recognized under the Convention.
 The Climate Neutral Now Initiative encourages and supports organizations to act now in order to achieve
a climate-neutral world by 2050, as enshrined in the Paris Agreement. It is a tool to promote additional
voluntary action on climate and to provide recognition for it.
38 www.visionias.in ©Vision IAS
 The initiative is not a certification scheme for its participants. An organization can become a participant
by signing the Climate Neutral Now Pledge, following the three steps (Measure, Reduce, Contribute) and
reporting on its actions and achievements annually.
 Hence, option (d) is the correct answer.

Q 92.B
 The chemical oxygen demand (COD) determines the amount of oxygen required for the chemical
oxidation of organic matter using a strong chemical oxidant, such as potassium dichromate under
reflux conditions. Hence, statement 1 is correct.
 This test is widely used to determine: a) the Degree of pollution in water bodies and their self-purification
capacity, b) the Efficiency of treatment plants, c) Pollution loads, and d) Provides a rough idea of
Biochemical oxygen demand (BOD) which can be used to determine sample volume for BOD
estimation. Hence, statement 3 is correct.
 The limitation of the test lies in its inability to differentiate between the biologically oxidizable and
biologically inert material and to find out the system rate constant of aerobic biological
stabilization. Hence, statement 2 is not correct.
 Most of the organic matter is destroyed when boiled with a mixture of potassium dichromate and
sulphuric acid producing carbon dioxide and water.
 A sample is refluxed with a known amount of potassium dichromate in sulphuric acid medium and the
excess of dichromate is titrated against ferrous ammonium sulphate.
 The amount of dichromate consumed is proportional to the oxygen required to oxidize the oxidizable
organic matter.

Q 93.A
 Recent context: Countries reinforced their commitment to combat Transnational Organized Crime at a
Ministerial Conference in Palermo, Sicily (Italy).
 The Palermo Convention, officially known as the "United Nations Convention against
Transnational Organized Crime," is an international treaty adopted by the United Nations in 2000.
It is supplemented by three protocols that address specific issues related to transnational organized
crime. These protocols are often referred to as the "Palermo Protocols."
o Protocol to Prevent, Suppress, and Punish Trafficking in Persons, Especially Women and
Children: This protocol, also known as the "Trafficking in Persons Protocol," addresses the
prevention and suppression of human trafficking, particularly the trafficking of women and children.
It outlines measures to be taken by countries to combat human trafficking and protect the rights of
victims.
o Protocol against the Smuggling of Migrants by Land, Sea, and Air: This protocol, also known as
the "Migrant Smuggling Protocol," focuses on the prevention and suppression of the illegal smuggling
of migrants, which is often associated with transnational organized crime. It aims to prevent and
combat migrant smuggling and protect the rights of smuggled migrants.
o Protocol against the Illicit Manufacturing of and Trafficking in Firearms, Their Parts and
Components and Ammunition: This protocol, known as the "Firearms Protocol," addresses the
illicit manufacturing and trafficking of firearms, their parts, and ammunition. It aims to prevent the
proliferation of illicit firearms and combat-related criminal activities.
 The Palermo Convention and its protocols collectively provide a framework for international cooperation
in combating transnational organized crime. They emphasize the importance of legal measures, law
enforcement cooperation, and the protection of the rights of victims in addressing various forms of
organized criminal activities, including human trafficking, migrant smuggling, and illicit firearms
trafficking.
 Hence option (a) is the correct answer.

Q 94.A
 Recent context : The Election Commission of India has designed in-house software for complete
Candidate and election management through ‘ENCORE’ which stands for Enabling
Communications on Real-time Environment.
 This provides a seamless facility for Returning Officers to process candidate nomination, affidavit,
Voter turnout, counting, results and data management.
 The ENCORE counting application is an end-to-end application for returning officers to digitize the votes
polled, tabulate the round-wise data and then take out various statutory reports of counting.
39 www.visionias.in ©Vision IAS
 Another application called ENCORE Scrutiny Application allows Returning Officers to do scrutiny of the
nominations filed by the candidates on-line.
o After verification of the nomination the status is marked as Accepted, Rejected or Withdrawn helping
the Returning Officer to prepare the final list of contesting candidates and assign the symbols.
 The National Technical Research Organisation is a technical intelligence agency of India. It was set up in
2004. The agency reports to the National Security Advisor and to the Prime Minister's Office.
 Hence statement 1 is correct but statement 2 is not correct.

Q 95.B
 The Ocean thermal energy conversion (OTEC) technology uses the temperature difference between
the cold water in the deep sea (5°C) and the warm surface seawater (25°C) to generate clean,
renewable electricity. Warm surface water is pumped through an evaporator containing a working
fluid. Hence, statement 1 is not correct.
 The vaporized fluid drives a turbine/generator. The vaporized fluid is condensed back to liquid in a
condenser cooled with cold ocean water pumped from deeper in the ocean. OTEC systems using seawater
as the working fluid can use the condensed water to produce desalinated water.
 The National Institute of Ocean Technology, an autonomous institute under the Union Ministry of
Earth Sciences (MoES) is establishing an Ocean Thermal Energy Conversion plant with a capacity
of 65 kilowatts (kW) in Kavaratti, the capital of Lakshadweep. It will be the first ocean thermal
energy plant in India. Hence, statements 2 and 3 are correct.

Q 96.A
 Global Tiger Forum
o Global Tiger Forum is an international intergovernmental body exclusively set up for
the conservation of tigers in the wild in the tiger range countries.
o Out of the 13 tiger range countries, seven are currently members of GTF: Bangladesh, Bhutan,
Cambodia, India, Myanmar, Nepal and Vietnam besides non-tiger range country U.K. The
secretariat is based in New Delhi, India.
o The other six tiger range countries but not members of GTF are China, Russia, Malaysia,
Indonesia and Thailand. Hence option (a) is the correct answer.
o GTF’s goal is to highlight the rationale for tiger preservation and provide leadership and a common
approach throughout the world in order to safeguard the survival of the tiger, its prey, and its habitat.
 Global Tiger Initiative (GTI): All 13 Tiger range countries are members of GTI
o The Global Tiger Initiative (GTI) is a worldwide alliance of governments, international organizations,
civil society, the conservation and scientific community, and the private sector dedicated to
collaborating on a common agenda to save wild tigers from extinction.
 Origins
o In 2008, the GTI was founded by founding partners the World Bank, Global Environment Facility,
Smithsonian Institution, Save the Tiger Fund, and the International Tiger Coalition (representing more
than 40 non-government organizations).
 Goals
o Double the wild tiger population by 2022.
o Conserve tiger habitats and corridors.
o Reduce human-tiger conflict.
o Enhance public support for tiger conservation.
 Achievements
The GTI has made significant progress in its efforts to conserve tigers, including:
o Increasing the global tiger population from 3,159 in 2010 to 3,949 in 2022.
o Expanding tiger habitat by over 10% since 2010.
o Reducing poaching by over 50% since 2010.
o Raising awareness of tiger conservation through public awareness campaigns and education
programs.
 Global Tiger Recovery Program (GTRP) was launched in 2010 under the Global Tiger Initiative (GTI)
by the World Bank to save wild tigers. This initiative set up the ambitious target of reversing the rapid
decline of wild tigers across their range and doubling their population numbers by 2022.

40 www.visionias.in ©Vision IAS


Q 97.B
 Carbon Credit Trading Scheme 2023 :
o The Indian Parliament passed the Energy Conservation (Amendment) Bill 2022, which modifies the
2001 Energy Conservation Act. The Ministry of Power notified the Carbon Credit Trading Scheme
2023 and will soon notify entities obligated to comply with GHG emission regulations. To ensure that
the identified entities’ percentage of total energy requirements come from non-fossil fuel sources, the
government will release the modalities based on the Bureau of Energy Efficiency (BEE)
recommendations.
o MoEFCC will notify the emission intensity target for obligated entities upon the recommendation of
the Ministry of Power. Emission intensity is the total amount of greenhouse gas emitted for every unit
of GDP.
Key Features of the CCTS 2023
 Cap-and-trade system:
The CCTS employs a cap-and-trade system, where a cap is set on the total amount of greenhouse gas
(GHG) emissions allowed from a group of entities. Entities covered under the scheme have emission
intensity reduction targets. Entities that achieve or exceed their targets earn carbon credit certificates,
while those that fall short need to purchase credits from the market.
 Trading mechanism:
Obligated entities will earn a carbon credit certificate if they surpass the target assigned to them. T he
certificate will be issued by BEE. Then the Carbon credit certificates are traded on designated power
exchanges. Hence statement 1 is not correct.
 Governance and oversight:
o The National Steering Committee for the Indian Carbon Market (NSCICM) oversees the overall
functioning of the carbon market.
o The Bureau of Energy Efficiency (BEE) will act as the administrator for the ICM, responsible for
developing GHG emissions trajectory and targets for obligated entities.
o The Grid Controller of India Limited will be the designated agency for maintaining the ICM
Registry and overseeing transactions among obligated entities.
o The Central Electricity Regulatory Commission (CERC) will act as the regulator for carbon credit
trading. They will register power exchanges for trading carbon credit certificates, protect buyer and
seller interests, and prevent fraud or mistrust. Hence statement 2 is correct.
Q 98.A
 Cocoyoc declaration was signed in October 1974 by an international group of social scientists, natural
scientists and economists. It was based on the theme of ‘Sustainable development’ and became the first
declaration to use the term. It also discussed themes of patterns of resource use, environment and
development strategies.
 Hence option (a) is the correct answer.

Q 99.C
 Basel Convention
o The Basel Convention is an international treaty that aims to control the transboundary movement of
hazardous wastes and their disposal. It was adopted in 1989 and entered into force in 1992. As of
today, there are 189 parties to the Basel Convention. Hence pair 1 is correctly matched.
o The Basel Convention's main objectives are to:
 Reduce the transboundary movement of hazardous wastes
 Ensure the environmentally sound management of hazardous wastes
 Protect human health and the environment from the adverse effects of hazardous wastes
 Hong Kong International Convention
o The Hong Kong Convention is an international treaty that aims to ensure the safe and
environmentally sound recycling of ships. It was adopted in 2009 and entered into force in 2015. As
of today, there are 15 parties to the Hong Kong Convention. Hence pair 2 is correctly matched.
o The Hong Kong Convention's main objectives are to:
 Prevent the uncontrolled disposal of ships
 Ensure that ships are recycled in a way that protects human health and the environment
 Promote the use of environmentally sound recycling technology
 Stockholm Convention on Persistent Organic Pollutants is an international environmental treaty,
signed on 22 May 2001 in Stockholm and effective from 17 May 2004, that aims to eliminate or restrict
the production and use of persistent organic pollutants (POPs). Hence pair 3 is correctly matched.
41 www.visionias.in ©Vision IAS
Q 100.D
 Climate-smart agriculture (CSA) is an approach that helps to guide actions needed to transform and
reorient agricultural systems to effectively support development and ensure food security in a changing
climate.
 CSA aims to tackle three main objectives:
o Adapting and building resilience of crops and livestock to climate change. Hence, statement 1 is
correct.
o Sustainably increasing agricultural productivity and incomes. Hence, statement 2 is correct.
o Reducing and/or removing greenhouse gas emissions wherever possible. Hence, help India is meeting
its Intended Nationally Determined Commitments under Paris Climate Agreement. Hence, statement
3 is correct.

Copyright © by Vision IAS


All rights are reserved. No part of this document may be reproduced, stored in a retrieval system or
transmitted in any form or by any means, electronic, mechanical, photocopying, recording or otherwise,
without prior permission of Vision IAS.

42 www.visionias.in ©Vision IAS


VISIONIAS
www.visionias.in

ANSWERS & EXPLANATIONS


GENERAL STUDIES (P) TEST – 4144 (2024)

Q 1.B
 Union Government of India approved the National Green Hydrogen Mission with an outlay of ₹ 19,744
crore from FY 2023-24 to FY 2029-30. The overarching objective of the Mission is to incentivize the
commercial production of green hydrogen and make India a net exporter of the fuel. The Mission
will facilitate demand creation, production, utilization and export of Green Hydrogen. Hence,
statement 1 is correct.
 The Mission will have wide-ranging benefits- the creation of export opportunities for Green Hydrogen
and its derivatives; Decarbonisation of industrial, mobility and energy sectors; reduction in dependence on
imported fossil fuels and feedstock; development of indigenous manufacturing capabilities; creation of
employment opportunities; and development of cutting-edge technologies. It aims to achieve the above
objectives, the Mission will build capabilities to produce at least 5 Million Metric tonnes (MMT) of
Green Hydrogen per annum by 2030, with the potential to reach 10 MMT per annum with the
growth of export markets. The Mission will support the replacement of fossil fuels and fossil fuel-based
feedstocks with renewable fuels and feedstocks based on Green Hydrogen. Hence, statement 2 is
correct.
o Achievement of Mission targets is expected to reduce a cumulative ₹ 1 lakh crore worth of fossil
fuel imports by 2030.
 The Ministry of New and Renewable Energy (MNRE) will be responsible for the overall
coordination and implementation of the Mission. The Mission Secretariat, headquartered in
MNRE, will formulate schemes and programs for financial incentives to support the production,
utilization and export of Green Hydrogen and its derivatives. Hence, statement 3 is not correct.

Q 2.C
 A carbon credit (also known as carbon offset) is a credit for greenhouse emissions reduced or removed
from the atmosphere by an emission reduction project, which can be used by governments, industry, or
private individuals to compensate for the emissions they generate elsewhere.
 Those that cannot easily reduce emissions can still operate, at a higher financial cost.
 Carbon credits are based on the "cap-and-trade" model that was used to reduce sulfur pollution in the
1990s. Hence, statement 2 is correct.
 One carbon credit is equal to one metric ton of carbon dioxide, or in some markets, carbon dioxide
equivalent gases (CO2-eq), and are bought and sold through international brokers, online retailers,
and trading platforms. Hence, statement 1 is correct.
 Offsetting one metric ton of carbon means that there will be one less Mt of carbon dioxide in the
atmosphere than there would otherwise have been. The Kyoto Protocol provides for three mechanisms
that enable countries, or operators in developed countries, to acquire greenhouse gas reduction
credits:
o Under Joint Implementation (JI) a developed country with relatively high costs of domestic
greenhouse reduction would set up a project in another developed country.
o Under the Clean Development Mechanism (CDM) a developed country can “sponsor” a
greenhouse gas reduction project in a developing country where the cost of greenhouse gas reduction
project activities is usually much lower, but the atmospheric effect is globally equivalent. The
developed country would be given credits for meeting its emission reduction targets, while the
developing country would receive the capital investment and clean technology or beneficial change in
land use.
o Under International Emissions Trading (IET) countries can trade in the international carbon credit
market to cover their shortfall in Assigned Amount Units (AAUs). Countries with surplus units can
sell them to countries that are exceeding their emission targets under Annex B of the Kyoto Protocol.
1 www.visionias.in ©Vision IAS
 Cap and trade is an approach that harnesses market forces to reduce emissions cost-effectively. Like
other market-based strategies, it differs from “command-and-control” approaches where the government
sets performance standards or dictates technology choices for individual facilities.
o Cap and trade allows the market to determine a price on carbon, and that price drives investment
decisions and spurs market innovation.
o Cap and trade differs from a tax in that it provides a high level of certainty about future emissions, but
not about the price of those emissions (carbon taxes do the inverse).

Q 3.D
 Indoor Air Pollution: The air within homes and other buildings can sometimes be more polluted than the
outdoor air even in the largest and most industrialised cities. Indoor air quality is an important concern for
the health and comfort of the occupants.
 Some of the sources of indoor air pollution are:
o Radon: Radon is an invisible, radioactive atomic gas that results from the radioactive decay of
radium, which may be found in rock formations beneath buildings or in certain building materials
themselves. Radon is the second most frequent cause of lung cancer, after cigarette smoking.
o Second-hand smoke: It is tobacco smoke which affects other people other than the 'active' smoker. It
includes both a gaseous and a particulate phase, with particular hazards arising from levels of carbon
monoxide and very small particulates.
o Biological chemicals: They can arise from a host of means, like moisture induced growth of
mould colonies and natural substances released into the air such as animal dander and plant
pollen. They are allergens and aggravate asthama.
o Ozone: Ozone is produced by ultraviolet light from the Sun hitting the Earth's atmosphere, lightning,
certain high-voltage electric devices and as a by-product of other types of pollution.
o Air Freshener: Many air fresheners employ carcinogens, volatile organic compounds and
known toxins such as phthalate esters in their formulas. Most of the products that have been
studied contain chemicals that can aggravate asthma and affect reproductive development.
 Hence, option (d) is the correct answer.

Q 4.D
 The Water (Prevention and Control of Pollution) Act was enacted in 1974 to provide for the
prevention and control of water pollution and for the maintaining or restoring of wholesomeness of
water in the country. The Act was amended in 1988.
 The Water (Prevention and Control of Pollution) Cess Act was enacted in 1977, to provide for the
levy and collection of a cess on water consumed by persons operating and carrying on certain types
of industrial activities. Hence, statement 1 is not correct.
 This cess is collected with a view to augment the resources of the Central Board and the State Boards for
the prevention and control of water pollution constituted under the Water (Prevention and Control of
Pollution) Act, 1974.
 The Act was last amended in 2003.
 On October 9, 2021, India’s Ministry of Environment, Forests and Climate Change published
“Environment (Protection) 115 Amendment Rules, 2021” to add regulation on the use of water
purification systems (WPS). Hence, statement 2 is not correct.
 Users of domestic water purification systems (DWPS) and other water purification systems (ODPWS)
have to comply with the guidelines issued by the Central Pollution Control Board (CPCB) within six
months of the promulgation of this regulation.

Q 5.C
 The Global Environment Facility:
The Global Environment Facility has a unique governing structure organized around an Assembly, the
Council, the Secretariat, 18 agencies, a Scientific and Technical Advisory Panel, and the Evaluation
Office.
 Financial mechanism provision by GEF
The GEF provides funding to assist developing countries in meeting the objectives of international
environmental conventions. The GEF serves as a "financial mechanism" to five conventions, which are
the Convention on Biological Diversity (CBD), the United Nations Framework Convention on
Climate Change (UNFCCC), the Stockholm Convention on Persistent Organic Pollutants (POPs),
the UN Convention to Combat Desertification (UNCCD), and Minamata Convention on Mercury.

2 www.visionias.in ©Vision IAS


 Five Focal Areas
It supports developing countries’ work to address the world’s most pressing environmental issues. GEF
organizes its work around five focal areas, They are as follows
o Biodiversity loss,
o Chemicals and waste
o Climate change
o International waters
o Land degradation
o and take an integrated approach to support more sustainable food systems, forest management, and
cities. Hence answer (c) is the correct answer.

Q 6.A
Status of India’s Nuclear Energy
 Nuclear energy is the fifth-largest source of electricity for India which contributes about 3% of the
total electricity generation in the country. Hence statement 1 is not correct.
 India has over 22 nuclear reactors in 7 power plants across the country which produces 6780 MW
of nuclear power. Hence statement 2 is not correct.
 In addition, one reactor,Kakrapar Atomic Power Project (KAPP-3) has also been connected to the grid in
January- 2021.
 18 reactors are Pressurised Heavy Water Reactors (PHWRs) and 4 are Light Water Reactors
(LWRs). Hence statement 3 correct.
 The existing nuclear power capacity of 6780 MW is going to be increased to 22480 MW by the year 2031
on progressive completion of projects under construction and accorded sanction. More nuclear power
plants are also planned in future.

Q 7.B
 The reduction of anthropogenic carbon dioxide (CO2) in the atmosphere is crucial for mitigating climate
change. CO2 capture and storage (CCS) is considered as one of the most promising options for carbon
reduction. The main means is the injection of CO2 into structural reservoirs in deep, permeable geologic
formations.
 Trapping mechanisms for geological sequestration or carbon sequestration include hydrodynamic
trapping, solubility trapping, and mineral trapping. Hence option (b) is the correct answer.
o Hydrodynamic trapping refers to that CO2 which is trapped as supercritical fluid or gas under a low-
permeability caprock. Carbon dioxide, being less dense than the formation fluid, will rise buoyantly
until it encounters a caprock.
o Solubility trapping: When CO2 is injected into a reservoir, a portion of the injected CO2 will dissolve
in the formation water in the aquifer and the dissolution of CO2 per unit volume of water is a function
of pressure, temperature, and salinity of the aqueous phase.
o Mineral carbonation refers to the incorporation of CO2 in a stable mineral phase via reactions with
mineral and organic matter in the formation. Over time the injected CO 2 will dissolve into the local
formation water and initiate a variety of geochemical reactions.
Q 8.A
 The International Solar Alliance (ISA) announced that the Global Solar Facility (GSF), formed by it
to stimulate investments into solar power projects, is set to receive a capital contribution of $35 million.
The Indian government is considering a $25 million investment as capital contribution in the GSF in
addition to $10 million coming from the ISA.
o It is a payment guarantee mechanism expected to stimulate investments into solar projects, with
two financial components:-
 Solar Payment Guarantee Fund to provide a partial guarantee and enable investments in
geographies that do not receive investments.
 Solar Insurance Fund to reduce the burden of insurance premium for solar developers in pre-
revenue phase of project.
 Initiatives taken by ISA but not under Solar Facility:-
o Green Grids Initiative - One Sun, One World, One Grid (OSOWOG).
o Global Energy Alliance for People and Planet (GEAPP) launched at COP26 with USD10
billions.
o ISA’s Programme on Scaling Solar Applications for Agriculture Use (SSAAU).
 Hence option (a) is the correct answer.

3 www.visionias.in ©Vision IAS


Q 9.B
 Resilient and Inclusive Supply-chain Enhancement—or RISE was launched by World Bank and
Japan—joined by Italy, the Republic of Korea, Canada, and the United Kingdom. Hence statement
1 is not correct.
 Resilient and Inclusive Supply-chain Enhancement—or RISE—initiative will help emerging
markets and developing countries (EMDCs) increase manufacturing of clean-energy products and
boost their participation in the minerals industry—leading to quality local jobs and economic
growth. Hence statement 2 is correct.
o Japan, Canada, Italy, the Republic of Korea, and the United Kingdom have pledged an initial total
contribution of more than $40 million to RISE, and more donors are expected to contribute.
 The energy transition will create a trillion-dollar market, with tens of millions of jobs along global mineral
value chains. It represents an opportunity for EMDCs to build manufacturing capacity, boost job growth,
and strengthen long-term economic development.

Q 10.C

 Greenhouse Gasses
o Greenhouse gases are a type of gas that traps heat in the atmosphere. This is what causes global
warming. Some of the most common greenhouse gasses include carbon dioxide (CO2), methane
(CH4), nitrous oxide (N2O), and fluorinated gases (SF6, HFCS, and PFCs).
 Global warming potential (GWP)
o Global warming potential (GWP) is a metric used to compare the relative impact of different
greenhouse gases on global warming. The GWP of a gas is defined as the amount of energy it absorbs
over a given period of time, relative to the same amount of CO2.

4 www.visionias.in ©Vision IAS


 Carbon dioxide (CO2):
o Carbon dioxide (CO2) is one of the primary greenhouse gases responsible for global warming. Global
warming potential (GWP) is a metric used to compare the relative impact of different greenhouse
gases on global warming. The GWP of a gas is defined as the amount of energy it absorbs over a
given period of time, relative to the same amount of CO2.
o The GWP of CO2 is defined as 1, meaning that it is the reference point for all other greenhouse
gases. The GWP of other gases is calculated by comparing their radiative forcing to that of CO2. For
example, methane (CH4) has a GWP of 21, which means that it traps 21 times more heat than the
same amount of CO2 over a 100-year period.
o GWPs are typically calculated over a 100-year time horizon, but they can also be calculated for
shorter or longer periods. This is because different greenhouse gases have different atmospheric
lifetimes. CO2, for example, has a very long atmospheric lifetime of about 100 years, meaning that it
can remain in the atmosphere for a long time and continue to trap heat. Methane, on the other hand,
has a shorter atmospheric lifetime of about 10 years, meaning that it is removed from the atmosphere
more quickly.
 Methane (CH4)
o Methane is a greenhouse gas that is about 56 times more potent than carbon dioxide. It is a major
contributor to climate change. Methane is released into the atmosphere from a variety of sources,
including:
 Natural sources: such as wetlands, wildfires, and termites.
 Human sources: such as agriculture, landfills, and oil and gas production.
 Nitrous Oxide (N2O)
o Nitrous oxide is a greenhouse gas that is about 280 times more potent than carbon dioxide. It is a
major contributor to climate change. Nitrous oxide is released into the atmosphere from a variety of
sources, including:
 Agricultural sources: such as fertilizers, manure, and irrigated agriculture.
 Industrial sources: such as nitric acid production and combustion processes.
 Hydrofluorocarbons (HFCs)
o Hydrofluorocarbons (HFCs) are a group of greenhouse gases that are used as refrigerants and in foam-
blowing applications. HFCs are very potent greenhouse gases. They are thousands of times more
potent than carbon dioxide. HFCs are not naturally occurring in the atmosphere. They are released
into the atmosphere from a variety of sources, including:
 Refrigeration and air conditioning
 Foam-blowing: HFCs are used in foam-blowing applications, such as in the production of
insulation and packaging materials.
 Sulphur Hexafluoride (SF6)
o Sulphur hexafluoride (SF6) is a greenhouse gas that is about 23,000 times more potent than carbon
dioxide. It is the most potent greenhouse gas known to man. SF6 is not naturally occurring in the
atmosphere. It is released into the atmosphere from a variety of sources, including:
o Electrical equipment: SF6 is used as an insulator in electrical equipment, such as power transformers
and switchgear.
o Semiconductor manufacturing: SF6 is used in the semiconductor manufacturing process.
 Hence option (c) is the correct answer.

Q 11.B
 Biochar is defined as a carbon-rich material produced during the pyrolysis process (not
incineration) that is a thermochemical decomposition of biomass with a temperature of about ≤700°C in
the absence or limited supply of oxygen. Hence statement 1 is not correct.
o Both incineration and pyrolysis are forms of combustion in which the thermal decomposition of
matter takes place. The key difference between incineration and pyrolysis is that incineration is the
combustion of organic matter in the presence of oxygen whereas pyrolysis is the combustion of
organic matter in the absence of oxygen.
 Biochar is primarily carbon (∼85%), but it can also contain oxygen, hydrogen, and inorganic ash if
present in the parent biomass. The heating value of biochar is in the range 25–32 MJ/kg dry basis, which
is substantially higher than that of the parent biomass or its liquid product. As biomass is carbon neutral,
the combustion of biochar is considered more environmentally friendly than coal.
 Biochar is characterized by a large pore surface area. Hence it has a large number of nonfuel uses such as
the adsorption of chemicals and carbon storage in the ground. When added back to the soil, biochar

5 www.visionias.in ©Vision IAS


absorbs and retains water and nutrients. Biochar enriches the soil and prevents it from leaching
pesticides and other nutrients into the runoff. Hence statement 2 is correct.
 Biochar is also an excellent carbon sink. When biomass is charred, it sequesters, or stores, its carbon
content. When biochar is added back to the soil, it can continue to absorb carbon and form large
underground stores of sequestered carbon that can lead to negative carbon emissions and healthier
soil. Hence statement 3 is correct.

Q 12.B
 Atmospheric lifetime of a greenhouse
o The atmospheric lifetime of a greenhouse gas is the average time it takes for half of the molecules of
that gas to be removed from the atmosphere. In other words, it is the length of time that a greenhouse
gas remains in the atmosphere before being removed through natural processes, such as chemical
reactions or absorption by the Earth's surface.
o The atmospheric lifetime of a greenhouse gas is important because it determines how long the gas will
continue to contribute to the greenhouse effect and climate change. Greenhouse gases with longer
atmospheric lifetimes, such as nitrous oxide (N2O) and hydrofluorocarbons (HFCs), will have a more
significant impact on climate change than gases with shorter atmospheric lifetimes, such as methane
(CH4).
Details of Green House gases:
 Methane (CH4)
o Atmospheric lifetime: 12 years
o Global warming potential (GWP): 25
o Sources: Natural sources such as wetlands, wildfires, and termites, as well as human sources such as
agriculture, landfills, and oil and gas production
o Properties: Methane is a colorless, odorless, and flammable gas. It is a potent greenhouse gas, with a
GWP of about 25 times that of carbon dioxide.
 Nitrous oxide (N2O)
o Atmospheric lifetime: 109 years.
o Hence option (b) is the correct answer.
 GWP: 280
o Sources: Natural sources such as soils and oceans, as well as human sources such as agriculture and
industrial processes
o Properties: Nitrous oxide is a colorless, odorless, and non-flammable gas. It is a potent greenhouse
gas, with a GWP of about 300 times that of carbon dioxide.
 Hydrofluorocarbons (HFCs)
o Atmospheric lifetime: Hundred years
o GWP: Thousands
o Sources: Man-made sources such as refrigeration and air conditioning, foam-blowing, and
semiconductor manufacturing
o Properties: HFCs are a group of colorless, odorless, and non-flammable gases. They are potent
greenhouse gases, with GWPs of thousands of times that of carbon dioxide.
 Chlorofluorocarbons (CFCs)
o Atmospheric lifetime: 50-100 years
o GWP: Thousands
o Sources: Man-made sources such as refrigerants, propellants, and foam-blowing agents
o Properties: CFCs are a group of colorless, odorless, and non-flammable gases. They are potent
greenhouse gases, with GWPs of thousands of times that of carbon dioxide.

Q 13.A
 Carbon trading began under the Kyoto Protocol of 1997 (which came into force in 2005). Under this,
‘certified emission reductions’ or CERs, were issued to entities that put up projects that reduced emissions
— such as wind, solar, or energy efficiency. Hence option (a) is the correct answer.
o It allows nations which are unable to meet their reduction targets to purchase carbon credits. Paris
Agreement also allows voluntary trading between countries to meet their NDC goals.
o If a country reduces more GHG emissions than its target, it can sell the emission reduction to another
country as an “internationally traded mitigation outcome.

6 www.visionias.in ©Vision IAS


Q 14.B
 Global Environment Facility (GEF), International Union for Conservation of Nature (IUCN) and
Conservation International announced the launch of a new global initiative to support the leadership
of Indigenous peoples and local communities in stewarding land, water and natural
resources. Hence, statement 1 is correct and 3 is not correct.
 The Inclusive Conservation Initiative (ICI) will support enhanced Indigenous and community stewardship
across 7.5 million hectares of landscapes, seascapes and territories with high biodiversity and
irreplaceable ecosystems. Recognizing the continuing historical role of Indigenous peoples and local
communities in safeguarding natural ecosystems, ICI will provide direct financial support to
Indigenous and locally-led initiatives in Africa, Central and South America, Asia and the Pacific.
Hence, statement 2 is correct.
 ICI provides site-based investments in nine subprojects to prioritize Indigenous and local community
organizations to take the lead in carrying out inclusive, culturally appropriate processes for decision-
making and strategy development that they have defined, implementing activities within their respective
territories, landscapes and/or sea scapes.
 The establishment of these nine subprojects in 12 countries demonstrates that there is high demand for
these inclusive finance models, as they were selected among over 400 Expressions of Interest (EOIs) that
were received from 80 countries.

Q 15.A
 Udangudi ‘Panangkarupatti’ (palm jaggery/ gur) from Tamil Nadu has been given a Geographical
Indication (GI) tag.
o The palm jaggery preparation procedure in this area is traditional to date without the inclusion of any
additional modern strategies.
o Triple superphosphate and phosphoric acid are used in many other areas, but no such chemical
additives are used in Udangudi Panangkarupatti. Hence pair 1 is not correctly matched.
o The karupatti prepared from the palm sap from the region around Udangudi in Tiruchendur taluk in
Thoothukudi district has some uniqueness. This is due to the presence of red sand dune soil found in
the region.
o This soil holds less groundwater. The moisture content in the atmosphere is less because of the dry
climatic conditions, which leads to high sucrose content, in turn adding taste.
 Khamti rice is a variety of sticky rice produced in the Namsai district of Arunachal Pradesh and is
known for its taste. It is cultivated by traditional Khampti tribal farmers. Hence pair 3 is correctly
matched.
 Tangsa textile is made by the Tangsa tribe of Changlang district in Arunachal Pradesh and is
famous for its exotic designs and colors. Hence pair 2 is not correctly matched.
 The first ever yak milk product, Arunachal Pradesh Yak Churpi also received a GI tag recently.

Q 16.B
 Carbon leakage refers to the situation that may occur if, for reasons of costs related to climate
policies, businesses were to transfer production to other countries with laxer emission constraints.
This could lead to an increase in their total emissions. Hence option (b) is the correct answer.
 How does Carbon Leakage work?
o We take an example of two countries A and B.
o Country A has a very strict emission policy and due to its stringent policy, the costs involved in the
production increases.
o Country B has a less strict and flexible emission policy and due to this flexible policy, the costs
involved in the production are less as compared to country A, keeping all other factors constant.
o So, a company located in country A faces increased costs due to emissions pricing as a result of the
strict climate policy. The company would take some action and as a result may decide to go for
reducing, closing or even relocating the production to Country B with less stringent climate
policies. This means that the Country A was though able to cut emissions, but now Country B
will increase the emissions due to transfer of greenhouse gas intensive industries from Country
A to B. The result is more Green House Gases emission and more industrial jobs.
 Other associated concepts –
o Carbon Pricing is an instrument that captures the external costs of greenhouse gas (GHG)
emissions - the costs of emissions that the public pays for, such as damage to crops, health care costs
from heat waves and droughts, and loss of property from flooding and sea level rise - and ties them to
their sources through a price, usually in the form of a price on the carbon dioxide (CO2) emitted.
7 www.visionias.in ©Vision IAS
o Carbon Tax - A carbon tax is a fee imposed on businesses and individuals that works as a sort
of "pollution tax." The tax is a fee imposed on companies that burn carbon-based fuels, including
coal, oil, gasoline, and natural gas.
o Carbon Trading is the buying and selling of credits that permit a company or other entity to emit a
certain amount of carbon dioxide. Carbon trade agreements allow for the sale of credits to emit
carbon dioxide between nations as part of an international agreement aimed at gradually
reducing total emissions.
o Zero Carbon Law by New Zealand - The Zero Carbon Act puts in place targets to reduce all
greenhouse gases: Carbon dioxide and nitrous oxide have to reduced to net zero by 2050.

Q 17.C
 As India shifts gears to meet its net-zero target by 2070, the transition towards clean energy and multiple
decarbonization initiatives will optimize the use of the country's energy mix and reduce the proportion of
fossil fuels over the long run.
 Since the Indian automotive industry has always been a key indicator of the nation’s economic growth,
contributing to approximately 6.5 percent of India’s overall gross domestic product; 40 percent of its
manufacturing GDP and supporting about 30 million direct and indirect jobs, the growth of this industry
should be in sync with the government’s vision of ‘green growth’. Going forward, the industry would
need to continue to build on a strong foundation of sustainability, environmental consciousness and safety.
Hence, a range of initiatives have been introduced by the Indian automobile industry under the
aegis of the Society of Indian Automobile Manufacturers (SIAM).
 One of the initiatives is for biofuels. Through a campaign called ‘Javik Pahal’, the industry
promotes biofuels produced from renewable biological materials. Alternative fuels can play a pivotal
role in powering vehicles, reducing the reliance on petroleum imports, lowering greenhouse gas emissions
and also improving farmer incomes. Hence statements 1 and 2 are correct.

Q 18.B
 Recent context: China to build the world’s largest ‘ghost particle’ or Neutrino detector.
 Neutrinos are a type of electron but, like neutrons, they do not have any charge. They are among the
most abundant particles in our universe — with trillions of neutrinos passing through you at any given
second — and also among the tiniest. Hence statement 3 is correct.
 Neutrinos were long believed to be massless until scientists found evidence that they do have a very
small mass. Hence statement 2 is correct.
 Neutrinos’ weak charge and almost nonexistent mass have made them notoriously difficult for scientists
to observe. They can only be “seen” when they interact with other particles.
o The rarity of interactions with other particles makes them almost impossible to track. That’s why
they’re called ghost particles — the vast majority skirt around undetected.
 They rarely interact with other particles. But rarely doesn’t mean “never”. Sometimes they interact with
water molecules, which is why China is building its ghost molecule telescope underwater. Hence
statement 1 is not correct.
 Scientists have observed ghost particles in fleeting instances when the particles create byproducts after
traveling through water or ice. These “muons” create flashes of light that can be detected by sophisticated
underwater telescopes and offer one of the fews ways to study the energy and source of neutrinos.
 Right now, the largest neutrino-detecting telescope is the University of Madison-Wisconson’s “IceCube”
telescope. Situated deep in the Antarctic, the telescope’s sensors span around 1 cubic kilometer.

Q 19.B
 The Convention on Wetlands of International Importance (Ramsar Convention):
o The Convention on Wetlands of International Importance (Ramsar Convention) was adopted in 1971
and entered into force in 1975.
o It is a legally binding intergovernmental framework instrument embodying the commitments of its
member countries to maintain the ecological character of their Wetlands of International Importance
and to plan for the "wise use", or sustainable use, of all wetlands in their territories. Hence statement
1 is correct
o The Convention’s mission is “the conservation and wise use of all wetlands through local and national
actions and international cooperation, as a contribution towards achieving sustainable development
throughout the world”.

8 www.visionias.in ©Vision IAS


 What are wetlands under the Ramsar Convention:
o Wetlands are areas where water is the primary factor controlling the environment and the associated
plant and animal life. They occur where the water table is at or near the surface of the land, or where
the land is covered by water. The Ramsar Convention takes a broad approach in determining the
wetlands which come under its aegis.
o Under the text of the Convention (Article 1.1), wetlands are defined as: “areas of marsh, fen,
peatland or water, whether natural or artificial, permanent or temporary, with water that
is static or flowing, fresh, brackish or salt, including areas of marine water the depth of which at
low tide does not exceed six meters”.
o This definition includes lotic water bodies, such as rivers and streams, if they meet specific
criteria. For example, they must be important waterbird habitats, representative examples of particular
types of wetland systems, or ecologically important for other reasons, such as providing flood control
or sediment retention. Hence statement 2 is not correct.
 Lotic water bodies:
o Lotic water bodies are water bodies that have a current or flow of water, such as rivers, streams,
and creeks.
o They are characterized by their high dissolved oxygen content and their ability to transport sediment
and nutrients.
o Lotic water bodies support a variety of aquatic and riparian (streamside) organisms, including fish,
insects, and plants.
 Lentic water bodies:
o Lentic water bodies are water bodies that do not have a current or flow of water, such as ponds,
lakes, and marshes.
o They are characterized by their low dissolved oxygen content and their tendency to stratify (form
layers) due to differences in temperature and density.
o Lentic water bodies support a variety of aquatic and emergent (water-loving) plants, as well as
amphibians, reptiles, and waterfowl.
 Five major wetland types are generally recognized under the Ramsar Convention:
o marine (coastal wetlands including coastal lagoons, rocky shores, and coral reefs);
o estuarine (including deltas, tidal marshes, and mangrove swamps);
o lacustrine (wetlands associated with lakes);
o riverine (wetlands along rivers and streams);
o palustrine (meaning “marshy” - marshes, swamps and bogs).
 Ramsar Wetlands in India:
o In the 75th year of Independence, the number of Ramsar sites in India stands at 75, covering an area
of 13,26,678 ha, thanks to the recent addition of 11 more wetlands to the list in 2022.
o Among the eleven new sites,
 four are in Tamil Nadu (Chitrangudi Bird Sanctuary, SuchindramTheroor Wetland Complex,
Vaduvur Bird Sanctuary, Kanjirankulam Bird Sanctuary)
 three in Odisha (Tampara Lake, Hirakud Reservoir, Ansupa Lake)
 two in Jammu & Kashmir (Hygam Wetland Conservation Reserve, Shallbugh Wetland
Conservation Reserve)
 and one each in Madhya Pradesh (Yashwant Sagar) and Maharashtra (Thane Creek).
 Currently India, with 75, has the largest network of Ramsar Wetlands in Asia surpassing China.
Hence statement 3 is correct.

Q 20.A
 E-waste, electronic waste, e-scrap and end-of-life electronics are terms often used to describe used
electronics that are nearing the end of their useful life, and are discarded, donated or given to a recycler.
 The UN defines e-waste as any discarded products with a battery or plug, and features toxic and
hazardous substances such as mercury, that can pose severe risk to human and environmental health.
 The overarching objective of the Basel Convention on the Control of Transboundary Movements of
Hazardous Wastes and their Disposal is to protect human health and the environment against the
adverse effects of hazardous wastes. E-waste is categorized as hazardous waste due to the presence
of toxic materials such as mercury, lead and brominated flame retardants are considered as
hazardous waste according to the Basel Convention. In addition, transboundary movements of hazardous
and other wastes, including e-waste ending up in dumps, are deemed to be illegal traffic under the Basel
Convention, Article 9. Hence, statement 1 is correct.

9 www.visionias.in ©Vision IAS


 India is the third largest electronic waste generator in the world after China and the USA and these
three countries together contributed 38% of total 53.6 million tonnes (Mt) of e-waste generated world
wide in 2019. Hence, statement 2 is not correct.

Q 21.A
 Recently, in 2023 the Dhordo village in the Kutch district of Gujarat was awarded as the Best
Tourism Village by the United Nations World Tourism Organization (UNWTO).
o From Salty marshland to an Iconic destination of the Western Indian Subcontinent – Dhordo, in the
Rann of Kutch, has become the face of Gujarat’s development.
o Tourism was launched through Rann Utsav, a four-month festival whereby a special tent city
was created in the desertic land with all the necessary amenities made available to tourists.
o It is during these four months that the village comes to life displaying all the natural and cultural
resources/products to its consumers.
 Mawlynnong is a village in the East Khasi Hills district of the Meghalaya state in North East India. It is
notable for its cleanliness and also was chosen by Discover India magazine as Asia's cleanest village.
 The remote village of Malana is nestled in the pristine Parvati Valley of Himachal Pradesh, and stands
as a mysterious and enchanting destination. Known for its unique customs, centuries-old traditions, and
breathtaking natural beauty, Malana is a place that has captured the imaginations of travelers for years.
 Odanthurai of Tamil Nadu has taken energy generation to another level. The village not only produces
its own electricity but also sells it to the state’s government. The village has come up with self-help
ventures Wind-power generation.
o They have replaced the grid electricity with a 9kW biomass gasifier power generation system for the
means of pumping water to the houses.
 Hence option (a) is the correct answer.

Q 22.B
 The Least Developed Countries Fund (LDCF) was established at the 7th Conference of the Parties
in 2001 (COP7) to meet the adaptation needs of least developed countries (LDCs). Hence, statement
1 is not correct.
 LDCF is active in sectors including water, agriculture and food security, health, disaster risk management
and prevention, infrastructure and fragile ecosystems and has the largest portfolio of adaptation projects of
its kind. The Global Environmental Facility (GEF) administers the LDCF as a specialized trust fund and
serves as a basis for programming resources.
 The LDCF aims to address the needs of the 51 LDCs which are particularly vulnerable to the
adverse impacts of climate change. As a priority, the LDCF supports the preparation and
implementation of the National Adaptation Programmes of Action (NAPAs), which are country-
driven strategies that identify the immediate needs of LDCs in order to adapt to climate change.
Hence, statements 2 and 3 are correct.

Q 23.C
 The India GHG Program led by WRI India, Confederation of India Industry (CII) and The Energy and
Resources Institute (TERI) is an industry-led voluntary framework to measure and manage
greenhouse gas emissions. Hence option (c) is the correct answer.
 The programme builds comprehensive measurement and management strategies to reduce emissions and
drive more profitable, competitive and sustainable businesses and organisations in India.
 The programme is supported by the Shakti Sustainable Energy Foundation, the German Federal Ministry
for the Environment, Nature Conservation and Nuclear Safety (BMU) and Pirojsha Godrej Foundation.

Q 24.C
 Recently, Scientists have made history by successfully turning on the world's smallest particle
accelerator for the first time. With a size similar to a small coin, this astonishing discovery
holds enormous potential for a variety of applications, including the use of miniature particle
accelerators for medicinal purposes. Hence statements 1 and 2 are correct.
 The NEA is a cutting-edge technology made up of a microscopic microprocessor and an even
smaller vacuum tube made up of thousands of individual pillars.
 By directing microscopic laser beams onto these tiny pillars, researchers have developed a way for
accelerating electrons. Surprisingly, the main acceleration tube within the NEA is only 0.02 inches (0.5
millimeters) long, which is 54 million times shorter than the massive 16.8-mile-long (27-kilometer) ring
10 www.visionias.in ©Vision IAS
housing CERN's Large Hadron Collider (LHC) in Switzerland, the world's largest and most powerful
particle accelerator.
 Nano-Scale Precision: The interior of the NEA's tiny tunnel is around 225 nanometers broad. According
to the National Nanotechnology Institute, the thickness of a human hair ranges between 80,000 and
100,000 nanometers.
 Acceleration of Electrons: Researchers from the Friedrich-Alexander University of Erlangen-Nuremberg
(FAU) in Germany used the NEA to accelerate electrons in a recent study published in the journal Nature.
They effectively increased electron energy from 28.4 kiloelectron volts (keV) to 40.7 keV, a 43%
increase. This is the first successful activation of a nanophotonic electron accelerator, which was first
proposed in 2015.
 It is a Particle Acceleration on a Microchip.
 In contrast to Large Particle Accelerators: The LHC uses almost 9,000 magnets to create a magnetic
field for particle acceleration, whereas the NEA uses light beams focused at the vacuum tube's
pillars to magnify energy, resulting in a far lower energy field. Electrons accelerated by the NEA have
substantially less energy than electrons accelerated by major colliders such as the LHC. However, experts
believe that by experimenting with different materials or stacking many tubes together, they can improve
particle acceleration even further. Despite this, the NEA will not achieve energy levels equivalent to those
of massive particle colliders. Hence statement 3 is correct.
 Possibilities for Medical Applications: One of the key goals of developing these small accelerators is
to exploit the energy released by accelerated electrons to deliver tailored medicinal treatments,
perhaps replacing more invasive forms of radiotherapy used to battle cancer cells. The ultimate goal
is to mount a particle accelerator on an endoscope, allowing radiotherapy to be delivered directly to
afflicted parts of the body. This transformational application, however, remains a long-term goal.
Q 25.A
 E-Waste Rules (Management), 2022
 These rules shall apply to every manufacturer, producer refurbisher, dismantler and recycler involved in
the manufacture, sale, transfer, purchase, refurbishing, dismantling, recycling and processing of e-waste or
electrical and electronic equipment listed in Schedule I, including their components, consumables, parts
and spares which make the product operational.
 But they shall not apply to - (a) waste batteries as covered under the Battery Waste Management
Rules, 2022; (b) packaging plastics as covered under the Plastic Waste Management Rules, 2016; (c)
micro-enterprise as defined in the Micro, Small and Medium Enterprises Development Act, 2006
(27 of 2006); and (d) radio-active wastes as covered under the provisions of the Atomic Energy Act,
1962 (33 of 1962) and rules made there under. Hence, statement 1 is correct.
 ‘e-retailer’ means an individual company or business entity that uses an electronic network such as
the internet, social media, telephone or any other media, to sell its goods. Hence, statement 2 is not
correct.
 ‘historical e-waste’ means e-waste generated from electrical and electronic equipment as specified in
Schedule I which was available on the date from which these rules came into force;
 'orphaned products’ means non-branded or assembled electrical and electronic equipment as
specified in Schedule I or those produced by a company which has closed its operations; Hence,
statement 3 is not correct.
 ‘extended producer responsibility’ means the responsibility of any producer of electrical or electronic
equipment as given in Schedule-I for meeting recycling targets as per Schedule-III and Schedule-IV, only
through registered recyclers of e-waste to ensure environmentally sound management of such waste;

Q 26.A
 Statement 1 is correct. ‘Zero budget’ farming promises to end a reliance on loans and drastically cut
production costs, ending the debt cycle for desperate farmers. The word ‘budget’ refers to credit and
expenses, thus the phrase 'Zero Budget' means without using any credit, and without spending any money
on purchased inputs.
 Statement 2 is correct. The main aim of ZBNF is eliminate use of chemical pesticides and uses
biological pesticides and promote of good agronomic practices. Farmers use earthworms, cow dung,
urine, plants, human excreta and such biological fertilizers for crop protection.
 Statement 3 is not correct. It has attained wide success in southern India, especially the southern Indian
state of Karnataka where it first evolved. The movement in Karnataka state was born out of collaboration
between Mr Subhash Palekar, who put together the ZBNF practices, and the state farmers association
Karnataka Rajya Raitha Sangha (KRRS).
11 www.visionias.in ©Vision IAS
Q 27.C
 Recent context : Raising concerns over the use of World Bank’s Worldwide Governance Indicators
in ratings assessment by credit ratings agencies, especially for emerging economies, Chief Economic
Adviser V Anantha Nageswaran said there is a need for the World Governance Index to be more
transparent and less subjective.
 The World Bank’s Worldwide Governance Indicators provide a ranking of 215 countries territories
based on six dimensions of governance: ‘Voice and Accountability’; ‘Political Stability and Absence of
Violence’; ‘Government Effectiveness’; ‘Regulatory Quality’; ‘Rule of Law’ and ‘Control of Corruption.
 The WGI were developed in 1999 by two World Bank researchers, Daniel Kaufmann and Aart Kraay. The
data are updated annually each September.
 The WGI aggregate data from more than 30 think tanks, international organizations,
nongovernmental organizations, and private firms across the world selected on the basis of three key
criteria: 1) they are produced by credible organizations; 2) they provide comparable cross-country data;
and 3) they are regularly updated.
 Hence option (c) is the correct answer.

Q 28.A
 The United Nations Environment Assembly (UNEA):
o The United Nations Environment Assembly (UNEA) is the world's highest-level decision-
making body on the environment. It is a universal body with the membership of all 193 UN
Member States. Hence statement 2 is correct.
o UNEA meets every two years to set the global environmental agenda, provide overarching policy
guidance, and define policy responses to address emerging environmental challenges. Hence
statement 3 is not correct.
 Establishment of UNEA
o UNEA was established in 2012 as a result of the United Nations Conference on Sustainable
Development (UNCSD), also known as Rio+20. Hence statement 1 is not correct.
o The UN General Assembly then adopted resolution on strengthening and upgrading UNEP and
establishing universal membership of its Governing Council (GC).
o In March 2013, the UN General Assembly further adopted resolution, which changed the designation
of the UNEP Governing Council (GC) to the UNEA of the UNEP.
 Key Functions of UNEA
UNEA plays a pivotal role in shaping international environmental policy and driving action towards a
sustainable future. Its key functions include:
o Setting environmental priorities: UNEA identifies and prioritizes key environmental issues that
require international attention and action.
o Developing policy frameworks: UNEA develops and adopts policy frameworks to address global
environmental challenges, providing a roadmap for action.
o Monitoring and review: UNEA monitors the implementation of environmental policies and
agreements, conducting regular reviews to assess progress and identify areas for improvement.
o Strengthening environmental governance: UNEA strengthens environmental governance by
providing a platform for international cooperation and collaboration on environmental matters.
o Promoting sustainable development: UNEA promotes sustainable development by integrating
environmental considerations into social and economic policies.
 Structure of UNEA
UNEA is governed by the Conference of the Parties (COP), which is composed of representatives from all
193 UN Member States. The COP meets every two years to deliberate and adopt decisions on
environmental matters.

Q 29.D
 The Ministry of New and Renewable Energy (MNRE) is the nodal Ministry of the Government of India
for all matters relating to new and renewable energy. The broad aim of the Ministry is to develop and
deploy new and renewable energy to supplement the energy requirements of the country.
 Renewable Energy in India includes:
o Wind energy
o Nuclear energy
o Solar energy
o Biomass energy

12 www.visionias.in ©Vision IAS


o Geothermal energy
o Hydropower
 Hence option (d) is the correct answer.

Q 30.A
 As a specialized UN agency to address all matters related to international civil aviation, including
environmental protection, ICAO has been diligently addressing GHG emissions from international
aviation. The ICAO agreement on carbon-neutral growth and CORSIA complements the ambition of the
Paris Agreement and constitutes the most significant international climate-related agreement since its
adoption.
 CORSIA is the first global market-based measure for any sector and represents a cooperative
approach that moves away from a “patchwork” of national or regional regulatory initiatives. It
offers a harmonized way to reduce emissions from international aviation, minimizing market
distortion, while respecting the special circumstances and respective capabilities of ICAO Member
States. Hence, statement 1 is correct.
 CORSIA complements the other elements of the basket of measures by offsetting the amount of CO2
emissions that cannot be reduced through the use of technological improvements, operational
improvements, and sustainable aviation fuels with emissions units from the carbon market.
 Why does the Paris Agreement not include international aviation emissions?
o Specifically, governments working under the auspices of the UNFCCC have agreed that while all
domestic GHG emissions are dealt with under the UNFCCC, GHG emissions associated with
international aviation and maritime transport are to be dealt with under ICAO and
International Maritime Organization (IMO), respectively. This approach is consistent with similar
UNFCCC decisions that also apply to the Kyoto Protocol.
o In this regard, GHG emissions from domestic aviation, as per other domestic sources, are calculated
as part of the UNFCCC national GHG inventories and are included in national totals (part of the
Nationally Determined Contributions (NDCs) of the Paris Agreement), while GHG emissions from
international aviation are reported separately and are not included in NDCs.
 CORSIA will be implemented in three phases, as follows:
o Pilot phase: from 2021 to 2023;
o First phase: from 2024 to 2026; and
o Second phase: from 2027 to 2035.
 For the pilot and first phase, participation is voluntary, while for the second phase, participation is
based on the States' RTK level in 2018 and voluntary participation. Hence, statement 2 is not
correct.

Q 31.B
 The Montreal Protocol on Substances that Deplete the Ozone Layer is a global agreement to protect
the Earth’s ozone layer by phasing out the chemicals that deplete it. This phase-out plan includes both
the production and consumption of ozone-depleting substances.
 The landmark agreement was signed in 1987 and entered into force in 1989.
 The most recent amendment, the Kigali Amendment, called for the phase-down of hydrofluorocarbons
(HFCs) in 2016. These HFCs were used as replacements for a batch of ozone-depleting substances
eliminated by the original Montreal Protocol.
 Although they do not deplete the ozone layer, they are known to be powerful greenhouse gases and, thus,
contributors to climate change.The Montreal Protocol provided a set of practical, actionable tasks that
were universally agreed on.
 Under the Kigali Amendment; Parties to the Montreal Protocol will phase down production and
consumption of Hydrofluorocarbons, commonly known as HFCs.
o Hydrofluorocarbons were introduced as a non-ozone-depleting alternative to
Hydrofluorocarbons (HFCs). Hence, statement 1 is not correct.
13 www.visionias.in ©Vision IAS
o While HFCs do not deplete the stratospheric ozone layer, they have a high global warming potential
ranging from 12 to 14,000, which has an adverse impact on climate.
o Recognizing the growth in the use of HFCs, especially in the Refrigeration and Air-conditioning
sector the Parties to the Montreal Protocol, reached an agreement at their 28th Meeting of the Parties
(MOP) held in October 2016 in Kigali, Rwanda to add HFCs to the list of controlled substances and
approved a timeline for their gradual reduction by 80-85 per cent by the late 2040s. Hence,
statement 2 is correct.
o India will complete its phase-down of HFCs in 4 steps from 2032 onwards with a cumulative
reduction of 10% in 2032, 20% in 2037, 30% in 2042 and 85% in 2047.
o All amendments and adjustments of the Montreal Protocol, prior to the Kigali Amendment have
Universal support.
o India has successfully met the phase-out targets of all the Ozone Depleting Substances as per the
Montreal Protocol Schedule. Hence, statement 3 is correct.
Q 32.C
 Ethanol, anhydrous ethyl alcohol is produced from sugarcane, maize, wheat, etc which are having high
starch content. In India, ethanol is mainly produced from sugarcane molasses by the fermentation process.
Ethanol can be mixed with the gasoline to form different blends.
 The Ethanol Blending Programme (EBP) seeks to achieve the blending of Ethanol with motor spirit with a
view to reducing pollution, conserving foreign exchange and increasing value addition in the sugar
industry enabling them to clear cane price arrears of farmers.The level of ethanol blending in petrol in
India has reached 9.99 per cent.
 India had targeted 10 per cent ethanol blending in petrol by the end of 2022 and 20 per cent blending by
2030 as per National Biofuel Policy-2018. Later the Central Government advanced it by five years for
achieving 20% ethanol blending in petrol.
 The amended National Biofuel Policy-2018 has now set the new target for 2025-26 instead of
2030. To achieve this target within the timeframe the central government announced premium rates for
ethanol produced from sugar syrup, cane juice as well as B-heavy molasses. Hence statement 1 is
correct.
 The Centre has also targeted a 5 per cent blending of biodiesel with diesel by 2030. Mixing 20 per cent
ethanol in a petrol can potentially reduce the auto fuel import bill by a yearly Rs 30,000 crore.
 In India, the Department of Food and Public Distribution (DFPD) is the nodal department for the
promotion of fuel-grade ethanol-producing distilleries in the country. It launched the Interest
Subvention Scheme for enhancement and augmentation of the ethanol production capacity. Hence
statement 2 is correct.

Q 33.B
 Ethanol is one of the principal biofuels, which is naturally produced by the fermentation of sugars by
yeasts or via petrochemical processes such as ethylene hydration. It has medical applications as an
antiseptic and disinfectant. It is used as a chemical solvent and in the synthesis of organic compounds,
apart from being an alternative fuel source.
 Ethanol blending offers significant advantages such as an increase in the Research Octane Number
(RON) of the blend, fuel-embedded oxygen and higher flame speed. These properties of ethanol
help in complete combustion and reduce vehicular emissions such as hydrocarbon, carbon
monoxide and particulate matter.
 The calorific value of ethanol is around 2/3rd of gasoline. This indicates that the increase in ethanol
content will decrease the heating value of the ethanol-gasoline blend. Hence, more fuel is required to
achieve the same engine power output.
 However, ethanol has a higher octane number and thus the engine can be operated with a high
compression ratio without knocking. This increases the efficiency of the engine considerably.
 This combined with optimal spark timing negates the fuel economy debit due to the low calorific
value of ethanol. Hence, ethanol is considered as an efficient fuel provided suitable modifications
are made in the vehicle. Hence, option (b) is the correct answer.

Q 34.C
 The wastewater containing hazardous waste chemicals should be detoxified and neutralised through
treatment.
 There are many technologies available for treating hazardous wastes before they are ultimately disposed
of.
14 www.visionias.in ©Vision IAS
 Their aim is to modify the physical and/or chemical properties of the wastes so that they are rendered
harmless.
 Selection of a treatment process depends on many factors such as the nature of the waste, the desired
characteristics of the output stream, and economic and energy considerations.
 The treatment technologies can be divided into the following groups, namely:
o Physical treatment
 is conducted using various methods such as phase separation. Phase separation includes three
steps, namely: lagooning, prolonged storage in tanks and sludge drying in beds. Lagooning and
tank storage are collectively used to separate particulate impurities.
o Chemical treatment
 is used to facilitate the complete breakdown of hazardous wastes and more usually to modify the
chemical properties of the wastes, e.g., to reduce water solubility or to neutralise acidity or
alkalinity. The techniques involve oxidation, chemical reduction, neutralization, heavy metal
precipitation, oil-water separation and solvents/ fuels recovery.
o Biological treatment
 The gross impurities obtained from the treatment of sewage are collectively known as sludge,
which is given biological treatment, before disposal.
 This is known as sludge processing which has become important since improvements in industrial
waste water treatment.
 The typical technologies for sludge processing include conditioning, digestion, composting,
thickening or dewatering and solidification.
 Conditioning: In this step, the sludge is exposed to the atmosphere for a stipulated period
until a desired consistency is reached.
 Digestion: In this process, the sludge is treated with bacteria which break down the long-
chain compounds into simpler ones.
 Composting: In this step, the organic matter in the waste sludge is converted into a usable
stable material.
o Solidification
 processes convert liquid waste into insoluble, rock-hard material and are used as pre-treatment
prior to landfill disposal. This is usually done by mixing the waste with various reactants to
produce a solid mass. The basic aim of the solidification process is to immobilize the hazardous
constituents of the waste so that these do not leach out at the landfill disposal site.
o Incineration
 Apart from the above-mentioned methods, incineration is also a method of detoxification, in
which oxidation of waste detoxifies the waste from its toxic proportion.
o Hence option (c) is the correct answer.

Q 35.A
 National Action Plan on Climate Change (NAPCC)
The National Action Plan on Climate Change (NAPCC) is a comprehensive framework for India's climate
change strategy. It was launched in 2008 and outlines eight key missions to address climate change in
various sectors of the economy.
o The Eight Missions and Objectives of NAPCC
 National Mission on Sustainable Habitat: Promote sustainable urban planning and
transportation to reduce the impact of cities on the environment. It is executed under the Ministry
of Housing and Urban Affairs (MoHUA)
 National Mission for Enhanced Energy Efficiency: Improve energy efficiency across all
sectors of the economy to reduce greenhouse gas emissions. It is executed under the Ministry of
Power (MoP)
 National Mission For Sustainable Agriculture (Nmsa): Promote sustainable agricultural
practices to reduce greenhouse gas emissions and enhance food security. It is executed under the
Ministry of Agriculture and Farmers Welfare (MoAFW)
 National Water Mission: Conserve and efficiently use water resources to adapt to the impacts of
climate change. It is executed under the Ministry of Jal Shakti.
 National Mission For Sustaining The Himalayan Ecosystem: Protect and preserve the
Himalayan ecosystem, which is highly vulnerable to climate change. It is executed under the
Department of Science & Technology, Ministry of Science & Technology.
 National Solar Mission: It aims to increase the share of solar energy in India's energy mix. It is
executed under the Ministry of New and Renewable Energy (MNRE)
15 www.visionias.in ©Vision IAS
 National Mission on Strategic Knowledge for Climate Change (NMSKCC): It aims to
generate and disseminate knowledge about climate change to inform policy decisions and
adaptation strategies. It is executed under the Department of Science & Technology, Ministry of
Science & Technology.
 National Mission on Natural Farming (NMNF)
o The National Mission on Natural Farming (NMNF) is a separate and independent scheme launched
by the Government of India in 2023-24. It aims to promote natural farming practices in India and to
reduce the use of chemical fertilizers and pesticides. It is not included in NAPCC.
o To motivate farmers to adopt chemical-free farming and enhance the reach of natural farming, the
Government has formulated the National Mission on Natural Farming (NMNF) as a separate and
independent scheme from 2023-24 by upscaling the Bhartiya Prakritik Krishi Paddati (BPKP).
o The success of NMNF will require behavioral change in farmers to shift from chemical-based inputs
to cow-based locally produced inputs and thus requires continuous creation of awareness, training,
handholding, and capacity building of farmers in the initial years.
 Hence Option (a) is the correct answer.

Q 36.D
 Recent context: India is embarking on a major program to launch its maiden national-level framework
towards providing climate services and information.
o Spearheaded by the India Meteorological Department (IMD), the National Framework for Climate
Services (NFCS) envisions bringing a seamless working platform for users of climate information and
services and helping decide and mitigate climate risks for key sectors — agriculture, energy, disaster
management, health, and water.
 In line with the global framework, the national framework will be based on country-specific weather and
stakeholder needs. Unlike the GFCS, the nodal agency for the formulation and implementation of the
national framework in India will be the IMD.
 The Global Framework for Climate Services (GFCS) was established by the international
community at the World Climate Conference-3 in 2009 to enable better management of the risks of
climate variability and change, and adaptation to climate change, through the development and
incorporation of science-based climate information and prediction into planning, policy and practice on
the global, regional and national scale. Hence statements 1 and 2 are not correct.
o At the invitation of the Government of Switzerland, World Climate Conference-3 (WCC-3) was held
in Geneva, Switzerland, from 31 August to 4 September 2009.
o It was organized by the World Meteorological Organization (WMO), in collaboration with the United
Nations Educational, Scientific and Cultural Organization (UNESCO), the United Nations
Environment Programme (UNEP), the Food and Agriculture Organization of the United Nations
(FAO), the International Council for Science (ICSU) and other intergovernmental and non-
governmental partners.
 The main sources of GFCS funding thus far are Member’s contributions to the GFCS Trust Fund
or through bilateral and multi-lateral investments for projects in selected countries or regions.
o In addition, various actors can support projects – listed in a compendium of GFCS projects – or
designate their activities as contributing to the GFCS if they fulfill the criteria endorsed by the
Intergovernmental Board on Climate Services (IBCS)
Q 37.D
 Recent context: A new study has revealed that two species of mosquitofish also known as Gambusia Fish
have invaded various ecosystems across India.
 Gambusia fish are native to the southeastern United States and parts of Mexico, but they have been
introduced to various regions worldwide for mosquito control.
 Gambusia fish are often used for biological mosquito control. They are known for their voracious appetite
for mosquito larvae. Introducing these fish into water bodies can help reduce mosquito populations.
 Gambusia fish are livebearers, meaning they give birth to live young ones instead of laying eggs.
They are known for their rapid reproduction rates.
 While Gambusia fish are effective at controlling mosquito larvae, their introduction to non-native
habitats has been controversial. In some cases, they have outcompeted or preyed upon native fish
species, affecting the local ecosystem. Hence statement 1 is not correct.
o Wildlife biologists and conservations consider mosquitofish to be among the hundred most
detrimental invasive alien species. Thus they are not critically endangered. Hence statement 2 is
not correct.

16 www.visionias.in ©Vision IAS


o In 2018, the National Biodiversity Authority of the Government of India also designated G. affinis
and G. holbrooki as invasive alien species.
 Gambusia Fish do not have any symbiotic relationship with Olive Ridley Turtles. Hence statement 3
is not correct.
Q 38.B
 The Convention on International Trade in Endangered Species of Wild Fauna and Flora (CITES):
o The Convention on International Trade in Endangered Species of Wild Fauna and Flora (CITES) is an
international agreement between governments that aims to ensure that international trade in specimens
of wild animals and plants does not threaten the survival of the species in the wild. It is a legally
binding agreement that is implemented by the Parties to the Convention, which are currently 183
countries. Hence statement 1 is not correct.
o Although CITES is legally binding on the Parties – in other words, they have to implement the
Convention – it does not take the place of national laws.
o CITES was drafted as a result of a resolution adopted in 1963 at a meeting of members of IUCN (The
World Conservation Union).
 CITES Implementation:
CITES is implemented through a system of permits and quotas. All international trade in specimens of
species listed on the CITES Appendices must be accompanied by a permit from the appropriate export or
import authority. Quotas are also used to control the trade in certain species, such as elephants and tigers.
o CITES Appendices: CITES has three Appendices:
 Appendix I: Includes all species threatened with extinction that are or may be affected by
international trade.
 Appendix II: Includes all species which are not necessarily threatened with extinction but which
may become so if trade is not strictly controlled.
 Appendix III: Includes all species which any Party to the Convention identifies as being subject
to regulation within its jurisdiction for the purpose of preventing or restricting exploitation and
which it has requested the Conference of the Parties to the Convention to add to the Appendices.
 The CITES Secretariat
The CITES Secretariat is administered by UNEP and is located in Geneva, Switzerland. It has a
pivotal role, fundamental to the Convention, and its functions are laid down in Article XII of the text of
the Convention. Hence statement 2 is correct. They include:
o Playing a coordinating, advisory, and servicing role in the working of the Convention.
o Assisting with communication and monitoring the implementation of the Convention to ensure that its
provisions are respected.
o Providing assistance in the fields of legislation, enforcement, science, and training.
Q 39.A
 The International Whaling Commission (IWC):
The International Whaling Commission (IWC) was established in 1946 to regulate whaling and conserve
whale stocks. The Commission has 88 member countries and meets every other year to review the status
of whale stocks and adjust conservation measures.
The IWC regulates three types of whaling: commercial, scientific research, and aboriginal subsistence
whaling.
 Commercial Whaling
o In 1986, a global moratorium was placed on commercial whaling due to overexploitation of
whale stocks. However, some countries, such as Norway and Iceland, continue to engage in
commercial whaling. Hence statement 1 is correct.
o In recent years, Norway and Iceland have caught whales commercially. The Government of Norway
lodged a formal objection to the moratorium decision when it was introduced. Iceland left the IWC in
1992 and re-joined in 2002 with a reservation to the moratorium. When the moratorium was
introduced, the Russian Federation also registered an objection but did not exercise it. The
moratorium is binding on all other members of the IWC.
 Scientific research
o Scientific research whaling is permitted under special permits issued by the IWC. Japan's lethal
scientific research in the North Pacific and Southern Oceans ended in 2019.
 Subsistence whaling
o Aboriginal subsistence whaling is also regulated by the IWC. Aboriginal subsistence whaling is
the practice of hunting and killing whales for food, cultural, and spiritual purposes by
indigenous peoples who have a long history of whaling traditions. Hence statement 2 is not
correct.
17 www.visionias.in ©Vision IAS
o Indigenous communities in Denmark, the Russian Federation, St. Vincent and the Grenadines, and the
United States are permitted to engage in this type of whaling.
o The IWC places a strong emphasis on scientific advice and has established a Scientific Committee to
provide guidance. The Scientific Committee meets annually and has produced catch-limit algorithms
to ensure sustainable whaling practices.
 Members
o Membership of the IWC is open to any country in the world. Each member formally adheres to the
1946 International Convention for the Regulation of Whaling and is represented by a Commissioner
who is nominated by their government. Currently, there are 88 member nations in the IWC.
o India has been a member of the International Whaling Commission since 1981 and has
maintained a consistent stand on whale conservation.
o Japan left the IWC in 2019 and began to catch whales commercially the same year. Having left
the IWC is no longer bound by the moratorium. Japanese catches are also reported to the IWC.
 Other G2O nations that are not members of the IWC include Canada, Indonesia, Saudi Arabia, and
Turkiye along with Japan.
Q 40.C
 Indo-Pacific Partnership for Maritime Domain Awareness (IPMDA) is a technology and training
initiative to enhance maritime domain awareness in the Indo-Pacific region and to bring increased
transparency to its critical waterways. IPMDA harnesses innovative technology, such as commercial
satellite radio frequency data collection, to provide partners across Southeast Asia, the Indian Ocean
region, and the Pacific with near real-time information on activities occurring in their maritime zones.
 To contribute to maritime domain awareness in the Indo-Pacific region, which is fundamental for stability
and prosperity. At the 2022 Quad Leaders’ Summit in Tokyo, Quad Leaders announced the Indo-
Pacific Partnership for Maritime Domain Awareness (IPMDA) to enhance existing maritime domain
awareness capabilities.
 Only QUAD countries are members of the IPMDA initiative, which are India, Japan, the USA, and
Australia.
 Hence option (c) is the correct answer.

Q 41.B
 Statement 1 is not correct: Global Footprint Network is an independent think tank originally founded in
2003. It was established as a charitable not-for-profit organization. Global Footprint Network develops
and promotes tools for advancing sustainability, including the ecological footprint and biocapacity, which
measure the amount of resources we use and how much we have. These tools aim at bringing ecological
limits to the center of decision-making.
 Every year, Global Footprint Network produces a new edition of its National Footprint Accounts, which
calculate Ecological Footprint and biocapacity of more than 200 countries and territories from 1961 to the
present. It has also launched Ecological Footprint Explorer, an open data platform for the National
Footprint Accounts.
 Statement 2 is correct: Earth Overshoot Day marks the date when humanity’s demand for ecological
resources and services in a given year exceeds what Earth can regenerate in that year. Earth Overshoot
Day is hosted and calculated by Global Footprint Network.
 World Summit on Sustainable Development, 2002 (WSSD)
o Also known as the Earth Summit, it was held in Johannesburg, South Africa. It was convened to
discuss sustainable development by the United Nations. WSSD gathered a number of leaders from
business and non-governmental organizations, 10 years after the first Earth Summit in Rio de Janeiro.
(It was therefore also informally nicknamed "Rio+10").
Q 42.D
 The National River Conservation Plan (NRCP) was initiated in 1995. Subsequently, all the projects for
river cleaning in the country were brought under NRCP. It is under the purview of Ministry of
Environment, Forest & Climate Change. Hence, option (a) is not correct.
 The objective of the NRCP is to reduce the pollution load in rivers through implementation of various
pollution abatement works which include -:
o Interception and diversion works/ laying of sewerage systems to capture raw sewage flowing into the
rivers through open drains and diverting them for treatment.
o Setting up of Sewage Treatment Plants (STPs) for treating the diverted sewage
o Construction of Low Cost Sanitation Toilets to prevent open defecation on river banks.
o Construction of Electric Crematoria and Improved Wood Crematoria to conserve the use of wood.

18 www.visionias.in ©Vision IAS


o River Front Development works, such as improvement of bathing ghats.
o Public participation & awareness and capacity building, etc.
 Hence, option (d) is correct.
 NRCP doesn’t have any sub-schemes. National Lake Conservation Programme is a separate Centrally
Sponsored Schemes (CSS) under Ministry of Environment, Forest & Climate Change with an objective to
restore and conserve the urban and semi-urban lakes of the country. Since 2013, National Lake
Conservation Programme along with the National Wetlands Conservation Programme (NWCP) have been
merged into a new integrated scheme named ‘National Plan for Conservation of Aquatic Eco-systems
NPCA’. Hence, option (b) is not correct.
 Currently, NRCP (excluding Ganga and its tributaries) has covered polluted stretches of 33 rivers in 76
towns spread over 15 States at a sanctioned cost of Rs.4801.57 crore. NRCP covers the following states -
Andhra Pradesh; Goa; Gujarat; Jammu & Kashmir; Jharkhand; Karnataka; Kerala; Madhya Pradesh;
Maharashtra; Nagaland; Odisha; Punjab; Sikkim; Tamil Nadu; Telangana. Hence, option (c) is not
correct.

Q 43.A
 Recently, at COP28, 196 parties adopted the decision to operationalise the Loss and Damage fund.
Hence statement 2 is not correct.
 Loss and damage refers to the negative consequences that arise from the unavoidable risks of climate
change, like rising sea levels, prolonged heatwaves, desertification, the acidification of the sea and
extreme events, such as bushfires, species extinction and crop failures.
 The fund was set up during COP27 at Sharm el-Shaikh in Egypt, 2022. The establishment of a Loss
and Damage Fund was, for many, the highlight of the United Nations Climate Conference (COP 27) and
the culmination of decades of pressure from climate-vulnerable developing countries.
 The fund aims to provide financial assistance to nations most vulnerable and impacted by the effects of
climate change. Hence sattement 1 is correct.

Q 44.A
 Recently, the Prime Minister launched the ₹24,000-crore Pradhan Mantri Particularly Vulnerable
Tribal Groups (PM PVTG) Mission aimed at the holistic development of around 28 lakh primitive
tribals spread across 220 districts across the country, on the occasion of tribal icon Birsa Munda’s
birth anniversary and the third “Janjatiya Gaurav Diwas.
 The mission is aimed to improve the socio-economic conditions of the 75 particularly vulnerable
tribal groups (PVTGs) by saturating PVTG families and habitations with basic facilities such as safe
housing, clean drinking water, and sanitation, improved access to education, health and nutrition, road and
telecom connectivity, and sustainable livelihood opportunities. Hence statement 1 is correct.
 PVTGs are characterized by a “pre-agriculture level of technology, stagnant or declining population,
extremely low literacy, and subsistence level of the economy. PVTGs were recognized as a separate
category based on the findings of the 1961 Dhebar Commission.

19 www.visionias.in ©Vision IAS


 To ensure coordination and effective implementation, The ministry has appointed 1 nodal officer
for each PVTG community and they are visiting their habitations to understand their
requirements. Hence statement 2 is not correct.
 The Ministry of Tribal Affairs is the Nodal Ministry for overall policy planning and coordination of
programs for the development of STs. Hence statement 3 is not correct.
 Hence option (a) is the correct answer.

Q 45.A
 Central Pollution Control Board is executing a nationwide programme of ambient air quality monitoring
known as the National Air Quality Monitoring Programme (NAMP).
 The network consists of 804 operating stations covering 344 cities/towns in 28 states and 6 Union
Territories of the country.
 The objectives of the N.A.M.P. are to determine the status and trends of ambient air quality;
o to ascertain whether the prescribed ambient air quality standards are violated;
o to Identify Non-attainment Cities;
o to obtain the knowledge and understanding necessary for developing preventive and corrective
measures and
o to understand the natural cleansing process undergoing in the environment through pollution dilution,
dispersion, wind-based movement, dry deposition, precipitation and chemical transformation of
pollutants generated.
 Under N.A.M.P., four air pollutants viz ., Sulphur Dioxide (SO2), Oxides of Nitrogen as NO2,
Respirable Suspended Particulate Matter (RSPM / PM10) and Fine Particulate Matter
(PM2.5) have been identified for regular monitoring at all the locations.
 Monitoring meteorological parameters such as wind speed and wind direction, relative humidity
(RH) and temperature were also integrated with monitoring air quality.
 Hence, option (a) is the correct answer.

Q 46.D
 Climate finance is one of the key agenda items up for discussion at the much-awaited 28th Conference of
Parties (COP28) to the United Nations Framework Convention on Climate Change to be held in Dubai
later this month.
 Climate Policy Initiative’s (CPI) new report Global Landscape of Climate Finance 2023 brings
together the latest data and analysis in this regard.
 The report breaks down the flow of climate finance by its application, geographical distribution and
sources, using data from 2021 and 2022. The flows tracked in the report “represent targeted climate
mitigation and adaptation-specific project-level allocation of capital”
 The Climate Policy Initiative: It is an independent non-profit research group and international
climate policy organization based in San Francisco, California with other offices worldwide. CPI is
supported primarily by philanthropic organizations and government development finance.

Q 47.B
 Recent context: The Government of India has given the approval for "Acceptance of Necessity" for
Project Kusha recently.
 Project Kusha focuses on the development of Long-Range Surface-to-Air Missiles (LR-SAM) and
ultimately an air defense system for India. Hence statement 1 is not correct.
 It is being jointly developed with Israel Aerospace Industries, Israel’s major aerospace and aviation
manufacturers. Hence statement 2 is correct.
 The mobile LR-SAM, with its long-range surveillance and fire control radars, would have different types
of interceptor missiles designed to hit hostile targets at 150 km, 250 km, and 350 km ranges.
 LR-SAM isn’t the only project India is developing with Israel. India’s Kalyani Strategic Systems has
already entered a joint venture with Israel’s Rafael Advanced Systems Ltd for indigenous production of
Medium-Range Surface to Air Missile (MRSAM).
Q 48.B
Coal Sector in India:
 India has the fifth largest coal reserves, is second in coal production (after China) and is among the
largest importer of coal (mainly from Indonesia, South Africa and Australia). Hence statement 1 is
correct.
20 www.visionias.in ©Vision IAS
 Coal accounts for almost 50% share in the total installed electricity generation capacity in India.
 Indian coal has high ash content, high ash fusion temperature but low sulphur content. Hence
statement 3 is not correct.
Distribution of Coal in India:
 Gondwana Coal Fields (250 million years old): Gondwana coal makes up to 98 % of the total
reserves and 99 % of the production of coal in India. Hence statement 2 is correct.
 Gondwana coal forms India’s metallurgical grade as well as superior quality coal.
 It is found in Damodar (Jharkhand-West Bengal), Mahanadi (Chhattisgarh-Odisha), Godavari
(Maharashtra), and Narmada valleys.
 Tertiary Coal Fields (15 – 60 million years old): Carbon content is very low but is rich in moisture
and Sulphur.
 Tertiary coalfields are mainly confined to extra-peninsular regions Important areas include Assam,
Meghalaya, Nagaland, Arunachal Pradesh, Jammu and Kashmir, Himalayan foothills of Darjeeling in
West Bengal, Rajasthan, Uttar Pradesh, and Kerala.

Q 49.D
 Tuberculosis, often called TB, is a bacterial infection that primarily affects the lungs. It's caused
by Mycobacterium tuberculosis and usually spreads through the air when an infected person coughs or
sneezes. Symptoms include coughing, chest pain, fatigue, weight loss, and fever.
 Mycobacterium tuberculosis is the primary agent responsible for TB and not Staphylococcus
aureus. Hence statement 1 is not correct.
 The bacteria usually attack the lungs, but TB bacteria can attack any part of the body such as the
kidney, spine, and brain. Not everyone infected with TB bacteria becomes sick. As a result, two TB-
related conditions exist: latent TB infection (LTBI) and TB disease. Hence statement 2 is not correct.
 The Mantoux tuberculin skin test (TST) is one method of determining whether a person is infected
with Mycobacterium tuberculosis, and not to monitor sugar level. Hence statement 3 is not correct.
 As per the global TB Report 2023, the Reduction of TB incidence has almost doubled the pace of the
Decline in global TB Incidence, Improvement in treatment coverage to 80% of the estimated TB cases;
an increase of 19% over the previous year, In India TB incidence declined by 16% and TB mortality
reduction by 18% since 2015.

Q 50.B
 With a population of 1.3 billion, India has a massive demand for energy to fuel its rapidly growing
economy. From a power deficit nation at the time of Independence, the efforts to make India energy-
independent have continued for over seven decades. Today, we are a power surplus nation with a total
installed electricity capacity of over four lakh MW.
 Keeping in mind the sustainable development goals, India's power generation mix is rapidly shifting
towards a more significant share of renewable energy. Today, India is the world's third largest
producer of renewable energy, with 40% of its installed electricity capacity coming from non-fossil
fuel sources.
 India has an estimated renewable energy potential of about 900 GW from commercially exploitable
sources, viz., wind - 102 GW (at 80-metre mast height); small hydro - 20 GW; bioenergy - 25 GW;
and 750 GW solar power, assuming 3 percent wasteland is made available. Hence, option (b) is the
correct answer.

Q 51.B
India’s Solar Photovoltaic (PV) Capacity:
 As of the latest data available from Ministry of New & Renewable Energy (MNRE), India’s solar
photovoltaic (PV) capacity has surged to an impressive 71,780.74 MW (71.78 GW) and is is fourth
largest in the world. Hence statement 1 is not correct.
 This capacity is distributed across various segments of the solar energy sector, with rooftop solar,
ground-mounted solar, hybrid projects, and off-grid solar playing significant roles in this energy
revolution.
 Ground-mounted solar plants has the largest share followed by roof-top and hybrid projects
respectively. Hence statement 2 is correct.

21 www.visionias.in ©Vision IAS


Q 52.C
 The World Heritage Outlook Report-IUCN:
o The World Heritage Outlook Report is the flagship assessment of the status of natural World Heritage
sites by the International Union for Conservation of Nature (IUCN). The report is produced every 3
years and provides an overview of the conservation status of all natural World Heritage sites, as well
as trends and challenges. Hence option (c) is the correct answer.
 The latest World Heritage Outlook Report, 2020:
o It found that the overall outlook for natural World Heritage sites is of concern, with only 63% of sites
assessed as being in good or good with some concerns condition. The remaining 37% of sites were
assessed as being of significant concern or critical condition.
o The report identified a number of factors that are threatening the status of natural World Heritage
sites, including:
 Climate change
 Habitat loss and fragmentation
 Invasive alien species
 Pollution
 Unsustainable tourism
 Overexploitation of natural resources
 The report also found that there has been some progress in conserving natural World Heritage sites in
recent years. For example, the number of sites that are well-managed has increased from 25% in 2000 to
38% in 2020. However, the report warned that this progress is not enough to keep pace with the threats
that natural World Heritage sites face.

Q 53.C
 The Perform Achieve and Trade Scheme is one of the initiatives under the National Mission on Enhanced
Energy Efficiency (NMEEE). Energy Saving Certificate (ESCert) under the Perform, Trade and
Achieve (PAT) scheme is a market-based mechanism to reduce the specific energy consumption (SEC)
in energy-intensive large industries. Hence statements 1 and 3 are correct.
 This mechanism is facilitated through the trading of Energy Saving Certificates (ESCerts) which are
issued to those plants that have overachieved their targets. Those plants which underachieve their targets
are entitled to purchase ESCerts.
 As per PAT rules, when a designated consumer overachieves the notified SEC targets in the compliance
year, the ESCerts are to be issued by Central Government for the difference of quantity between notified
target and the achieved SEC.
 ESCerts are traded on Indian Energy Exchange (IEX) and Power Exchange India Limited (PXIL).
ESCerts trading is regulated by the Central Electricity Regulatory Commission (CERC) with the Bureau
of Energy Efficiency (BEE), Ministry of Power as the Administrator. Hence statement 2 is not correct.

Q 54.C
 Air pollution control devices (APCD) are a series of devices which are used to prevent a variety of
different pollutants, both gaseous and solid, from entering the atmosphere mainly out of the industrial
stacks.
22 www.visionias.in ©Vision IAS
 These control devices can be separated into two broad categories namely:
o devices which control the amount of particulate matter escaping into the environment, and
o devices which control the acidic gas emissions into the atmosphere.
 The APCDs collect particulate emissions work on the principle of (i) gravity separation, (ii) cyclonic
separation, (iii) filtration, (iv) electrostatic precipitation, and (v) wet scrubbing.
 The most commonly used APCDs are:
o dust catchers
o cyclones or multi-cyclones,
o fabric filters also known as bag houses,
o electrostatic precipitators (wet and dry types), and
o scrubbers.
 The first four control devices control the amount of particulate matter escaping into the environment while
scrubbers control the acidic gas emissions into the atmosphere.
 Hence, option (c) is the correct answer.

Q 55.A
 The International Tropical Timber Organization (ITTO):
o The International Tropical Timber Organization (ITTO) is an intergovernmental organization that
promotes the sustainable management and conservation of tropical forests and the expansion and
diversification of international trade in tropical timber from sustainably managed and legally
harvested forests. Hence statement 1 is not correct.
o It was established in 1986 under the International Tropical Timber Agreement (ITTA), which
was sponsored by the United Nations Conference on Trade and Development (UNCTAD) and ratified
in 1985.
 The main objectives of the ITTO are to:
o Develop internationally agreed policy guidelines and norms to encourage sustainable forest
management (SFM) and sustainable tropical timber industries and trade.
o Assist tropical member countries in adapting such guidelines and norms to local circumstances and to
implement them in the field through projects and other activities.
o Collect, analyze, and disseminate data on the production and trade of tropical timber.

 Members:
ITTO divides its members into two categories - Producer Countries and Consumer Countries.
 Producer Countries:
o Producer countries are countries that are located between the Tropic of Cancer and the Tropic of
Capricorn, have tropical forest resources, and/or are net exporters of tropical timber in volume terms.
These countries are typically developing countries that rely on the export of tropical timber products
for a significant portion of their income.
o There are 37 Producer Countries which include Bolivia, Nigeria, India, etc. Hence statement 3 is
correct.
 Consumer Countries:
o Consumer countries are countries that are located outside of the tropics and generally import more
primary tropical wood products than they export. These countries are typically developed countries
that have a high demand for tropical timber products.
o There are 38 Consumer countries including Albania, Australia, China, etc.
The ITTO is open to all countries, regardless of whether they have tropical forests or engage in
the timber trade. As of today, the ITTO has 156 member countries, including developed and
developing countries with and without tropical forests. Hence statement 2 is not correct.
 Structure of the ITTO
o The ITTO is governed by the International Tropical Timber Council (ITTC), which is composed of
representatives from all ITTO member countries. The ITTC meets every two years to deliberate
and adopt decisions on environmental matters.
Q 56.A
 The Graded Response Action Plan is a set of anti-air pollution measures, generally implemented in
Delhi-NCR during the winter. Hence, statement 1 is not correct.
 The Commission for Air Quality Management (CAQM), an autonomous body tasked with
improving the air quality in Delhi and its adjoining areas, made crucial changes to the Graded
Response Action Plan (GRAP) last year and again in July. Hence, statement 2 is correct.
23 www.visionias.in ©Vision IAS
 GRAP for the NCR is implemented in four different stages based on the stages of adverse air quality in
Delhi. Stage 1 - 'poor' (AQI 201-300); Stage 2 - 'very poor' (AQI 301-400); Stage 3 - 'severe' (AQI 401-
450); and Stage 4 - 'severe plus' (AQI >450).
 Under the new plan, BS-III petrol and BS-IV diesel four-wheelers will be immediately banned in Delhi
and Gurugram, Faridabad, Ghaziabad and Gautam Buddh Nagar if the AQI crosses the 400-mark.
 The CAQM, a statutory body formed under an Act in 2021, amended the GRAP in 2022 to ensure
proactive implementation of anti-air pollution steps based on forecasts up to three days in advance.
 Earlier, the authorities would implement these measures, including a ban on construction and demolition,
entry of high-emission vehicles and the use of coal and firewood, only after the pollution level touched a
particular threshold.
 It also calls for a complete ban on the use of coal and firewood in eateries, restaurants and hotels as
soon as the AQI crosses the 200 mark. Earlier, the authorities would implement this measure under
Stage 2. Hence, statement 3 is not correct.New steps suggested as part of the revised GRAP include
"strict action to curb air pollution at all identified hotspots in the region under Stage 2".
 Under Stage 3, states will have to impose strict restrictions on the plying of BS-III petrol and BS-IV diesel
four-wheelers in Delhi and Gurugram, Faridabad, Ghaziabad and Gautam Buddh Nagar. The authorities
may discontinue physical classes in schools for children up to Class 5 in such a scenario, the CAQM
suggested.
 At AQI above 450, four-wheelers registered outside Delhi, other than electric vehicles and those using
CNG and BS-VI diesel, will not be permitted to enter Delhi. However, those carrying essential
commodities or providing essential services are exempt.
 All other existing measures taken under different stages of GRAP will continue.

Q 57.C
 Champions of the Earth award honour individuals, groups, and organizations whose actions have
transformative impact on environment. Hence statement 2 is correct.
 Awarded annually since its inception in 2005, it is UN’s highest environmental honour which
recognizes outstanding leaders from government, civil society and private sector. Hence statement 1
is correct.
 To eliminate plastic pollution, 2023 award recognized in four categories. Hence statement 3 is
correct.
o Policy leadership category: Mayor Josefina Belmonte of Quezon City (Philippines) to ban single-use
plastics, a trade-in programme for plastic pollution etc.
o Inspiration and Action category: Ellen MacArthur Foundation (United Kingdom) in mainstreaming
lifecycle approach, including for plastics.
o Entrepreneurial Vision category:
 Blue Circle (China) uses blockchain technology and internet of things to track and monitor full
lifecycle of plastic pollution.
 José Manuel Moller (Chile) is founder social enterprise (Algramo) dedicated to providing refill
services that reduce plastic pollution and lower everyday essentials costs.
o Science and Innovation category: Council for Scientific and Industrial Research (South Africa) uses
cutting-edge technology and multidisciplinary research to develop innovations to tackle plastic
pollution.

Q 58.C
 Recent context: Six Indian restaurants from around the world serving desi cuisine have been chosen for
this year’s Annapurna Certificate Programme.
 It is an initiative by the Indian Council for Cultural Relations (ICCR) to recognize Indian
restaurants abroad that are contributing to the country’s cultural cause in the culinary way. Hence
both statements 1 and 2 are correct.
 Those who were shortlisted from a total of 115 nominations received from Indian missions abroad are
Balaji Dosa, Kandy, Sri Lanka; Indian Street Food & Co., Stockholm, Sweden; Amber India Restaurant,
San Francisco, USA; Naans and Curries, An Ethnic Indian Restaurant, San Jose, Costa Rica; Mumtaz
Mahal Restaurant, Muscat, Oman; and Namaste Indian Restaurant, Ulaanbaatar, Mongolia.
 The Indian Council for Cultural Relations, is an autonomous organization of the Government of
India, involved in India's global cultural relations, through cultural exchange with other countries
and their people.

24 www.visionias.in ©Vision IAS


o The Indian Council for Cultural Relations (ICCR) was founded in 1950 by Maulana Abul Kalam
Azad, independent India’s first Education Minister.
o Its objectives are to actively participate in the formulation and implementation of policies and
programs about India’s external cultural relations
o To foster and strengthen cultural relations and mutual understanding between India and other
countries; to promote cultural exchanges with other countries and people, and to develop relations
with nations.

Q 59.B
 BHARAT TAP initiative and Nirmal Jal Prayas initiative are under the Ministry of Housing and
Urban Affairs.
 BHARAT TAP initiative:
o Focus: Promotes low-flow fixtures and sanitaryware to reduce water usage.
o Goal: Aim to achieve a minimum 40% reduction in water consumption through taps, showers, and
other fittings.
o Implementation: Encourages the use of Bureau of Indian Standards (BIS) certified water-efficient
plumbing fixtures.
o Benefits: Reduced water usage, lower water bills, lessened strain on water resources, and potentially
decreased energy consumption for water treatment and pumping.
o Launched by: Ministry of Housing and Urban Affairs (MoHUA) in collaboration with the Indian
Plumbing Association (IPA).
 Nirmal Jal Prayas:
o Focus: Tackles wastewater management and promotes sustainable water reuse.
o Goal: Aim to save 500 crore litres of water per year through wastewater treatment and reuse.
o Implementation: Supports development and adoption of wastewater treatment technologies,
encourages rainwater harvesting, and promotes public awareness campaigns.
o Benefits: Reduced reliance on freshwater sources, improved sanitation and hygiene, potential for
agricultural irrigation or industrial use of treated wastewater.
o Launched by: National Real Estate Development Council (NAREDCO) through its MAHI (Municipal
Authorities Helping Infrastructure) initiative.
 The National Real Estate Development Council(NAREDCO) was established as an autonomous self-
regulatory body in 1998 under the aegis of the Ministry of Housing and Urban Affairs.
 Hence, option (b) is the correct answer.

Q 60.D
 About Biofuels
o Refers to liquid transportation fuels, such as ethanol and biodiesel, derived from agricultural produce,
forests or any other organic material (feedstock).
o Based on the feedstock (raw materials) used to produce biofuels they are classified into four
generations.
o Presently, first-generation biofuels are the main source of biofuels globally despite 140 billion
tonnes of agro-waste or biomass generated from agriculture every year.
o First Generation Biofuels: These are made from food sources such as sugar, starch, vegetable
oil, or animal fats using conventional technology. They include Bioalcohols, Biodiesel, Vegetable
oil, Bioethers, Biogas. Hence pair 1 is correctly matched.
o Second Generation Biofuels: These are produced from non-food crops or portions of food crops
that are not edible and considered as wastes, e.g., stems, husks, wood chips, and fruit skins and
peeling. Hence pair 2 is correctly matched.
o Third Generation Biofuels: These are produced from micro-organisms like algae. Hence pair 3
is correctly matched.
o Fourth Generation Biofuels: In the production of these fuels, crops that are genetically engineered
to take in high amounts of carbon are grown and harvested as biomass. The crops are then
converted into fuel using second-generation techniques. Hence pair 4 is correctly matched.
o The fuel is pre-combusted, and the carbon is captured. Then the carbon is geo-sequestered, meaning
that the carbon is stored in depleted oil or gas fields or in unmineable coal seams.

25 www.visionias.in ©Vision IAS


Q 61.D
 The State of the World's Forests Report: The State of the World's Forests Report, is a flagship
publication of the Food and Agriculture Organization of the United Nations (FAO). It is published
every two years and provides a comprehensive overview of the state of the world's forests. The recent
publication is released in 2022 which focuses on the potential of forests to contribute to green recovery
and a transition to sustainable economies. Hence option (d) is the correct answer.
 Key Findings
 The 2022 edition of The State of the World's Forests found that:
o Deforestation and forest degradation are continuing at an alarming rate.
o Forests are facing a number of other threats, including climate change, wildfires, and invasive species.
o Despite these threats, forests still provide a wide range of benefits to people and the planet.
o Forests can play a key role in green recovery and a transition to sustainable economies.
 Recommendations
o The report makes a number of recommendations for how to protect and sustainably manage forests,
including:
 Halting deforestation and restoring degraded lands
 Investing in sustainable forest management
 Promoting forest-based livelihoods
 Strengthening international cooperation on forest issues

Q 62.D
 The Bharat Stage emission standards are standards instituted by the government to regulate the output
of air pollutants from motor vehicles from internal combustion engine equipment, including motor
vehicles. India has been following European (Euro) emission norms, although with a time lag of five
years.
 In 2016, the government said India would directly progress from BS-IV norms to BS-VI, skipping the
intermediary stage of BS-V. In April 2020, India moved towards more stringent BS-VI norms. Hence
statement 1 is not correct.
 The main difference in standards between the existing BS-IV and the new BS-VI auto fuel norms is the
presence of sulphur. The BS-VI fuel is estimated to bring around an 80 per cent reduction of sulphur,
from 50 parts per million to 10 ppm. Hence statement 3 is not correct.
 While the difference in emission levels is not that drastic for petrol engines, it is significant for diesel
vehicles. For example, the nitrous oxide emission levels (responsible for acid rain) will drop by 25
percent in the case of petrol vehicles and 68 percent for diesel vehicles. Hence getting a BS-IV diesel
engine to comply with BS-VI emission norms requires major changes to the hardware and layout of a
diesel engine compared to a petrol engine. That is why many car manufacturers in India are already
rolling out BS-VI compliant petrol models, but holding back on diesel models. Hence statement 2 is not
correct.
 The new technologies needed to be included in the vehicles include Diesel Particulate filter to expel
particulate matters, Selective catalytic reduction and exhaust gas regulator to reduce NOx emissions.

26 www.visionias.in ©Vision IAS


Q 63.B
 Short-lived climate pollutants are powerful climate forcers that remain in the atmosphere for a
much shorter period of time than carbon dioxide, yet their potential to warm the atmosphere can be
many times greater. Hence, statement 1 is not correct.
 Certain short-lived climate pollutants are also air pollutants that have harmful effects on people,
ecosystems and agricultural productivity.
 The short-lived climate pollutants black carbon, methane, tropospheric ozone, and
hydrofluorocarbons (HFCs) are the most important contributors to anthropogenic global warming
after carbon dioxide, responsible for up to 45% ( according to CCAP a UN convened initiative) of
current global warming. Hence, statement 2 is correct.
 If no action to reduce emissions of these pollutants is taken in the coming decades, they are expected to
account for as much as half of the warming caused by human activity.
 In 2011 The United Nations Environment Programme (UNEP) and the World Meteorological
Organisation (WMO) found that readily available solutions targeting compounds known as short-lived
climate pollutants (SLCPs) would slow the rate of global warming much faster than action on carbon
dioxide alone. These solutions would also reduce air pollution, representing “win-win” results for the
climate, air quality, and human well-being within a relatively short timeframe.
 In 2012, the importance of seizing this opportunity to reduce SLCPs and achieve the associated climate
and development benefits led to action by the governments of Bangladesh, Canada, Ghana, Mexico,
Sweden, and the United States, along with UNEP. They jointly set out to treat short-lived climate
pollutants as an urgent and collective challenge. Together, they formed the Climate & Clean Air
Coalition (CCAC) – with its Secretariat and Trust Fund hosted by UNEP – to support fast action and
deliver benefits across climate, public health, energy efficiency, and food security.
 Since its founding, the CCAC has raised awareness of the need to act on these pollutants to rapidly reduce
the rate of warming in the near term, improved the science behind them, and carried out activities in
emitting sectors to prove the feasibility of available solutions.
 It is the only international body working to connect climate and clean air action. Hence, statement 3
is correct.

Q 64.D
 Incineration is a waste treatment process that involves the combustion of organic substances
contained in waste materials. Incineration and other high-temperature waste treatment systems are
described as "thermal treatment". Hence the correct option is (d)
 Incineration of waste materials converts the waste into ash, flue gas and heat. The ash is mostly formed by
the inorganic constituents of the waste and may take the form of solid lumps or particulates carried by the
flue gas. The flue gases must be cleaned of gaseous and particulate pollutants before they are dispersed
into the atmosphere. In some cases, the heat that is generated by incineration can be used to generate
electric power.
 As of July 2017, thermal-based waste-to-energy (WTE) plants in India have the capacity to process
5,300 tonnes of garbage and produce 63.5 MW per day.
 According to a 2015-16 report by the Ministry of New and Renewable Energy (MNRE), this capacity
can be enhanced to 1,075 MW by 2031 and to 2,780 MW by 2050.
 The new policy seeks to increase the energy generated from solid waste to 330 MW per day in the next
five months, and over 400 percent increase from the current installed capacity, and to 511 MW by 2018-
19. To this end, five WTE plants are already under construction in the country and tenders have been
floated to build another 47.

27 www.visionias.in ©Vision IAS


Q 65.A
 Recent context: Lebanon accuses Israel of white phosphorus attacks.
 White phosphorus is a chemical waxy solid substance typically appearing yellowish or colorless, and
some have described its odor as resembling garlic.
o It ignites instantly upon contact with air. Hence statement 1 is correct.
o It is often used by militaries to illuminate battlefields, generate a smokescreen, and as an incendiary.
o Once ignited, white phosphorus is very difficult to extinguish. It sticks to surfaces like skin and
clothing.
o White phosphorus is harmful by all routes of exposure.
o White phosphorus can cause deep and severe burns, penetrating even through bone, and has been
known to reignite after initial treatment. After exposure, the priority is to stop the burning process.
 Convention on Certain Conventional Weapons (CCW) imposes restrictions on the use of white
phosphorus, to safeguard civilians. It is not banned. Hence statement 2 is not correct.
 The United Nations Convention on Certain Conventional Weapons, concluded in Geneva on October 10,
1980, and entered into force in December 1983, seeks to prohibit or restrict the use of certain conventional
weapons that are considered excessively injurious or whose effects are indiscriminate.

Q 66.B
 The Fagradalsfjall system is related to the volcanic system. It is situated on the Reykjanes Peninsula
in Iceland, a region known for its remarkable geological and geothermal features. It is the “world’s
newest baby volcano.” It had been dormant for eight centuries before erupting in 2021, 2022 and
2023.
 The Reykjanes Peninsula is located in the southwestern part of Iceland and is known for its proximity
to the Mid-Atlantic Ridge, a tectonic boundary where the North American and Eurasian tectonic
plates meet. This geological setting makes the Reykjanes Peninsula a hub of dynamic volcanic and
geothermal activity.
 The eruption marked the first time in over 800 years that a volcanic event had occurred in this particular
area. The eruption is characterized by its effusive style, where molten lava flows out relatively calmly
compared to more explosive volcanic eruptions.
 Deep under the Earth’s surface, intense heat melts rocks to form magma, a thick flowing substance
lighter than solid rock. This drives it upwards and most of it gets trapped in magma chambers deep
underground. Over time, this viscous liquid cools and solidifies once again. However, a tiny fraction
erupts through vents and fissures on the surface, causing volcanic eruptions. The movement of magma
close to the Earth’s surface exerts a force on the surrounding rock, which often causes earthquake
swarms.
 Hence option (b) is the correct answer.

Q 67.B
 Background
o India is grappling with a severe air pollution crisis, with many cities consistently exceeding safe limits
for particulate matter (PM) and other pollutants. In response to this growing concern, the
Government of India launched the National Clean Air Programme (NCAP) in 2019.
 What are the Target Levels?
o The NCAP initially set a target of reducing key air pollutants PM10 and PM2.5 by 20-30% in 2024,
taking the pollution levels in 2017 as the base year to improve upon.
o However, in September 2022, the Centre moved the goalposts and set a new target of a 40% reduction
in particulate matter concentration, by 2026. Hence Statement 1 is not correct.
 Objectives
o The NCAP is a comprehensive national-level program aimed at addressing air pollution across the
country. Its primary objectives are:
o Reduce Particulate Matter (PM) Concentrations: The NCAP aims to reduce PM concentrations by
20-30% in cities across India by 2024, and by 50% in non-attainment cities by the same year.
o Improve Air Quality: The NCAP aims to improve air quality in all cities to meet the National
Ambient Air Quality Standards (NAAQS) by 2024.
o Strengthen Air Quality Monitoring: The NCAP aims to strengthen air quality monitoring
infrastructure and data management across India.
 Key Components
The NCAP encompasses a range of measures to achieve its objectives, including:
28 www.visionias.in ©Vision IAS
o Strengthening Emission Standards: The NCAP proposes stricter emission standards for vehicles and
industries to reduce their contribution to air pollution.
o Promoting Cleaner Fuels and Technologies: The NCAP encourages the adoption of cleaner fuels,
such as compressed natural gas (CNG) and electric vehicles, and promotes the use of energy-efficient
technologies.
o Improving Urban Planning and Transportation Systems: The NCAP emphasizes the need for
better urban planning and transportation systems to reduce traffic congestion and associated
emissions.
o Strengthening Monitoring and Enforcement Mechanisms: The NCAP aims to strengthen
monitoring and enforcement mechanisms to ensure compliance with emission standards and
regulations.
 Implementation
The NCAP is being implemented by the Ministry of Environment, Forest and Climate Change
(MoEFCC) in collaboration with various state governments, municipal corporations, and industry
stakeholders. Hence statement 2 is correct.
o The program has a two-phase implementation plan:
 Phase 1 (2019-2024): Focus on reducing PM concentrations and improving air quality in non-
attainment cities.
 Phase 2 (2024-2028): Expand the scope of the program to cover all cities in India and achieve
further reductions in air pollution.

Q 68.B
 The Bank for International Settlements (BIS), in collaboration with the central banks of France,
Singapore, and Switzerland, has announced the successful conclusion of Project Mariana. Hence
statement 1 is not correct.
o Project Mariana was jointly developed by three BIS Innovation Hub centers, including the Swiss,
Singapore, and Eurosystem Hub Centers, in partnership with the Bank of France, the Monetary
Authority of Singapore, and the Swiss National Bank.
 This innovative project focused on testing the cross-border trading and settlement of wholesale
central bank digital currencies (wCBDCs) between financial institutions while incorporating
decentralized finance (DeFi) technology concepts on a public blockchain. Hence statement 2 is
correct.
 The project explored how multi-currency settlement may be performed atomically while maintaining the
independence of respective domestic settlement systems.
 The project's proof of concept achieved successful cross-border trading and settlement of hypothetical
euro, Singapore dollar, and Swiss franc WCBDCs between simulated financial institutions.
 Project Mariana's architectural design addresses the balance between central banks' need for domestic
oversight and autonomy and financial institutions' desire for efficient cross-border wCBDC management.
o This equilibrium is maintained through the use of a common token standard on a public blockchain,
facilitating interoperability and seamless exchange of wCBDCs across various local payment and
settlement systems managed by participating central banks.
o Consequently, Mariana offers possible strategies for incorporating an international dimension into
current wCBDC design explorations.

Q 69.C
 'SATAT’ (Sustainable Alternative Towards Affordable Transportation) scheme on Compressed Bio
Gas (CBG) was launched in 2018. The scheme envisages target production of 15 MMT (million tons)
of CBG 2023 from 5000 Plants. Independent entrepreneurs shall set up the CBG Plants. Hence,
statement 1 is correct.
 It encourages entrepreneurs to set up CBG plants, produce & supply CBG to Oil Marketing
Companies (OMCs) for sale as automotive & industrial fuels. Hence, statement 2 is correct.
 The SATAT initiative has the potential to boost the availability of more affordable transport fuels,
better use of agricultural residue, cattle dung and municipal solid waste, and provide additional revenue
sources to farmers. It will also benefit vehicle users as well as farmers andentrepreneurs. Hence,
statement 3 is correct.
 It will boost entrepreneurship, rural economy and employment and provide additional sources of
revenue to farmers. It will also help achievenation’s climate change goals and bring down dependency
on natural gas and crude oil imports and act as a buffer against crude oil and gasprice fluctuations.

29 www.visionias.in ©Vision IAS


 It will also help to integrate with existing networks such as city gas distribution (CGD) networks to
boost supplies to domestic and retail users in existing and upcoming markets. Besides retailing from OMC
fuel stations, CBS can at a later date be injected into CGD pipelines too forefficient distribution and
optimized access of cleaner and more affordable fuel.

Q 70.A
 The Invasive Species Specialist Group (ISSG) is a global network of scientific and policy experts on
invasive species, organized under the auspices of the Species Survival Commission (SSC) of the
International Union for Conservation of Nature (IUCN). Hence option (a) is the correct answer.
 The ISSG promotes and facilitates the exchange of invasive species information and knowledge across the
globe and ensures the linkage between knowledge, practice and policy so that decision making is
informed. The two core activity areas of the ISSG are policy and technical advice, and information
exchange through our online resources and tools and through networking.

Q 71.D
 Recent context: National Organ and Tissue Transplant Organisation data shows four out of five organ
recipients in the country are men highlighting gender disparity.
 National Organ and Tissue Transplant Organization (NOTTO) is a National level organization set
up under the Directorate General of Health Services, Ministry of Health and Family Welfare. It has
the following two divisions :
o National Human Organ and Tissue Removal and Storage Network
o National Biomaterial Centre
 Hence statement 1 is not correct.
 National Organ and Tissue Transplant Organization functions as an apex center for All India
activities of coordination and networking not just for procurement but the distribution of Organs
and Tissues and registry of Organs and Tissues Donation and Transplantation in the
country. Hence statement 2 is not correct.
o The following activities would be undertaken to facilitate Organ Transplantation most safely in the
shortest possible time and to collect data to develop and publish the National registry:
 Lay down policy guidelines and protocols for various functions.
 Network with similar regional and state-level organizations.
 All registry data from States and Regions would be compiled and published.
 Creating awareness, and promotion of organ donation and transplantation activities.
 Co-ordination from procurement of organs and tissues to transplantation when the organ is
allocated outside the region.
 Dissemination of information to all concerned organizations, hospitals, and individuals.
 Monitoring of transplantation activities in the Regions and States and maintaining data in this
regard.
 To assist in data management for organ transplant surveillance & organ transplant and Organ
Donor registry.
 Consultancy support on the legal and non-legal aspects of donation and transplantation.
 Coordinate and Organize trainings for various cadre of workers.
Q 72.B
 Recent context: Recently a 5 judge bench of the Supreme Court has started hearing the petitioner's case
challenging the constitutional validity of electoral bonds.
 Electoral bonds are interest-free bearer bonds or money instruments that can be purchased by companies
and individuals in India from authorized branches of the State Bank of India (SBI).
o These bonds are sold in multiples of Rs 1,000, Rs 10,000, Rs 1 lakh, Rs 10 lakh, and Rs 1 crore. They
can be purchased through a KYC-compliant account to make donations to a political party. The
political parties have to encash them within a stipulated time.
30 www.visionias.in ©Vision IAS
 The name and other information of the donor are not entered on the instrument and thus electoral bonds
are said to be anonymous. There is no cap on the number of electoral bonds that a person or
company can purchase.
 Political parties that secured at least 1% of the votes polled in the recent Lok Sabha or State Assembly
elections and are registered under the RPA can get a verified account from the Election Commission of
India (ECI). The bond amounts are deposited in this account within 15 days of their purchase.
o The political party has to encash the amount within those 15 days, and the amount received as a
donation gets deposited into the Prime Minister's Relief Fund. These bonds, however, are not
available for purchase all the time.
 In India, there are no donation limits on individuals. Moreover, the Finance Act, of 2017 also removed
any official contribution limits on companies. In other words, an individual or a company can donate
as much as they want to a political party. Hence statement 1 is correct and statement 2 is not
correct.
 Similarly, there is no legal expenditure limit on expenditure by political parties. A party can spend
as much as it wants for its national or state-level campaign as long as it does not spend that money
on the election of any specific candidate. Hence statement 3 is correct.
 However, parties are required to disclose donations of more than Rs 20,000, unless they are made through
electoral bonds.
o Parties are not required to disclose the sum or the source of any single donation that is below Rs
20,000.
o This is where the legal loophole steps in — parties generally break large donations from a single
donor into multiple small donations. This practice exempts them from any disclosure requirement.
Q 73.A
 India’s updated Nationally Determined Contribution (NDC) the United Nations Framework
Convention on Climate Change (UNFCCC).
o It would protect the interests of the country and safeguard its future development needs based on the
principles and provisions of the UNFCCC.
 India at the 26th session of the Conference of the Parties (COP26) to the UNFCCC held in
Glasgow, United Kingdom, expressed to intensify its climate action by presenting to the world five
nectar elements (Panchamrit) of India’s climate action.
 This update to India’s existing NDC translates the ‘Panchamrit’ announced at COP 26 into enhanced
climate targets.
 As per the updated NDC, India now stands committed to reduce Emissions Intensity of its GDP by
45 percent by 2030, from 2005 level. Hence statement 1 is not correct.
 India aims to achieve about 50 percent cumulative electric power installed capacity from non-fossil
fuel-based energy resources by 2030. Hence statement 2 is not correct.
 It targets net zero emissions by 2070 and it does not bind it to any sector specific mitigation
obligation or action. Hence statement 3 is correct.
 India’s goal is to reduce overall emission intensity and improve energy efficiency of its economy over
time and at the same time protecting the vulnerable sectors of economy and segments of our society.
Q 74.A
 Just Energy Transition Partnerships (JETP) aim to bridge the gap between developed and
developing nations in moving towards clean energy. Essentially, JETP is a financing mechanism. In
a Partnership, wealthier nations fund a coal-dependent developing nation to support the country’s
own path to phase out coal and transition towards clean energy while addressing the social
consequences. Hence, statement 1 is correct.
 JETP funding can go through grants, loans, or investments. As of March 2023, the donor pool includes the
International Partners Group (IPG) and the Glasgow Financial Alliance for Net Zero (GFANZ) Working
Group. The IPG consists of Japan, the USA, Canada, Denmark, France, Germany, Italy, Norway, the EU,
and the UK. The GFANZ Working Group comprises multilateral and national development banks and
finance agencies such as HSBC and Citi Bank.
 The first Just Energy Transition Partnership was with South Africa and was announced at COP 26
Glasgow in November 2021. The funders, five of the current IPG members, pledged 8.5 billion USD
in the first financing round. A year later, at COP 27 Sharm El Sheikh, South Africa published its
JETP Implementation Plan (JETP IP). This JETP is expected to prevent up to 1-1.5 gigatons of
emissions from the atmosphere over the next 20 years. Hence, statement 2 is not correct.
 The second Partnership was announced at the G20 Bali Summit in November 2022. Indonesia is set to
receive an initial 20 billion USD in public and private financing over the next three to five years. The
31 www.visionias.in ©Vision IAS
donors will assist via grants, concessional loans, market-rate loans, guarantees, private investments, and
technical assistance. In February 2023, Indonesia launched the Secretariat for the Just Energy
Transition Partnership.
 The next one was Vietnam. The Vietnam JETP was announced in December 2022 after a lengthy
negotiation process. This Partnership will assist Vietnam in finance, technology, and capacity building. It
will also support the country’s policy and regulation improvement to increase private investment in
renewable energy. Hence, statement 3 is not correct.

Q 75.C
 The Convention on the Conservation of Migratory Species of Wild Animals (Bonn Convention; CMS) is
an environmental treaty under the aegis of the United Nations Environment Programme. Hence,
statement 2 is correct.
 It provides a global platform for the conservation and sustainable use of migratory animals and their
habitats. CMS is the only global and UN-based intergovernmental organisation established
exclusively for the conservation and management of terrestrial, aquatic and avian migratory species
throughout their range. Hence, statement 1 is correct.
 Migratory species threatened with extinction are listed on Appendix I of the Convention. CMS Parties
strive towards strictly protecting these animals, conserving or restoring the places where they live,
mitigating obstacles to migration and controlling other factors that might endanger them. Besides
establishing obligations for each State joining the Convention, CMS promotes concerted action among the
Range States of many of these species.
 Migratory species that need or would significantly benefit from international co-operation are listed in
Appendix II of the Convention. For this reason, the Convention encourages the Range States to conclude
global or regional agreements.

Q 76.A
 Recent Context: Indian Army successfully test-fired rocket and turret guns of indigenous Light Combat
Helicopter (LCH) Prachand (means fierce).
o It was inducted into Indian Air Force in 2022.
o LCH Prachand is a multi-role combat helicopter, designed and developed by Hindustan
Aeronautics Ltd.
o Only attack helicopter in the world that can land and take off at an altitude of 5,000 metres.
o Capable of firing air-to-ground and air-to-air missiles.
o Fitted with 5.8-tonne twin-engine named Shakti engine, primarily designed for deployment in high-
altitude areas (like Siachen glacier).
o It has best stealth features, armored-shield systems, and dark-mode attack capability.
 Hence option (a) is the correct answer.

Q 77.B
 The ‘’Vajra Mushti Kalaga’’ is a form of wrestling different from conventional grappling and entails
two jettys taking a swipe at each other’s head with a knuckleduster.
o The fight is real and the jettys make all efforts to draw blood from the opponent’s head and a referee
intervenes on noticing the first drop.
 Whosoever draws the blood from the opponent’s head first is declared the winner. Though this form of
wrestling was popular during the period of the Vijayanagar rulers who reigned between the 14th
and the 17th centuries, it has gone extinct and takes place only during Dasara. It is not associated with
the Vajrayana form of Buddhism.
 Medieval travelers from the Portuguese noticed this form of wrestling during the Navaratri celebrations in
the Vijayanagar empire and have left detailed accounts of it. Historically this form of martial art was
popular and has a hoary past but has been on a decline and has gone out of vogue in modern times.
 Hence option (b) is the correct answer.

Q 78.C
 Ministry of Environment, Forest and Climate Change (MoEFCC) launched National Clean Air
Programme (NCAP) in January, 2019 with an aim to improve air quality in 131 cities (non-
attainment cities and Million Plus Cities) in 24 States/UTs by engaging all stakeholders. The
programme envisages to achieve reductions up to 40% or achievement of National Ambient Air Quality
Standards for Particulate Matter10 (PM 10) concentrations by 2025-26.

32 www.visionias.in ©Vision IAS


 The NCAP targets to achieve 20% to 30% reduction in concentrations of PM10(particulate matter of
diameter between 10 and 2.5 micrometer) and PM2.5(particulate matter of diameter 2.5 micrometer or
less) by the year 2024, keeping 2017 as the base year for comparison of concentration.
 Non-attainment cities (NAC)- Cities are declared nonattainment if over a 5-year period they
consistently do not meet the National Ambient Air Quality Standards (NAAQS) for PM 10
(Particulate matter that is 10 microns or less in diameter) or N02 (Nitrogen Dioxide).
 Central Pollution Control Board (CPCB) identified 94 non-attainment cities (NAC) while circulating the
draft NCAP in 2018 that was revised to 102 NAC's in January 2019. Two more were added to the list in
2020, taking the total to 124. The list of cities under NCAP focus now stands at 131 cities in 2021 with
a million-plus population.
 Hence, option (c) is the correct answer.

Q 79.D
 Smog is air pollution that reduces visibility. The term "smog" was first used in the early 1900s to describe
a mix of smoke and fog. The smoke usually came from burning coal. Smog was common in industrial
areas and remains a familiar sight in some cities today.
 Today, most of the smog we see is photochemical smog. Photochemical smog is produced when sunlight
reacts with nitrogen oxides and at least one volatile organic compound (VOC) in the atmosphere. Nitrogen
oxides come from car exhaust, coal power plants, and factory emissions. VOCs are released from
gasoline, paints, and many cleaning solvents. When sunlight hits these chemicals, they form airborne
particles and ground-level ozone—or smog.
 Smog is still a problem in many places. Everyone can do their part to reduce smog by changing a few
behaviours, such as:
o Drive less. Walk, bike, carpool, and use public transportation whenever possible.
o Take care of cars. Getting regular tune-ups, changing oil when scheduled, and inflating tires to
the proper level can improve gas mileage and thus reduce emissions.
o Fuel up during the cooler hours of the day—night or early morning. This prevents gas fumes
from heating up and producing ozone.
o Avoid products that release high levels of VOCs. For example, use low-VOC paints.
o Avoid gas-powered yard equipment, like lawnmowers. Use electric appliances instead.
 Hence, option (d) is the correct answer.

Q 80.A
 The Mouvement Du 23 Mars (M23) is an armed group operating in the Democratic Republic of the
Congo (DRC). The prominent rebel groups in the DRC, besides M23 include the Allied Democratic
Forces (ADF) and the Cooperative for Development of the Congo (CODECO).
 Recently, the UN International Organization of Migration (IOM) reported that the number of people who
have been internally displaced in the Democratic Republic of Congo (DRC) has risen to 6.9 million. In
the eastern province of North Kivu, nearly a million people have been displaced due to the ongoing
conflict with the rebel group, Mouvement du 23 Mars (M23).
 The conflict in the DRC dates back to the 1990s when it went through two civil wars in 1996 and 1998.
The conflict erupted in the wake of the Rwandan genocide in 1994 where ethnic Hutu extremists killed
nearly one million minority ethnic Tutsis and non-extremist Hutus.

33 www.visionias.in ©Vision IAS


 The armed groups have been supported by the governments of Rwanda, Uganda, and Burundi at
various points, acting as proxies for each country’s interests in the region.
 Hence option (a) is the correct answer.

Q 81.C
 These Bio-Medical Waste (Management and Handling) Rules apply to all persons who generate, collect,
receive, store, transport, treat, dispose of, or handle bio-medical waste in any form including hospitals,
nursing homes, clinics, dispensaries, veterinary institutions, animal houses, pathological laboratories,
blood banks, and Ayush. These rules shall apply to all persons who generate, collect, receive, store,
transport, treat, hospitals, clinical establishments, research or educational institutions, health camps,
medical or surgical camps, vaccination camps, blood donation camps, first aid rooms of schools, forensic
laboratories and research labs.
 Only Pair 1 is not correctly matched.

 Hence, option (c) is the correct answer.


34 www.visionias.in ©Vision IAS
Q 82.B
 Recent context: The National Aeronautics and Space Administration (NASA) is set to launch the
Atmospheric Waves Experiment (AWE) to study one of the important drivers of Space weather – the
Earth’s weather.
 AWE is a first-of-its-kind NASA experimental attempt aimed at studying the interactions between
terrestrial and Space weather. Hence statement 1 is correct but statement 2 is not correct.
 It is planned under NASA’s Heliophysics Explorers Program, the $42 million mission will study the links
between how waves in the lower layers of the atmosphere impact the upper atmosphere, and thus, Space
weather.
 AWE will be launched and mounted on the exterior of the Earth-orbiting International Space
Station (ISS). From the vantage point, it will look down at the Earth and record the colorful light bands,
commonly known as airglow. Hence statement 3 is correct.
o AWE will measure the airglow at mesopause (about 85 to 87 km above the Earth’s surface), where
the atmospheric temperatures dip to minus 100 degrees Celsius.
o At this altitude, it is possible to capture the faint airglow in the infrared bandwidth, which appears the
brightest enabling easy detection.
 Onboard AWE is an Advanced Mesospheric Temperature Mapper (ATMT), an instrument that will scan
or map the mesopause

Q 83.A
 The crackdown on fossil fuel emissions continues, with the recent implementation of CAFE II norms
(April 2022).
 In short, more stringent emission regulations are designed to get car manufacturers to cut down on
their overall CO2 levels, chiefly through software and hardware-related upgrades and by shifting to
electrified or hybridized powertrains. Hence, statement 1 is not correct.
 CAFE stands for Corporate Average Fuel Efficiency/Economy, so it is not the emission and economy
of the individual models that are considered, but the weighted average of emissions and fuel economy
for the entire fleet sold in the country. Hence, statement 2 is correct.
 This means, that the manufacturer can very well sell heavy, highly polluting and fuel-efficient cars, but
then they will have to make that up by selling huge volumes of much more efficient and light cars.
 The Corporate Average Fuel Economy (CAFE) norms are primarily designed to increase fuel efficiency,
which, in turn, will lower a company’s overall CO2 footprint.
 At present, the existing limit for average CO2 emissions happens to be 130g/km. Hence, statement 4
is not correct.
 With CAFE II in effect, the target has been lowered to 113g/km. The key difference between CAFE
norms and BS6 II is that the former focuses on reducing all manner of harmful by-products from a car’s
exhaust (sulphur, nitrogen oxide, etc), while the latter focuses exclusively on CO2 emissions.
 However, both norms force manufacturers to lower fuel consumed by their vehicles, and simultaneously,
move towards electric mobility.
 These standards were released by the Union Ministry of Power (MoP) for the first time in 2017,
under the Energy Conservation Act, of 2001. They apply to petrol, diesel, LPG and CNG vehicles.
Hence, statement 3 is not correct.
Q 84.A
 Air (Prevention and Control of pollution) Act,1981
 The act Provides for the prevention, control and abatement of air pollution. It makes provisions, Interalia,
for Central and State Boards, power to declare pollution control areas, restrictions on certain industrial
units, the authority of the Boards to limit the emission of air pollutants, power of entry, inspection, taking
samples and analysis, penalties, offences by companies and Government and cognizance of offences etc.
 The Act specifically empowers the State Government to designate air pollution areas and to prescribe the
type of fuel to be used in these designated areas.
 Power to declare air pollution areas: Section 19
o The Act states that the State Government, after consulting the State Board, may declare an area within
the State as an air pollution area.
o The State Government may also order for the extension or reduction of an air pollution area or may
even merge one or more areas to make a new pollution area or any part or parts thereof.
o The State Government after consulting the State Board, may also by notification in the Official
Gazette, prohibit the use of any fuel or appliance that may cause or is likely to cause air
pollution. Hence, statement 1 is not correct.

35 www.visionias.in ©Vision IAS


o The State Government may also prohibit the burning of any material (which is not a fuel) if it causes
or is likely to cause air pollution. This is also done after consultations with the respective State Board.
 Powers and Functions
o Central Board- Section 16
 The main function of the Central Board shall be to improve the quality of air and to prevent,
control or abate air pollution in the country.
 It may:
 Advise the Central Government on any matter concerning the improvement of the quality of
air.
 Plan and cause to be executed a nation-wide programme for the prevention, control or
abatement of air pollution
 Co-ordinate the activity of the State and resolve disputes among them
 Provide technical assistance and guidance to the state boards, carry out and sponsor
investigations and research relating to problems of air pollution
 Organize through mass media a comprehensive programme regarding the prevention, control
or abatement of air pollution
 The Central Board may establish or recognize a laboratory to enable the Central Board
to perform its functions under this section efficiently. Hence, statement 3 is correct.
o State Board - Section 17
 To plan a comprehensive programme for the prevention, control or abatement of air pollution;
 Advise the State Government on any matter concerning the prevention of air pollution;
 To inspect, at all reasonable times, any control equipment, industrial plant or
manufacturing process, and to give by order such directions to such person as it may be
necessary to take steps for the prevention of air pollution. Hence, statement 2 is not correct.
 It shall collect and disseminate information regarding air pollution. It shall organise training and
mass awareness programmes regarding air pollution control, prevention and abatement.
 It shall lay down standards for the emission of air pollutants into the atmosphere from
automobiles or industries, or any other pollutant from any source. However, a ship or aircraft
cannot come into the ambit of a source.
 The State Boards shall also advise the State Government regarding the suitability of any location
which is to be used for setting up any industry, keeping in mind the air quality which would be
impacted if that industry is set up.
 The Boards shall also set up labs in their States, to enable the State Board to perform its functions
effectively.
 Hence, option (a) is the correct answer.

Q 85.C
Global Methane Pledge
 The Global Methane Pledge was launched at COP (Conference of Parties) 26 in November 2021 in
Glasgow, Scotland to catalyse action to reduce methane emissions and is led by the United States
and the European Union. Hence statement 2 is correct.
 It has 111 country participants who together are responsible for 45% of global human-caused methane
emissions.India, which is not a part of the Global Methane Pledge, is among the top five methane emitters
globally. Most emissions can be traced back to agriculture.
 By joining the Pledge, countries commit to work together in order to collectively reduce methane
emissions by at least 30% below 2020 levels by 2030. Hence statement 1 is correct.
 India did not sign the pledge because of its concerns over the impact on trade, on the country's vast farm
sector, and the role of livestock in the rural economy. Hence statement 3 is correct.

Q 86.C
 Environmental Impact Assessment (EIA):
o The UNEP defines Environmental Impact Assessment (EIA) as a tool used to identify the
environmental, social and economic impacts of a project prior to decision-making. It aims to
predict environmental impacts at an early stage in project planning and design, find ways and means
to reduce adverse impacts, shape projects to suit the local environment and present the predictions and
options to decision-makers.
o EIA comes under the Notification on Environmental Impact Assessment (EIA) of developmental
projects 1994 under the provisions of the Environment (Protection) Act, 1986.
36 www.visionias.in ©Vision IAS
o “Public Consultation” refers to the process by which the concerns of locally affected persons and
others who have a plausible stake in the environmental impact of the project or activity are
ascertained with a view to taking into account all the material concerns in the project or activity
design as appropriate.
 2022 Amendments in EIA rules
o Thermal power plants up to 15 MW based on biomass or non-hazardous municipal solid
waste using auxiliary fuel such as coal, lignite or petroleum products up to 15 per cent have also been
exempted — as long as the fuel mix is eco-friendly.
o Fish handling ports and harbours-Taking into account issues of livelihood security of fishermen
involved at fish handling ports and harbours, and the less pollution potential of these ports and
harbours compared to others, increasing the threshold of ports which exclusively deals in fish
handling, and cater to small fishermen, will be exempted from environmental clearance.
o Toll plazas that need more width for the installation of toll collection booths to cater to a large
number of vehicles are exempted.
o Airport-Expansion activities in existing airports related to terminal building expansion without an
increase in the airport’s existing area, rather than expansion of runways also exempted.
o Strategic Projects-For projects of strategic importance-“Highway projects related to defence and
strategic importance in border States are sensitive in nature and that needs to be executed on priority
keeping in view strategic, defence and security considerations.
o Exemption of such projects from the requirement of Environmental Clearance in border areas, subject
to specified Standard Operating Procedure along with standard environmental safeguards for such
projects for self-compliance by the agency executing such projects.Any project which is 100 km from
the Line of Control, among other locations, will be exempted from an environmental clearance before
construction.
 Hence option (c) is the correct answer.

Q 87.B
 Global Energy Transition Index is released by World Economic Forum (WEF).In the latest published
index, India has moved up two places to rank 76th out of 115 economies. Hence option (b) is the correct
answer.
 It ranks economies on how well they are able to balance energy security and access with environmental
sustainability and affordability. The WEF index considers both the current state of the countries’ energy
system and their structural readiness to adapt to future energy needs.
 The ‘transition readiness’ component of the index has taken into account six individual indicators:
o capital and investment
o regulation and political commitment
o institutions and governance
o institutions and innovative business environment
o human capital and consumer participation
o energy system structure
 India’s Performance- India is amongst the countries with high pollution levels and has a relatively high
CO2 intensity in its energy system. India has made significant strides to improve energy access in recent
years, and currently scores well in the area of regulation and political commitment towards energy
transition. However, India is the only amongst the five economies to improve its rank since last year.
 Global Rankings: Sweden remains on the top on this annual list and is followed by Switzerland and
Norway in the top three.

Q 88.D
 Composition, Generation and Utilization of Fly Ash in India
 Fly ash is a by-product of coal-based power plants.
 It is a fine powder with substantial amounts of oxides of silica, aluminium and calcium. It also
contains traces of Arsenic, Boron, Chromium, lead etc. which leads to air and water pollution if
disposed on land. Hence option (d) is the correct answer.
 With low grade of Indian coal, its ash content is as high as 30-45% in comparison to imported coal with
10- 15%.
 With nearly 55% of our total power production through coal and lignite based Thermal Power Plants
(TPP), the fly ash generation in India is very high.

37 www.visionias.in ©Vision IAS


Q 89.C
 Renewable diesel can be produced by several different technology pathways. Currently, commercial
production facilities are using the hydrotreating pathway and fats, oils, and greases are the most common
feedstocks. It is made from fats and oils, such as soybean oil or canola oil, and is processed to be
chemically the same as petroleum diesel. Hence, pair 2 is correctly matched.
 Renewable gasoline (also called green or drop-in gasoline) is a fuel produced from biomass sources
through a variety of biological, thermal, and chemical processes. The fuel is chemically identical to
petroleum gasoline. It can be produced from various biomass sources. These include lipids (such as
vegetable oils, animal fats, greases, and algae) and cellulosic material (such as crop residues, woody
biomass, and dedicated energy crops). Hence, pair 1 is correctly matched.
 Sustainable aviation fuel (SAF), made from non-petroleum feedstocks, is an alternative fuel that reduces
emissions from air transportation. SAF can be blended at different levels with limits of 10% to 50%,
depending on the feedstock and how the fuel is produced. SAF can be produced from non-petroleum-
based renewable feedstocks including, but not limited to, the food and yard waste portion of
municipal solid waste, woody biomass, fats/greases/oils, and other feedstocks. Hence, pair 3 is
correctly matched.

Q 90.C
 The Rotterdam Convention (formally, the Rotterdam Convention on the Prior Informed Consent
Procedure for Certain Hazardous Chemicals and Pesticides in International Trade) is a multilateral treaty
to promote shared responsibilities in relation to the importation of hazardous chemicals. Hence,
statement 1 is correct.
 The convention promotes an open exchange of information and calls on exporters of hazardous chemicals
to use proper labelling, include directions on safe handling, and inform purchasers of any known
restrictions or bans.
 Signatory nations can decide whether to allow or ban the importation of chemicals listed in the treaty, and
exporting countries are obliged to make sure that producers within their jurisdiction comply.
 The movements of hazardous chemicals listed in Annex III are subject to the Prior Informed
Consent procedure (“PIC procedure”). Hence, statement 2 is correct.
 Exports are only allowed if the State of import has consented to the future import of the specific
chemical through an Import Response. Hence, statement 3 is correct.
 To achieve its objectives the Convention includes two key provisions, namely the Prior Informed Consent
(PIC) Procedure and Information Exchange.
o The Prior Informed Consent (PIC) procedure – The PIC procedure is a mechanism for formally
obtaining and disseminating the decisions of importing Parties as to whether they wish to receive
future shipments of those chemicals listed in Annex III of the Convention and for ensuring
compliance with these decisions by exporting Parties.
o Information Exchange - The Convention facilitates information exchange among Parties for a very
broad range of potentially hazardous chemicals. The Convention requires each Party to notify the
Secretariat when taking a domestic regulatory action to ban or severely restrict a chemical.
 The Convention does not apply to chemicals imported for the purpose of research and analysis.
Hence, statement 4 is not correct.
 What is the maximum quantity of a chemical that can be imported for such purposes?
o The Convention does not specify an amount that is considered for the purpose of research or analysis.
Some Parties in implementing the Convention have set a level of 10 kilograms whereas others have
set lower amounts.
o Whatever amount Parties choose to apply, it is important to recognize that these should be small
amounts compared to commercially traded quantities and must not be likely to affect human health
and the environment.

Q 91.D
 The Climate Neutral Now Initiative is one of several initiatives launched by the UNFCCC secretariat to
increase climate action by engaging non-party stakeholders (sub-national governments, companies,
organizations, and individuals). It was launched in 2015 based on a mandate to promote the voluntary
use of carbon market mechanisms recognized under the Convention.
 The Climate Neutral Now Initiative encourages and supports organizations to act now in order to achieve
a climate-neutral world by 2050, as enshrined in the Paris Agreement. It is a tool to promote additional
voluntary action on climate and to provide recognition for it.
38 www.visionias.in ©Vision IAS
 The initiative is not a certification scheme for its participants. An organization can become a participant
by signing the Climate Neutral Now Pledge, following the three steps (Measure, Reduce, Contribute) and
reporting on its actions and achievements annually.
 Hence, option (d) is the correct answer.

Q 92.B
 The chemical oxygen demand (COD) determines the amount of oxygen required for the chemical
oxidation of organic matter using a strong chemical oxidant, such as potassium dichromate under
reflux conditions. Hence, statement 1 is correct.
 This test is widely used to determine: a) the Degree of pollution in water bodies and their self-purification
capacity, b) the Efficiency of treatment plants, c) Pollution loads, and d) Provides a rough idea of
Biochemical oxygen demand (BOD) which can be used to determine sample volume for BOD
estimation. Hence, statement 3 is correct.
 The limitation of the test lies in its inability to differentiate between the biologically oxidizable and
biologically inert material and to find out the system rate constant of aerobic biological
stabilization. Hence, statement 2 is not correct.
 Most of the organic matter is destroyed when boiled with a mixture of potassium dichromate and
sulphuric acid producing carbon dioxide and water.
 A sample is refluxed with a known amount of potassium dichromate in sulphuric acid medium and the
excess of dichromate is titrated against ferrous ammonium sulphate.
 The amount of dichromate consumed is proportional to the oxygen required to oxidize the oxidizable
organic matter.

Q 93.A
 Recent context: Countries reinforced their commitment to combat Transnational Organized Crime at a
Ministerial Conference in Palermo, Sicily (Italy).
 The Palermo Convention, officially known as the "United Nations Convention against
Transnational Organized Crime," is an international treaty adopted by the United Nations in 2000.
It is supplemented by three protocols that address specific issues related to transnational organized
crime. These protocols are often referred to as the "Palermo Protocols."
o Protocol to Prevent, Suppress, and Punish Trafficking in Persons, Especially Women and
Children: This protocol, also known as the "Trafficking in Persons Protocol," addresses the
prevention and suppression of human trafficking, particularly the trafficking of women and children.
It outlines measures to be taken by countries to combat human trafficking and protect the rights of
victims.
o Protocol against the Smuggling of Migrants by Land, Sea, and Air: This protocol, also known as
the "Migrant Smuggling Protocol," focuses on the prevention and suppression of the illegal smuggling
of migrants, which is often associated with transnational organized crime. It aims to prevent and
combat migrant smuggling and protect the rights of smuggled migrants.
o Protocol against the Illicit Manufacturing of and Trafficking in Firearms, Their Parts and
Components and Ammunition: This protocol, known as the "Firearms Protocol," addresses the
illicit manufacturing and trafficking of firearms, their parts, and ammunition. It aims to prevent the
proliferation of illicit firearms and combat-related criminal activities.
 The Palermo Convention and its protocols collectively provide a framework for international cooperation
in combating transnational organized crime. They emphasize the importance of legal measures, law
enforcement cooperation, and the protection of the rights of victims in addressing various forms of
organized criminal activities, including human trafficking, migrant smuggling, and illicit firearms
trafficking.
 Hence option (a) is the correct answer.

Q 94.A
 Recent context : The Election Commission of India has designed in-house software for complete
Candidate and election management through ‘ENCORE’ which stands for Enabling
Communications on Real-time Environment.
 This provides a seamless facility for Returning Officers to process candidate nomination, affidavit,
Voter turnout, counting, results and data management.
 The ENCORE counting application is an end-to-end application for returning officers to digitize the votes
polled, tabulate the round-wise data and then take out various statutory reports of counting.
39 www.visionias.in ©Vision IAS
 Another application called ENCORE Scrutiny Application allows Returning Officers to do scrutiny of the
nominations filed by the candidates on-line.
o After verification of the nomination the status is marked as Accepted, Rejected or Withdrawn helping
the Returning Officer to prepare the final list of contesting candidates and assign the symbols.
 The National Technical Research Organisation is a technical intelligence agency of India. It was set up in
2004. The agency reports to the National Security Advisor and to the Prime Minister's Office.
 Hence statement 1 is correct but statement 2 is not correct.

Q 95.B
 The Ocean thermal energy conversion (OTEC) technology uses the temperature difference between
the cold water in the deep sea (5°C) and the warm surface seawater (25°C) to generate clean,
renewable electricity. Warm surface water is pumped through an evaporator containing a working
fluid. Hence, statement 1 is not correct.
 The vaporized fluid drives a turbine/generator. The vaporized fluid is condensed back to liquid in a
condenser cooled with cold ocean water pumped from deeper in the ocean. OTEC systems using seawater
as the working fluid can use the condensed water to produce desalinated water.
 The National Institute of Ocean Technology, an autonomous institute under the Union Ministry of
Earth Sciences (MoES) is establishing an Ocean Thermal Energy Conversion plant with a capacity
of 65 kilowatts (kW) in Kavaratti, the capital of Lakshadweep. It will be the first ocean thermal
energy plant in India. Hence, statements 2 and 3 are correct.

Q 96.A
 Global Tiger Forum
o Global Tiger Forum is an international intergovernmental body exclusively set up for
the conservation of tigers in the wild in the tiger range countries.
o Out of the 13 tiger range countries, seven are currently members of GTF: Bangladesh, Bhutan,
Cambodia, India, Myanmar, Nepal and Vietnam besides non-tiger range country U.K. The
secretariat is based in New Delhi, India.
o The other six tiger range countries but not members of GTF are China, Russia, Malaysia,
Indonesia and Thailand. Hence option (a) is the correct answer.
o GTF’s goal is to highlight the rationale for tiger preservation and provide leadership and a common
approach throughout the world in order to safeguard the survival of the tiger, its prey, and its habitat.
 Global Tiger Initiative (GTI): All 13 Tiger range countries are members of GTI
o The Global Tiger Initiative (GTI) is a worldwide alliance of governments, international organizations,
civil society, the conservation and scientific community, and the private sector dedicated to
collaborating on a common agenda to save wild tigers from extinction.
 Origins
o In 2008, the GTI was founded by founding partners the World Bank, Global Environment Facility,
Smithsonian Institution, Save the Tiger Fund, and the International Tiger Coalition (representing more
than 40 non-government organizations).
 Goals
o Double the wild tiger population by 2022.
o Conserve tiger habitats and corridors.
o Reduce human-tiger conflict.
o Enhance public support for tiger conservation.
 Achievements
The GTI has made significant progress in its efforts to conserve tigers, including:
o Increasing the global tiger population from 3,159 in 2010 to 3,949 in 2022.
o Expanding tiger habitat by over 10% since 2010.
o Reducing poaching by over 50% since 2010.
o Raising awareness of tiger conservation through public awareness campaigns and education
programs.
 Global Tiger Recovery Program (GTRP) was launched in 2010 under the Global Tiger Initiative (GTI)
by the World Bank to save wild tigers. This initiative set up the ambitious target of reversing the rapid
decline of wild tigers across their range and doubling their population numbers by 2022.

40 www.visionias.in ©Vision IAS


Q 97.B
 Carbon Credit Trading Scheme 2023 :
o The Indian Parliament passed the Energy Conservation (Amendment) Bill 2022, which modifies the
2001 Energy Conservation Act. The Ministry of Power notified the Carbon Credit Trading Scheme
2023 and will soon notify entities obligated to comply with GHG emission regulations. To ensure that
the identified entities’ percentage of total energy requirements come from non-fossil fuel sources, the
government will release the modalities based on the Bureau of Energy Efficiency (BEE)
recommendations.
o MoEFCC will notify the emission intensity target for obligated entities upon the recommendation of
the Ministry of Power. Emission intensity is the total amount of greenhouse gas emitted for every unit
of GDP.
Key Features of the CCTS 2023
 Cap-and-trade system:
The CCTS employs a cap-and-trade system, where a cap is set on the total amount of greenhouse gas
(GHG) emissions allowed from a group of entities. Entities covered under the scheme have emission
intensity reduction targets. Entities that achieve or exceed their targets earn carbon credit certificates,
while those that fall short need to purchase credits from the market.
 Trading mechanism:
Obligated entities will earn a carbon credit certificate if they surpass the target assigned to them. T he
certificate will be issued by BEE. Then the Carbon credit certificates are traded on designated power
exchanges. Hence statement 1 is not correct.
 Governance and oversight:
o The National Steering Committee for the Indian Carbon Market (NSCICM) oversees the overall
functioning of the carbon market.
o The Bureau of Energy Efficiency (BEE) will act as the administrator for the ICM, responsible for
developing GHG emissions trajectory and targets for obligated entities.
o The Grid Controller of India Limited will be the designated agency for maintaining the ICM
Registry and overseeing transactions among obligated entities.
o The Central Electricity Regulatory Commission (CERC) will act as the regulator for carbon credit
trading. They will register power exchanges for trading carbon credit certificates, protect buyer and
seller interests, and prevent fraud or mistrust. Hence statement 2 is correct.
Q 98.A
 Cocoyoc declaration was signed in October 1974 by an international group of social scientists, natural
scientists and economists. It was based on the theme of ‘Sustainable development’ and became the first
declaration to use the term. It also discussed themes of patterns of resource use, environment and
development strategies.
 Hence option (a) is the correct answer.

Q 99.C
 Basel Convention
o The Basel Convention is an international treaty that aims to control the transboundary movement of
hazardous wastes and their disposal. It was adopted in 1989 and entered into force in 1992. As of
today, there are 189 parties to the Basel Convention. Hence pair 1 is correctly matched.
o The Basel Convention's main objectives are to:
 Reduce the transboundary movement of hazardous wastes
 Ensure the environmentally sound management of hazardous wastes
 Protect human health and the environment from the adverse effects of hazardous wastes
 Hong Kong International Convention
o The Hong Kong Convention is an international treaty that aims to ensure the safe and
environmentally sound recycling of ships. It was adopted in 2009 and entered into force in 2015. As
of today, there are 15 parties to the Hong Kong Convention. Hence pair 2 is correctly matched.
o The Hong Kong Convention's main objectives are to:
 Prevent the uncontrolled disposal of ships
 Ensure that ships are recycled in a way that protects human health and the environment
 Promote the use of environmentally sound recycling technology
 Stockholm Convention on Persistent Organic Pollutants is an international environmental treaty,
signed on 22 May 2001 in Stockholm and effective from 17 May 2004, that aims to eliminate or restrict
the production and use of persistent organic pollutants (POPs). Hence pair 3 is correctly matched.
41 www.visionias.in ©Vision IAS
Q 100.D
 Climate-smart agriculture (CSA) is an approach that helps to guide actions needed to transform and
reorient agricultural systems to effectively support development and ensure food security in a changing
climate.
 CSA aims to tackle three main objectives:
o Adapting and building resilience of crops and livestock to climate change. Hence, statement 1 is
correct.
o Sustainably increasing agricultural productivity and incomes. Hence, statement 2 is correct.
o Reducing and/or removing greenhouse gas emissions wherever possible. Hence, help India is meeting
its Intended Nationally Determined Commitments under Paris Climate Agreement. Hence, statement
3 is correct.

Copyright © by Vision IAS


All rights are reserved. No part of this document may be reproduced, stored in a retrieval system or
transmitted in any form or by any means, electronic, mechanical, photocopying, recording or otherwise,
without prior permission of Vision IAS.

42 www.visionias.in ©Vision IAS

You might also like